Download as pdf or txt
Download as pdf or txt
You are on page 1of 159

HELPING YOU THROUGH YOUR CORE SURGICAL INTERVIEW 12th EDITION: 2023

Core Surgical Interview Guide


The Guide for the Core Surgical Interview
Core Surgical Training

Core Surgical Training in the UK is a two-year programme


designed to provide junior surgeons with general operative skills,
clinical experience of both acute and ward based surgical care,
and to introduce trainees to a range of sub-specialities. By the end
of CT2 you will have completed 24 months in surgery, a
prerequisite for progression to ST3 level.

The structure of training varies throughout the country but normally comprises of 4 or 6-month
rotations. These are themed in some deaneries such as in Manchester where a core trainee will do
around 18 months of one speciality and 6 months of another, or varied, like in London where you do
4-month rotations in CT1 and then two 6-month rotations in CT2, one of which will be the sub-
speciality of your choice.

At the end of Core Training you apply to Speciality Training, ST3-ST8, which leads to a Certification
of Completion of Training (CCT), allowing you to become a consultant.

Ranking for your CT1 job matters. If you get a high enough rank to
be given 2 or 3 of the sub-specialities of your choice, or indeed the
themed job of your choice, then you are setting yourself up with a
great chance at that ST3 number. If you just scrape into core training
then you have to take what you are given, and may not be able to do
your chosen specialty before ST3 interviews. Clearly you would be
at a disadvantage to the candidate above who has already got 6
months of experience and project completion. This is why we
encourage all applicants to put every effort into preparing for the
interview. It may be too late now to publish another paper, but it is
not too late to polish your interview technique and read up on the
topics commonly asked. The Core Surgical Interview Guide aims to make this process easier for you by
providing a structure with which you can answer questions, through in-depth discussion of the questions
that we were asked over the last few years.

We aim to demystify the core surgical application and provide relevant advice on how to prepare for
the interview. The stations provided here follow the actual format you will experience at interview,

CORE SURGICAL INTERVIEW GUIDE 1


www.surgicalinterview.co.uk
HELPING YOU THROUGH YOUR CORE SURGICAL INTERVIEW 12th EDITION: 2023

Introduction
National Recruitment

All core surgical training applications are now


centralised and run by via the www.oriel.nhs.uk
website. This means you only must submit one
application. This portal is now used for all specialty
applications. Important information specifically about
core surgical applications can be found at
https://medical.hee.nhs.uk/medical-training-
recruitment%20/medical-specialty-training

2011/12 was the first year that England, Wales,


Scotland and Ireland were accessed through a single
application process. In the application form you list
your deanery preferences. Candidates are guaranteed
one interview at their top preference deanery. If you
are successful, you will be contacted and requested to
sub-preference the programmes/job combinations available. Core surgery posts are allocated based on
your overall ranking. This is based on your interview score. However, if two candidates have the same
interview score, they are separated by their application form score. The application form therefore gives
the portfolio station panel another source of information to form their questions from but does not
necessarily contribute to your ranking.

The centralisation of the application process aims to make it fairer by standardising selection across
deaneries. The old boys’ network no longer influences who gets in. Today, each application form and
interview question must be validated to show that it truly is selecting out the best candidates.

The application form is relatively straight forward compared to what you will be asked at ST3 level.
However, it allows the deaneries to place you into general categories. Excellent, Good, Average, and Below
Average. Do everything you can to ensure that you are in the ‘excellent’ category on your application form
so that you are ahead of the competition as you walk into your interview.

Standardisation of the interview means that no matter where you are being interviewed, they should be
testing the same skill type, be that decision making, clinical judgment or academic achievement. Therefore,
you can learn from people who have been through it before you and predict the type of question you will
be asked. They change the stations each year, and during the interviews, from day to day, but practicing
similar stations is by far the best way to prepare for your interview. The panels have a moderately sized
question bank of validated questions that they vary subtly from interview to interview.

We have included sample interview questions here. They will not be exactly the same, as the deaneries
may change them, however, we encourage you to practice them more than once, to place yourself a step
closer to your CT1 job.

CORE SURGICAL INTERVIEW GUIDE 2


www.surgicalinterview.co.uk
HELPING YOU THROUGH YOUR CORE SURGICAL INTERVIEW 12th EDITION: 2023

SECTION 1: CLINICAL
Past stations with mock answers given at interview by candidates. Advice on how to approach the
question, communicate your knowledge effectively and supplementary clinical information to make
revising for the clinical station easy

SECTION 2: PORTFOLIO
Optimise your portfolio and make it stand out and impress your interviewers. Prepare for the new
format of the portfolio station including prepared presentation station. Supplementary questions from
recent interviews on your portfolio with advice on how to best answer them

SECTION 3: MANAGEMENT
The dreaded management station made simple. Past stations with useful structures that you can use
when faced with difficult questions. Our authors, who all previously went through the interview take
you through common questions, and advise you on how to score maximum points

ISBN: 978-0-9576100-3-3

ABOUT THE AUTHORS

We are a group of surgical trainees, working in London. We have all been through the Core Surgical
application process, all ranking highly, and all successful in getting our top choice jobs. We began
writing down some of the questions and themes that have come up in recent years, adding advice to
those applying in the future. The result is this Core Surgical Interview Guide. It did not exist when we
applied and the only way of getting this advice was by asking those who had gone. We hope it is
helpful on your journey.

CORE SURGICAL INTERVIEW GUIDE 3


www.surgicalinterview.co.uk
HELPING YOU THROUGH YOUR CORE SURGICAL INTERVIEW 12th EDITION: 2023

Applications Guide
Content Page

1. CLINICAL STATION 2. PORTFOLIO STATION

1.1 Introduction to the clinical station 2.1 Introduction to the portfolio station
1.2 Elderly woman with a fall 2.2 How to structure your portfolio
1.3 Painless haematuria 2.3 Writing the perfect Curriculum Vitae
1.4 Post operative pyrexia 2.4 ePortfolio
1.5 Wound dehiscence 2.5 2-minute Leadership Biography
1.6 Epistaxis 2.6 Publications, presentation, audit
1.7 Loin pain 2.7 Teaching and audit
2.8 Commitment to specialty
1.8 Bowel obstruction I
2.9 Research and audit
1.9 Bowel obstruction II
2.10 How to critique a paper
1.10 Burns
1.11 Slipped upper femoral epiphysis
1.12 Trauma
3. MANAGEMENT STATION
1.13 Hip dislocation
1.14 Breakdown of anastomosis
1 Introduction to the management station
1.15 Abdominal pain
3.2 Consultant appears drunk at work
1.16 Projectile vomiting 3.3 Conflict Resolution
1.17 Difficulty urinating 3.4 Dealing with colleagues
1.18 A painful leg 3.5 WHO checklist
1.19 A neck lump 3.6 Risk Stratification
3.7 NCEPOD
1.20 Shin pain
3.8 Sterilisation
1.21 Penetrating trauma
3.9 Decision Making
1.22 Post CABG complications 3.10 Rota dispute
1.23 Swollen testicle 3.11 Data Protection
1.24 Paediatric hernia 3.12 Nurse Practitioners
1.25 Epigastric pain I 3.13 European working time directive
3.14 Incidence and prevalence
1.26 Child with a fracture
3.15 Screening
1.27 Post operative confusion 3.16 Medical ethics
1.28 Wounds and dressings 3.17 Statistics
1.29 Epigastric Pain II 3.18 Clinical Governance
1.30 RTA 3.19 Consent
3.20 Taking the initiative
1.31 Post operative assessment
3.21 Revalidation
3.22 Cancelled theatre list

4. CONCLUSION

CORE SURGICAL INTERVIEW GUIDE 4


www.surgicalinterview.co.uk
HELPING YOU THROUGH YOUR CORE SURGICAL INTERVIEW 12th EDITION: 2023

SECTION 1:
Clinical Station

CORE SURGICAL INTERVIEW GUIDE 5


www.surgicalinterview.co.uk
HELPING YOU THROUGH YOUR CORE SURGICAL INTERVIEW 12th EDITION: 2023

1.1 Clinical Stations


Introduction

The clinical station of the interview process can


cause most angst for applicants. How can you
prepare for this without revising the whole of the
oxford handbook? It is worth the same weighting as
the management and portfolio stations. It is feared
because for some this will be one of the first
experiences of an oral clinical examination Whilst a
written examination allows you to sieve through
your thoughts to r e a c h a conclusion, oral
examinations pressurise the candidate into
immediately revealing those thoughts. The key to
this station is therefore to take a moment, answer
questions clearly and logically, and practice,
practice, practice. The course will polish your
answers to the clinical scenarios rather than teach
you the knowledge required so try to read all the
cases beforehand.

The clinical station consists of questions in a short case format, which form the basis of a general discussion
around the topic. The number of questions that you are asked will depend on how quickly you answer the
questions and anecdotally at least, being asked many, or very complex questions, especially towards the
end of your station, means that you have done well, and the panel are trying to differentiate a good
candidate from an excellent one.

The questions will test your clinical knowledge and should be scenarios that are commonly encountered by
a surgical SHO on call. It is unlikely therefore that you will be examined on very uncommon presentations,
injuries or diseases. You are assessed on the structure of your answers and your clinical knowledge.
Structured answers, even when lacking some of the more challenging clinical details or differentials, will
mark you out as someone who thinks logically and in an organised fashion. There is no guarantee that any
interviewee will have covered a particular specialty during their training to-date. The panel will be looking
for a candidate who can give accurate and confident answers, but also someone who can be structured and
logical in their reasoning and presentation, as well as importantly, demonstrating that they are a ‘safe’
doctor.

In one of the author’s interviews, he was asked about orthopaedics, a speciality he had no experience of.
Despite this he did well as he was able to answer the questions in a sensible structured way.

The interviewers are looking for logical, structured thinkers, who will be able to cope with the demands of
a surgical career, rather than a preformed surgical registrar. That said, practicing the clinical stations that
commonly come up should allow you to prepare. We have included some of them here.

CORE SURGICAL INTERVIEW GUIDE 6


www.surgicalinterview.co.uk
HELPING YOU THROUGH YOUR CORE SURGICAL INTERVIEW 12th EDITION: 2023

1.2 An elderly woman with a fall

You are called to A&E to see an 88 year old woman who was found lying on the floor halfway down
the corridor in her nursing home. At baseline, she has a poor level of mobility, only being able to
mobilise around 15 yards with zimmerframe. She does not recall having a fall, but another resident saw
her trip and fall, holding her left hip afterwards. She currently complains of pain in this hip and on
examination the hip is shortened and externally rotated. She is in some pain at rest, and this increased
on attempts at passive movement of the hip. On your general inspection, you note that she has a
pacemaker in situ.

The following radiograph was performed by A&E.

What would be your assessment of this patient’s injury and her x-ray?

The main differential diagnosis consistent with this presentation is a fracture of the neck of femur and
although I want to see two views, the radiograph presented is consistent with an intertrochanteric
fracture of the left femur.

Supplementary note

The history could also suggest an anterior dislocation of the hip, which could cause a restricted range
of motion and pain on passive and active movement. However, only 10% of dislocations are anterior
and whilst the limb is externally rotated, it tends not to have the same degree of external rotation as in
cases of neck of femur fracture. A posterior dislocation, which would be more commonly sustained after
this type of injury, the femur would be shortened, but internally rotated, unlike this case.

What are the risk factors of this kind of injury?

This injury seems to have been associated with minor


trauma according to the collateral history, which in the
majority of cases points to someone with weakened bone.
(Most hip fractures in patients with physiologically normal
bone are the result of high velocity injury). The most
common bony weakness is due to osteoporosis, followed
by metastatic deposits, metabolic bone diseases such as
Paget’s and osteomalacia and more rarely, osteomyelitis.

How would you manage this patient initially?

I would assess and manage this patient according to the ATLS ABCDE protocol, making patient safety
my primary concern. She should receive adequate analgesia according to the WHO pain ladder. This
is likely to include the prudent use of opiates, taking into account her age and co-morbidities. I would
also ensure that my patient was suitably fluid resuscitated using either a colloid or crystalloid solution
depending on local guidelines. At this time, she should have two wide bore cannulae in situ, bearing in

CORE SURGICAL INTERVIEW GUIDE 7


www.surgicalinterview.co.uk
HELPING YOU THROUGH YOUR CORE SURGICAL INTERVIEW 12th EDITION: 2023

mind the blood losses that can occur from a femoral fracture. The neurovascular status of the leg should
be assessed and documented before any interventions take place. I would also make this patient ‘nil by
mouth’, in anticipation of a surgical intervention, at this time I would ensure she had maintenance fluid
prescribed. In the meantime, her leg should be placed in a ‘foam gutter splint’ for comfort and to reduce
the risk of pressure sores. As she will have reduced mobility for a period of time, she should have
mechanical and chemical thromboprophylaxis; TED stocking and low molecular weight heparin, unless
contraindicated.

What investigations would you order for this patient and what role would they have in
your management?

Important bedside investigations include a urine dip to check for signs of a urinary tract infection, which
could precipitate a fall in the elderly. I would order two views of the affected bone or joint. In this case,
I would order an AP pelvis, and a lateral right hip x-ray and full length femoral views. Pre-operative
work up of this patient would include full history and examination, importantly determining the nature
of the fall (mechanical or secondary to an underlying pathology), any co-morbidities, and the
identification of any additional injuries. I would take routine bloods and add a clotting screen and a
cross match of two units of blood. Depending on the length of time the patient was on the floor, a
creatinine kinase may be indicated. I would also order an ECG and CXR as this would help in my
assessment of this patient’s anaesthetic risk and identification of underlying pathology that may impact
on their ASA classification. I would also request a pre-operative pacemaker check.

What would be the surgical management options for an intertrochanteric fracture?

I would manage this type of injury with a dynamic hip screw, followed by fluoroscopic images taken
throughout the repair to ensure the maintenance of the fracture in satisfactory reduction and proper
positioning of the fixation device.

Supplementary information

Intertrochanteric fractures are inferior to the joint capsule and lie in the plane between the greater and
lesser trochanter. Despite a reportedly acceptable rate of healing rate with conservative methods,
surgical intervention has replaced previously prevalent methods of prolonged bed rest and traction in a
spica cast, each of which also carry their own risks. Surgical intervention is now most common.

What additional complications can result from intracapsular fractures of the femoral
head and how would you manage one of these injuries?

The femoral head in the adult has a retrograde blood supply with blood vessels running from the capsule
and along the femoral neck. Displacement of the femoral head and traumatic disruption of the capsule
can cause a disruption of this vasculature. This can lead to the problems of avascular necrosis of the
femoral head, as well as malunion.

Treatment options vary, depending on the age and mobility level of the patient. I would take a full
patient and collateral history concerning premorbid state and level of function. If she had limited
mobility before sustaining the fracture, I would look to perform a hemiarthroplasty. I would have a
choice of cemented and uncemented methods of securing the implant to the femoral shaft. This method
would allow early mobilisation once her pain score allowed it and I would ensure that appropriate post

CORE SURGICAL INTERVIEW GUIDE 8


www.surgicalinterview.co.uk
HELPING YOU THROUGH YOUR CORE SURGICAL INTERVIEW 12th EDITION: 2023

operative analgesia was prescribed to this end. Functional outcome of hemiarthroplasty in patients with
limited mobility tends to be satisfactory. I would ensure that thromboprophylaxis was not forgotten, as
the risk of venous thrombosis, possibly with progression to

pulmonary embolism, would be considerable.

How would your management differ in a younger, more active patient?

Intracapsular injuries in young patients would be more commonly associated with high impact injuries
such as RTA and sporting injuries. My initial management would be to assure haemodynamic stability
using an ABCD approach, with assessment of the airway, breathing and circulation before consideration
of bony or ligamentous injury. I would proceed to open reduction and internal fixation once the patient
was cardiovascularly stable. I would also note that the incidence of AVN in younger patients are
considerable, with some studies noting rates of up to 20% and ensure that this was explained to the
patient and included on their consent form.

Supplementary Information

You should be familiar with Garden’s classification of fractures. This is


the classic method of neck of femur

fracture description, although it is becoming outdated

Type I: stable fracture with valgus impaction

Type II: Non displaced, complete fracture

Type III: Displaced fracture with maintenance of ‘end


to end’ contact between the two bony fractures

Type IV: Completely displaced fracture with no contact


between bony fragments Type III and IV are associated
with an increased incidence of AVManagement can be
remembered easily - 1,2 screw (fixation with a cannulated
screw), 3,4 Austin Moore (or rather they will normally
require hemiarthroplasty to avoid AVN)

Sub-trochanteric fractures occur through or distal to the trochanteric line in a transverse orientation.
These are not manageable by plate fixation but are commonly managed with intramedullary nailing.
Rehabilitation and thrombosis prophylaxis are as previously described.

Advice

The danger this question, where you may have some knowledge and experience, is the temptation to
tell the examiner all that you know about neck of femur fractures in general, rather than answering the
question posed. In this example, the case surrounds a plain film of an intertrochanteric fracture and the

CORE SURGICAL INTERVIEW GUIDE 9


www.surgicalinterview.co.uk
HELPING YOU THROUGH YOUR CORE SURGICAL INTERVIEW 12th EDITION: 2023

specific management of this injury. You should answer the question you are asked and then stop. If they
want more the examiner will ask further questions. It is best to be guided by them, rather than trying to
include all the knowledge you in an answer, as you need to focus on where the points are. Examiners
are generally helpful and will ask you a more focused question if you have not scored all the points.

One strength of this answer is the appropriate exhibition of further reading of current literature, with
regards to the rates of AVN in young patients with neck of femur fractures. If you can include something
similar in your answer you will mark yourself out as one of the top candidates. Only do so if it flows
naturally from your answer. However, do not forget the importance of covering the important aspects
of what you actually would do as an SHO; a full history and examination, ensuring that the patient is
haemodynamically stable, appropriate investigations , prescription of analgesia and LMWH etc... These
are equally important, give structure to your answer, and show that you have actually been on the wards
and will be a safe surgeon.

Here we have included extra information in the answers for your information. The examiner may
wish to have a more unstructured discussion about a topic, especially if it concerns his/her specialty.
In this case, the examiner is probably trying to stretch you, and it is your opportunity to distinguish
yourself as an excellent, rather than just a good candidate.

CORE SURGICAL INTERVIEW GUIDE 10


www.surgicalinterview.co.uk
HELPING YOU THROUGH YOUR CORE SURGICAL INTERVIEW 12th EDITION: 2023

1.3 Painless Haematuria


You are asked to see a 69-year man who has been referred from his GP with painless macroscopic
haematuria with some clots. He denies urinary frequency or dysuria, but has noticed around 7kg of
unintentional weight loss over the past 4 months. He worked in a chemicals factory for 25 years but is
now retired. He has an 80-pack year history.

What would be your differential diagnosis?

Painless haematuria in an individual of this age, and with a history of unintentional weight loss often
suggests an underlying malignancy; Frank haematuria has a 20% rate of revealing a urological
malignancy. My main differential would be bladder cancer, especially given the additional detail about
the gentleman having worked in a chemicals factory for 25 years. His history of smoking and
occupational exposure to chemicals such as amines from the textiles industry are known risk factors for
transitional cell bladder cancer. Notably, 90% of bladder cancers present with haematuria.

I would also consider renal carcinoma. This would classically be associated with loin pain +/- an
abdominal mass and I would take a more detailed history and fully examine the patient to fully assess
this likelihood. There is a possibility that this could be prostatic in origin or related to a bladder stone,
but these often would be related to microscopic, rather than macroscopic haematuria. Cystitis would
also be a possible cause of this bleeding, but was be less likely given the patient’s gender, as well as his
lack of dysuria and urinary frequency.

Pain is usually associated with an inflammatory process and its absence in this case would make a renal
or bladder stone less likely and the length of the history together with weight loss point away from
infection as a cause.

How would you examine this patient?

Patient safety is my primary concern, and I would assess the patient clinically using the ABCDE
protocol. I would determine the extent of blood loss and ensure that the patient was haemodynamically
stable. I would complete with an examination to exclude abdominal and suprapubic masses and a digital
rectal examination to examine the prostate. One would also palpate for the bladder to assess for
retention, as blood clots (or a tumour) may cause an outflow obstruction. In this case a wide bore urinary
catheter with the option for saline irrigation may be indicated.

What investigations would you arrange for this patient?

I would perform a bedside urine dipstick test to confirm the presence of blood and subsequent urine
microscopy test to rule out infection. Microscopy may also identify malignant cells.

(Urine tests allow an initial assessment of the risk of urological malignancy, however overall sensitivity
is less than 75% for medium and low grade tumours. CT KUB, or intravenous urogram (IVU) in
some hospitals, will identify the structure and function of the urinary tract, as well as identifying stones
in either ureter, while ultrasonography will allow visualisation of the renal parenchyma.

CORE SURGICAL INTERVIEW GUIDE 11


www.surgicalinterview.co.uk
HELPING YOU THROUGH YOUR CORE SURGICAL INTERVIEW 12th EDITION: 2023

Flexible or rigid cystoscopy under general anaesthetic currently represents the gold standard in
assessment of the structure of the bladder and would allow identification and treatment of a bleeding
site, along with a biopsy or resection if necessary for histological diagnosis.

If a tumour was identified in the urological tract, a computed tomography (CT) staging scan would be
appropriate to assess the extent of muscular invasion and any metastatic spread.

What would be the treatment if bladder cancer was identified?

It is important to involve the MDT in all cases of malignancy, and of course, all options should be
discussed with the patient. Following appropriate imaging, a transurethral excision may be carried out.
CT scanning would allow staging and identification of distant metastases, with further treatment options
dependent on staging.

Supplementary information

Low grade tumours are treated by transurethral resection of bladder tumour (TURBT) followed by long
term outpatient cystoscopy screening. High grade tumours are also managed by TURBT if possible,
followed by chemotherapy, for instance single dose of intravesical chemotherapy or a weekly dose for 6
weeks after surgical procedure. All such decisions are made after discussion between surgical and
oncological teams in the uro-oncology MDT.

Invasive bladder cancer (T1-T4) can be managed by partial or radical cystectomy with pelvic
lymphadenectomy and urinary diversion (either continent or incontinent), external beam bladder
irradiation or systemic chemotherapy. Metastatic bladder cancer (as identified by CT scan) would be
primarily managed by a cisplatin based chemotherapy regimen. It would be essential to start this as
soon as possible, as untreated metastatic cancer has a 2-year survival rate of less than 5%.

How would you decide which patients to refer to urology?

I would refer any patient above the age of 50 years presenting with microscopic haematuria or
patients of any age with frank haematuria for specialist urology opinion to exclude the possibility of
urological malignancy

Supplementary Information

Staging of Bladder Cancer

The classical TNM system can be combined to calculate an overall cancer stage. Bladder cancers are
staged from I to IV, from least to most serious.

Stage

Stage 0a Non invasive papillary carcinoma with no invasion into bladder connective tissue or the muscle of the

CORE SURGICAL INTERVIEW GUIDE 12


www.surgicalinterview.co.uk
HELPING YOU THROUGH YOUR CORE SURGICAL INTERVIEW 12th EDITION: 2023

(T0, N0, M0) bladder wall. No lymph node or distant spread

Stage I Carcinoma spread into the layer of connective tissue under the lining layer of the bladder but no
(T1, NO, M0) bladder invasion. No lymph nodes or distant sites.

Stage II Carcinoma spread into bladder muscle wall, but no breach of the muscular layer into fatty layer
(T2a or T2b, surrounding the bladder. No lymph nodes or distant sites
N0, M0)

Stage III Carcinoma spread through the bladder muscular wall into surrounding fatty tissue. Possible prostatic,
(T3a, T3b, uterine and vaginal spread. No pelvic or abdominal wall spread. No lymph node or distal spread.
T4a, N0, M0)

Stage IV (T4b, N0, M0): Carcinoma growth through bladder muscular wall and into the pelvic or abdominal
wall. No lymph node or distal spread.
(Any T, N1-3,M0) Carcinoma spread to nearby lymph nodes (N1-3), but no distal spread
(Any T,any N,M1) Spread to distal sites, such as lungs, liver or bone (M1)

Advice

You do not need to remember the details of the TMN classification of bladder cancer, but be aware of
how if influences management. This answer concentrates on the differential diagnosis as appropriate
for the history. When answering questions on differential diagnoses it is important to be specific, and
not list every differential you know. However, concentration on the details of the case, namely the
occupational and smoking history, as well as gender, allows you to be more specific. While cystitis is a
legitimate cause of macroscopic haematuria, to mention it before urological malignancies would show
an absence of logical thinking and perhaps would signal that if you were faced with this case as an
admitting CT1 doctor, there could be some delay in initiating necessary investigations to identify and
treat malignancy.

CORE SURGICAL INTERVIEW GUIDE 13


www.surgicalinterview.co.uk
HELPING YOU THROUGH YOUR CORE SURGICAL INTERVIEW 12th EDITION: 2023

1.4 Post operative pyrexia

You are asked to see a 67 year old with a 40 pack year smoking history and a previous diagnosis of
COPD. He is one day post repair of a strangulated, indirect, right inguino-scrotal hernia. He has a
productive cough and a temperature of between 37.9 degrees and 38.6 degrees over the past 8 hours.
He is tachycardic at a rate of 110bpm and has a respiratory rate of 21 with saturations of 96%. His pain
score is 5/10.

What is the most likely cause of this gentleman’s tachypnoea, pyrexia and tachycardia?

The most likely cause is a postoperative pulmonary atelectasis secondary to the accumulation of mucus
secretions in the bronchial tree. This is a risk after any intra-abdomial or thoracic surgery, where
coughing will exacerbate pain by increasing intra cavity pressures. The risk is magnified in patients with
pre-existing pulmonary disease. Pyrexia so soon after surgery is unlikely to be due to wound infection,
however a pneumonia, pulmonary embolism and DVT should be considered. In addition, other sources
of infection such as a UTI, perhaps from an indwelling catheter, cellulitis, pressure sore or other open
wounds should be in one’s mind, and a comprehensive examination would aim to identify these.

Which factors could be responsible for these complications?

Pre-operative factors include his pack year history and prior diagnosis of COPD which would contribute
to excess mucus production and collection in his bronchial tree and would reduce his functional
respiratory reserve and effective alveolar exchange surface. A smoking history also makes him more
susceptible to infection.

Peri-operative factors, such as the gases used in a general anaesthetic and intubation irritate the
respiratory mucosa and increase mucus secretion from mucosal goblet cells, as well as a small amount
of oedema as a post inflammatory response to the endotracheal tube. The muscle relaxant used in
general anaesthesia can reduce post operative inspiratory effort. Being mechanically ventilated causes
alveolar barotrauma, making one more susceptible to alveolar collapse. Lying prone for the operation
can result in a ventilation/perfusion mismatch, increasing the likelihood of atelectasis. If the procedure
involved laparoscopy, then the insufflation of CO2 results in a splinting of the diaphragm and reduced
ventilation to the lung bases.

Post operatively, the pain of the groin incision, lying prone, increased sedation and reduced mobility
could inhibit clearance of the accumulated secretions, increasing the likelihood of alveolar collapse. In
addition, the chances of developing aspiration pneumonia, are increased. This most commonly occurs
5 days post operative

Advice

It is appropriate to group risk factors into pre-operative, operative and post operative providing a good
structure to hang your answer on. Remember: “categorise to survive!”

CORE SURGICAL INTERVIEW GUIDE 14


www.surgicalinterview.co.uk
HELPING YOU THROUGH YOUR CORE SURGICAL INTERVIEW 12th EDITION: 2023

How would you examine this patient and how would you proceed to management?

I would start by ensuring that the patient was haemodynamically stable with assessment of the airway,
breathing and circulation following ALS principles. If the patient was speaking to me, I would consider
his airway as being patent. I would percuss and auscultate the chest and carefully inspect for use of
accessory muscles, signalling respiratory distress. I would assess the patients pulse and BP and look for
a raised JVP, which could be a sign of congestive cardiac failure secondary to an MI.

If I was worried about the patient’s current clinical state, I would contact my registrar, preferably from
the operating team, to review the patient. I would ensure that the patient had two wide more cannulae
in situ, with fluids running as necessary. I would request a chest x-ray and review the most recent blood
results, including inflammatory markers and sputum culture for the patient. I would also take an arterial
blood gas (ABG) and send blood cultures and an MSU and blood cultures. It is important to send these
before starting any antibiotic therapy.

I would monitor the patient’s saturations and if below his pre-morbid baseline in the medical notes,
would start low dose oxygen by nasal cannula, taking care of the history of COPD and the importance
of not suppressing the hypoxic drive and worsening his symptoms. In the acute setting I would prioritise
satisfactory oxygenation, but monitor for CO2 retention by repeating an ABG 30 minutes after
commencing oxygen.

If I suspected pulmonary embolism was the diagnosis, I would organise a CTPA. (A d-dimer blood test
would be of limited use as a raised result may represent generalised inflammation post operatively,
rather than a thromboembolism.)

It is important to involve the respiratory physiotherapy team early in this patient’s management.
Vigorous breathing exercises and chest percussion would clear secretions. I would review his analgesia
and if necessary liaise with the pain team, as managing his pain more effectively encourages a more
effective cough. This could be managed by regular oral analgesia or patient controlled analgesia (PCA)
as required. Finally, if the blood results or cultures suggested any signs of infection, I would commence
antibiotics according to local protocol, after taking appropriate microbiology guidance.

If I were particularly concerned about his oxygenation, it may be prudent to contact the ITU
outreach team or anaesthetic registrar for advice ± review.

Advice

It is important to answer the question asked of you. On this occasion the question is “what is the most
likely cause of this gentleman’s symptoms”. Therefore begin your answer by saying the most likely
cause and your reasonings. You can then go on to talk about important differential diagnoses to
consider, and the examiner can stop you if this won’t score you any points.

This is a fairly typical presentation that will be managed initially by many FY1s post operatively. The
difficulty in this question is answering in a systematic manner. It is useful when asked about
complications post surgery to categorise your answer. In this case, we have used pre-operative (intrinsic
to the patient him/herself), peri-operative and post operative. This lets the examiner know that you are

CORE SURGICAL INTERVIEW GUIDE 15


www.surgicalinterview.co.uk
HELPING YOU THROUGH YOUR CORE SURGICAL INTERVIEW 12th EDITION: 2023

a logical thinker, and acts as a memory aid. You will find that you build up a bank of these aide memoirs
and that your answers become more structured, the more you practice using them. There are more
examples of these throughout this guide. Do not forget to mention that you would contact your seniors
in a situation such as this. It may sound straightforward, but it is good practice to contact senior support
early on, even if you will be managing the case mostly by yourself in the early stages, your seniors should
be aware of any concerning cases on their take. The multidisciplinary team has become an integral part
of our practice, so mention it when appropriate, including keeping the ward nurses up to date so that
they can help you hang fluids, will take more regular observations etc... It shows that you are on your
way to becoming a rounded surgeon and a good team member.

One of the key factors the examiners are looking for is a good SHO who is not only being able to
manage cases with the knowledge that you have, but also to be able to call in the help of other health
care professionals when needed and to be aware of limitations.

CORE SURGICAL INTERVIEW GUIDE 16


www.surgicalinterview.co.uk
HELPING YOU THROUGH YOUR CORE SURGICAL INTERVIEW 12th EDITION: 2023

1.5 Wound dehiscence


You are asked to see a 67-year-old retired architect who had a right hemicolectomy through a right
paramedian incision for a carcinoma of the caecum. Post operatively, he had a prolonged period for
paralytic ileus requiring passage of an NG tube, and IV fluids. He passed flatus on day 6 post
operatively. However, despite this, his notes suggest ongoing abdominal distention. On day ten post
operatively, after transfer to a general surgical ward, he has a period of coughing and called the nurses
because of pain in his abdomen and some pink fluid coming from his wound. You are called to assess
him.

What would you do?

Over the phone I would ensure that the essential initial steps had been taken, asking for the observations,
the patient’s symptoms and his current state, and assess what level of input was needed. If he were
deteriorating, I would ask them to get further help, for instance putting out a medical emergency call if
appropriate, before immediately attending the patient.

(You can immediately set yourself apart from the competition by picking up on the cue that
you are being called to see him. Therefore you can take the opportunity to show that you
would want to get an impression of the patients current state and the urgency of your review,
as well as instigating a plan for the ward staff while they wait for you to get to the ward.)

I would proceed by assessing the patients airway, breathing and circulation,


ensuring they were haemodynamically stable. , I would reassure the patient
and ensure that appropriate analgesia was prescribed. I would contact my
senior colleagues as this patient has signs of wound dehiscence and
potentially needs to be returned to theatre for wound exploration under a
general anaesthetic.

(The ‘pink fluid sign’ described here is produced by serous peritoneal exudate, tinged with blood and hence pink in colour
which oozes through a breaking down abdominal wound, sometimes for a number of days. It is a warning sign before
complete wound dehiscence.)

You leave the patient to try and contact your registrar. However, you are then called to a cardiac arrest
which you have to attend. You return to the patient 90 minutes later to find a distressed patient
complaining of a ‘lump coming from his stomach. The patient is in moderate discomfort.

How would you proceed?

I would re-assess the patients airway, breathing and circulation and treat as required. I would reassure
the patient whilst examining the abdomen to determine.

I would give the gentleman adequate analgesia together with an antiemetic to prevent opiate induced
retching that could result in more bowel escaping from the abdomen. I would ensure my registrar and

CORE SURGICAL INTERVIEW GUIDE 17


www.surgicalinterview.co.uk
HELPING YOU THROUGH YOUR CORE SURGICAL INTERVIEW 12th EDITION: 2023

consultant were aware and prepare the patient for theatre, liaising with theatre staff, ward staff and
the anaesthetist team.

Would you attempt to reduce the bowel on the ward and if so, how would you do this?

I would not attempt to reduce the bowel. The rigidity of the abdominal wall would not allow this, it
may cause damage to the bowel and it may be extremely painful. I would cover the exposed viscera
with a sterile towel or large dressing soaked in warm normal saline and keep this in place with a bandage
if necessary. I would then fast-bleep my registrar immediately and ready the patient for surgery. This
would include ensuring that a valid group and save was available, that the patient was consented for
theatre, and that the time of last eating and drinking was documented. This patient needs immediate
transfer to theatre.

What factors could be responsible for this emergency?

A numbers of factors could impact optimal wound healing. These include pre-operative, peri-operative
and post operative factors.

A patient with pre-existing co-morbidities, such as anaemia, jaundice, malignancy, vitamin C


deficiency, diabetes or protein deficiency would have impaired skin healing and a higher risk of wound
breakdown. Smoking is also an important risk factor for poor wound healing.

Peri-operatively, poor surgical technique or surgical error can play a significant role. Sutures can be
placed too close to the wound edge, with poorly tied knots, the suture ends cut too close to, or indeed
cut through the knot. Increased bowel handling intraoperatively can increase oedema of the bowel
and intra-abdominal pressure, putting tension on the closure. Any leakage of bowel contents, or other
contamination of the wound, will impair wound healing.

Post-operative factors increasing the risk of wound dehiscence include patients with chronic cough, for
example, those with chronic bronchitis, increase their intra-abdominal pressure and cause increased
tension on suture lines. The same would apply to patients with high body mass index, and patients
with constipation. Infection at the surgical site is also an important factor in poor wound healing.

What is Jenkins rule?

This is a rule for closure of the abdominal wound. It states that for a continuous suture, the length of
suture used should be at least four times the length of the wound with sutures 1cm apart and with 1cm
bites of the wound edge.

How should this wound be repaired?

The patient should be placed in the supine position under general anaesthesia. Firstly any non viable
tissue (including peritoneum and rectus fascia as necessary) around the wound edges should be debrided.
Repair should be carried out using interrupted nylon sutures, which are passed through all layers of the
abdominal wall, all layers of the rectus fascia muscle and peritoneum, but not including the skin.

CORE SURGICAL INTERVIEW GUIDE 18


www.surgicalinterview.co.uk
HELPING YOU THROUGH YOUR CORE SURGICAL INTERVIEW 12th EDITION: 2023

including the skin. The sutures should be held open until all are inserted, as this improves the accuracy
of suture placement and then each one is tied one after the other in series, taking care not to damage
underlying viscera.

If the deep layer of the abdominal wound gives way, but the skin sutures stay intact,
what is the diagnosis?

An incisional hernia

Supplementary information

Numerous studies have shown the benefit of the ‘mass closure’ technique of suturing the abdominal
wound. All layers of the abdominal wall apart from the skin and subcutaneous tissue are picked up a
minimum of 1cm from the wound edge on either side and the sutures inserted 1 cm or less apart. The
skin is closed as a separate layer. This gives the maximal strength to the abdominal wound, reducing
the risk of dehiscence. The suture material to repair the abdominal wound should be non absorbable
(e.g.: nylon) or only slowly absorbable (e.g.: PDS) material and should not be too fine. Size 1 is commonly
used.

Advice

The supplementary information here is useful knowledge, but not essential to answer the question.
The question about ‘Jenkins rule’ again was meant to stretch the candidate here and you would not
necessarily be expected to know that. The key again here is structure. You can see that this answer
benefits from using the distinction between pre and peri/post operative risk factors and this gives the
answer some direction. It may seem repetitive to keep answering questions with basic life support
diktats and in many cases, your examiner will let you skip over that to the specifics of the question.
However, it shows that above all, you are safe. Again, notice the early involvement of senior help.
This patient will need surgical intervention and whilst your role is not necessarily going to be to
provide that help directly yourself, especially in the early stages of your training, the good CT1 will
facilitate the most optimal management for the patient as quickly as possible.

CORE SURGICAL INTERVIEW GUIDE 19


www.surgicalinterview.co.uk
HELPING YOU THROUGH YOUR CORE SURGICAL INTERVIEW 12th EDITION: 2023

1.6 Epistaxis
You are called to see a 55-year-old office manager, who is on warfarin for atrial fibrillation. He presents
to A&E with profuse epistaxis. He is a known hypertensive and has a history of congestive cardiac
failure. Examination shows active bleeding from the right nostril.

What is the likely differential diagnosis and what further features of the history are
important?

There are local and systemic causes, but the majority of cases of epistaxis have no underlying pathology.
and are secondary to trauma, often minor trauma at the area of congenitally fragile blood vessels known
as Littles area (Kiesselbach's plexus). Pathological causes include impaired coagulation, secondary to
drugs such as warfarin or antiplatelet agents, diseases of the bone marrow or liver. Although local
trauma (or a nasal neoplasm) could present with unilateral epistaxis, this would not be profuse unless it
disrupted the wall of an artery.

I would ask about previous episodes of epistaxis and whether he had a history of bruising or bleeding
elsewhere on his body. I would also query whether he had a history of local trauma which could explain
the episode.

How would you proceed?

My priority would be to arrest the haemorrhage and stabilise the patient. I would start by assessing this
gentleman’s airway, breathing and circulation as appropriate. I would immediately resuscitate the
patient if I believed him to be clinically shocked. This would involve assessment of his pulse, blood
pressure, respiratory rate and an estimation of blood loss. If the patient was shocked, I would ask a
member of the nursing staff to put firm pressure on the cartilaginous section of the nose using my finger
and thumb, whilst I alerted my senior registrar to the details of the case and contacted the ENT registrar.
I would then establish intravenous access, take blood including FBC, clotting and group & save and
start to resuscitate the patient if necessary using colloid or crystalloid solutions.

If the patient was not shocked, I would sit him up, with his head titled downwards to prevent blood
passing posteriorly. I would put firm pressure on the cartilaginous section of the nose using my finger
and thumb. After a period of 10-15 minutes, I would pack the nose with a nose pack or simple gauze if
no other material was immediately available. Ideally I would use a substance such as bismuth iodoform
paraffin paste under local anaesthetic cover.

My first priority it to ensure that the patient is safe. This involves assessment using the ATLS ABCDE
protocol. If I were concerned about the safety of his airway due to the flow of blood I would put him
flat in the head down position to reduce blood flow into the airway and contact the anaesthetists. I
would also contact my senior and the ENT registrar in this situation.

I would assess the level of blood loss and determine whether he was clinically shocked. If this were the
case I would contact my senior. I would ensure that he had two wide bore cannulae in situ, taking

CORE SURGICAL INTERVIEW GUIDE 20


www.surgicalinterview.co.uk
HELPING YOU THROUGH YOUR CORE SURGICAL INTERVIEW 12th EDITION: 2023

blood including FBC, clotting and group & save and start to resuscitate the patient if necessary using
colloid or crystalloid solutions.

If appropriate, direct pressure over the cartilaginous section of the nose using finger and thumb can
help to reduce blood flow. An ice bag on the forehead at the top of the nose can help to promote
coagulation and reduce blood flow. If the patient were not clinically shocked, then sitting him up, with
the head leaning forwards helps to prevent blood from flowing posteriorly.

I would want to put nasal packs in place to reduce the bleeding if simple measures did not work. Ideally
I would use a substance such as bismuth iodoform paraffin paste under local anaesthetic cover. If a
single bleeding point were identified, this could be chemically cauterised using a silver nitrate stick, but
this is unlikely to be successful in a patient with a pathological bleed, who is bleeding profusely, as in
this case.

Would you need to admit this patient?

All patients with packing would require admission. They are at risk of hypoxia, and inhalation of
packing substances, and therefore it is advisable for them to be in hospital where they can be monitored.
This would also be a chance to investigate the underlying cause of the bleed; in this case it is likely to be
due to an elevated INR. The patient may need surgical intervention to stop the epistaxis.

His INR result comes back at 4.5. How would you manage this?

In a patient who has a raised INR and is acutely bleeding, it is important to consider the reason that
they are taking the warfarin (e.g. AF vs mechanical heart valve), and the relative risks and benefits of
stopping and/or reversing anti-coagulation. For this reason, it would be advisable to discuss the case
with the haematology registrar, and my senior.

I would stop the warfarin, and could give vitamin K orally or IV to reverse the effects of the drug. In
severe bleeding this can be supplemented with fresh frozen plasma, prothrombin complex concentrate,
or recombinant factor VIIa, depending on the urgency of the situation.

If the patient were acutely bleeding I would stop the warfarin, immediately reverse it using
prothrombin complex concentrate (Beriplex/Octaplex) or fresh frozen plasma after discussion with
the haematology registrar on call. I would get further advice from the ENT registrar

Supplementary Information

Anterior epistaxis is commonly septal. It is invariably from the area of anastomosis between the
sphenopalatine, facial and ethmoidal arteries, known as Little’s area (Kiesselbach's plexus).

The first line for investigation for anterior epistaxis is rhinoscopy, performed by ENT specialists in most
units. Posterior epistaxis is not normally easy to locate by this method. The gold standard is to visualise
the bleeding point by nasendoscopy and then to treat directly, e.g. with bipolar cautery.

CORE SURGICAL INTERVIEW GUIDE 21


www.surgicalinterview.co.uk
HELPING YOU THROUGH YOUR CORE SURGICAL INTERVIEW 12th EDITION: 2023

If the bleeding point cannot be found, the nose is generally packed with gauze, which is then left for 48
hours. Prophylactic antibiotics should be considered if the pack remains in situ for more than 48 hours.
Toxic shock syndrome is a potential sequelae.

Advice

It is important to realise that epistaxis can be fatal and should be taken seriously. You must discuss the
potential for hypovolaemic shock in this case, as well as your method of resuscitation. It would be fair
to say ‘I would stop the bleeding and then contact ENT specialists, especially if the patient is on warfarin.
However, some knowledge of the materials and methods involved in packing epistaxis and the anatomy
of Little’s area will be more than most candidates are able to achieve.

CORE SURGICAL INTERVIEW GUIDE 22


www.surgicalinterview.co.uk
HELPING YOU THROUGH YOUR CORE SURGICAL INTERVIEW 12th EDITION: 2023

1.7 Loin Pain


You are called to A&E and asked to see a 67-year-old gentleman who complains of a 7 hour history of
severe left sided loin pain. On detailed examination, he reports a mild history of dysuria which has not
bothered him considerably. A urine dip is negative for nitrites and leucocytes, but shows microscopic
haematuria. He has a history of hypertension but admits to not having taking any antihypertensive
medication for more than a year. His blood pressure is 110/75 and his pulse is 113 bpm.

How would you proceed?

I would start by assessing the patient’s airway, breathing and circulation following the ABCDE ATLS
principles and commence resuscitation. If the patient was stable, I would perform a full clinical
examination and take a full history. From the history given, of tachycardia and a relative hypotension,
I am concerned that the patient may have signs of haemodynamic shock, and want to rule out a ruptured
abdominal aortic aneurysm as a differential. Specifically, I would feel for an expansile, pulsatile
abdominal mass superior to the umbilicus. If I was concerned about the likelihood of a ruptured
aneurysmal sac, I would immediately contact senior surgical assistance to assist in the assessment of the
patient, and arrange an immediate CT with contrast. If available in accident and emergency, an
abdominal ultrasound may be helpful.

I would site two gray (16G) cannulae in the antecubital fossae, and send a set of bloods, including FBC
to check the haemoglobin, clotting to ensure it is normal, U&E to assess renal function, and I would
cross match at least 4 units of blood. I would perform an ABG to obtain an immediate Hb and a lactate,
which if high could point to tissue ischaemia. If I felt that his symptoms were more in keeping with renal
colic, I could order a CT KUB to visualise his urinary system and exclude obstruction.

What is the likely differential diagnosis?

There are a number of differential diagnoses in this case, such as ureteric colic, pyelonephritis and
diverticulitis, but the diagnosis that I would be most keen to exclude at an early stage is a ruptured
abdominal aortic aneurysm, which is associated with sudden deterioration and significant mortality.

• Ureteric colic - pain comes in waves, unable to lie still


• Ruptured abdominal aortic aneurysm - patient lying still
• Pyelonephritis - renal angle tenderness & pain on ballotting
• Diverticulitis - peritonitic, lower abdominal pain

What features of the history would lead you to a specific diagnosis?

The most concerning feature in the history is his tachycardia and relatively low blood pressure. This
gentleman’s blood pressure, whilst within normal limits for a healthy young person, could be a sign of
relative hypotension in a patient with known untreated hypertension. Grade 2 shock, in a patient with severe
abdominal pain should be considered as a ruptured AAA until proven otherwise.

The patients gender, age and history of hypertension are risk factors for developing a AAA.

CORE SURGICAL INTERVIEW GUIDE 23


www.surgicalinterview.co.uk
HELPING YOU THROUGH YOUR CORE SURGICAL INTERVIEW 12th EDITION: 2023

The diagnosis of ruptured abdominal aneurysm is commonly confused with renal colic. The pain of
renal colic is classically described as being relapsing remitting and the patient may demonstrate a
waveform pattern with their hands when questioned on the variation in intensity of the pain. Dysuria
in a gentleman of this age would not be exclusive to renal colic and could co-exist with a diagnosis of
ruptured AAA. A patient with renal colic would also classically be unable to lie still, classically
differentiating it from the peritonitic picture of a patient with perforated colonic diverticular disease.

Pyelonephritis would be less likely in this case, given the absence of fever, rigors, nausea or vomiting
and it would not explain the relative hypotension found in this case. Microscopic haematuria would
not exclude AAA rupture at the expense of renal colic in this case, mainly due to the age and gender
of the patient.

You review the patient’s past notes and find that a plain abdominal x-ray was performed
at his local hospital prior to transfer. What does this show?

This abdominal x-ray demonstrates the calcified wall of an aneurysm (arrowed). It bulges over to the
left side away from the inferior vena cava, which runs along its right side.

You return from reviewing the x-ray to find your patient with a blood pressure of 85/45
and GCS 14/15. Blood results are still pending.

What grade of shock does the patient have?

He is hypotensive and tachycardic, therefore he is in at least grade 3 shock.

This patient has clinically deteriorated and I would reassess using the ABCDE protocol. I would
immediately inform my seniors, the anaesthetist and theatres. It is likely that he requires urgent
operative intervention.

What further imaging methods would you use at this point to delineate the aneurysm?

Abdominal ultrasound is a useful and accessible imaging method and is used


in population screening and in the measurement of the AP diameter of the
aneurysm. FAST (focussed abdominal sonography in trauma) scanning can
now commonly be performed in A&E resus departments. CT scan with
intravenous contrast enhancement, gives provides preoperative mapping of
the aneurysm and detects associated aneurysms of the common and iliac
arteries. However, given the fact that the patient is showing signs of
cardiovascular compromise, I do not think transfer to the CT department would be appropriate at this
stage.

What would you do?

CORE SURGICAL INTERVIEW GUIDE 24


www.surgicalinterview.co.uk
HELPING YOU THROUGH YOUR CORE SURGICAL INTERVIEW 12th EDITION: 2023

I would aggressively fluid resuscitation and contact my senior colleagues and the anaesthetist to alert
them to the need for an immediate transfer to theatre for aneurysmal repair. It is important that the
patient has 4-6 units of blood available and two sites of wide bore access (grey cannula), whilst reassuring
the patient.

Supplementary information

Grade Symptoms

1 Mild tachycardia
15% (750 ml) blood loss
2 Moderate tachycardia, fall in pulse pressure, delayed capillary return
15-30%(750 - 1500 ml) blood loss
3 Hypotension, tachycardia, low urine output
30 - 40%(1500 - 2000 ml) blood loss
4 As above but with profound hypotension
40-50% (2000 -2500 ml) blood loss

Shock: Smoking and hypertension are the two risk factors for atheromatous aneurysmal arterial
disease. The most common site for aneurysmal formation is the infrarenal abdominal aorta. A
variable length of the abdominal aorta and the suprarenal abdominal aorta may be affected in 10-20
of patients, however the ascending aorta is usually unaffected.

Advice

The key in this case is recognition of a ruptured AAA as the most worrying differential in an older
gentleman presented with loin pain, with early signs of shock. This case has been used several times
both for the core surgical interview and in the MRCS part B. You must show an awareness of the
severity of this diagnosis, in terms of the significant associated morbidity and mortality. Discussion of
assessment of airway, breathing and circulation is essential. These patients are at high risk of rapid
clinical deterioration and you must show that you can identify the early stages of hypovolaemic shock.
This question leads you towards a discussion of ruptured AAA.

Whilst it is theoretically correct to list a number of differential diagnoses,


it is a good idea to start your answer with a sentence such as:

‘There are a number of differential diagnoses in this case, such as x, y and z, but the
diagnosis that I would be most keen to exclude at an early stage is a ruptured abdominal
aortic aneurysm, which is associated with sudden deterioration and significant
mortality’.
This leaves the examiner in no doubt that you have identified the red flag
and that most importantly, you will be a safe member of the team.

CORE SURGICAL INTERVIEW GUIDE 25


www.surgicalinterview.co.uk
HELPING YOU THROUGH YOUR CORE SURGICAL INTERVIEW 12th EDITION: 2023

1.8 Bowel Obstruction I


You are asked to see a 72 year old retired army officer, living in a retirement home who was sent into
A&E by his GP. The patient reports a 3 day history of severe generalised abdominal pain, during
which time his abdomen has become grossly distended. He admitted to taking ‘a few tablets of codeine
here and there’ for the pain, but this has not led to any improvement in his symptoms. He has not
opened his bowels since the start of this episode, nor has he been able to pass flatus. He has felt
nauseated for the past 24 hours, but has not vomited. He has had a fall in appetite and he has not
eaten properly for 2 days. His pulse is 95bpm with a BP of 170/90. His temperature is 37.2 degrees.
On examination, his abdomen was grossly distended and uniformly tender, with no previous surgical
scars and clear hernia orifices. He has no history of constitutional symptoms such as fever or recent
unintentional weight loss.

What could this patient have?

This patient appears to have a obstruction of his bowel as evidenced by the history of absolute
constipation for three days. The main differentials are:

1: Large bowel mechanical obstruction

2: Small bowel mechanical obstruction

3: Pseudo-obstruction (Ogilvie’s Syndrome)

4: Ileus

The history given for this patient makes a large bowel obstruction the most likely diagnosis; his
constipation preceded his nausea, he has not vomited and he has no evidence of previous abdominal
surgery or hernia. Intra-abdominal adhesions, from prior surgery or herniae are the two most common
causes of an acute small bowel obstruction in the UK. Obstruction of the small bowel would normally
be associated with early or profuse vomiting. The pain of small bowel obstruction is classically described
as a ‘cramping’ pain, with each spasm lasting a few minutes and vomiting precedes constipation.

How would you proceed?

After assessing the patients airway, breathing and circulation as appropriate, I would reassure the
patient and give him suitable analgesia, in this case most likely intravenous morphine. I would pass a
nasogastric tube to decompress the stomach and establish large bore IV access and start IV fluids. I
would take bloods, including a full blood count, U&Es, CRP, clotting, a group and save and a venous
gas to check his lactate. I would also order an x-ray of his abdomen whilst in the supine position, as
well as an erect chest x-ray to rule out perforation.

What does this abdominal x-ray demonstrate & is it supportive of your diagnosis?

CORE SURGICAL INTERVIEW GUIDE 26


www.surgicalinterview.co.uk
HELPING YOU THROUGH YOUR CORE SURGICAL INTERVIEW 12th EDITION: 2023

This x-ray shows a distended oval gas shadow, looped on itself to give the typical coffee bean sign. The
transverse markings of the dilated bowel do not extend across the width of the gas shadow, suggesting
that these are haustra and hence this is large intestine. These appearances would be typical of sigmoid
volvulus.

How would you further investigate and manage this?

I would contact the registrar on call and make them aware of the patient I would take a full history and
examine the patient. This would include abdominal palpation and auscultation for bowel sounds, which
would be classically ‘tinkling’ in bowel obstruction. I would also take a set of bloods, including a venous
gas and a lactate. I would expect a metabolic acidosis and elevated lactate if there were bowel ischaemia.
I would give IV fluids to rehydrate and correct electrolyte imbalance and insert an NG tube to ensure
adequate rest for the bowel.

The majority of patients with sigmoid volvulus can be effectively conservatively managed by
decompression with a rigid sigmoidoscope. This is passed with the patient lying in the left lateral
position. A large, well lubricated, soft rubber rectal tube is passed along the sigmoidoscope. The patient
should then be observed for 2-3 days for signs of bowel ischaemia.

How would you proceed if your attempts at conservative management were


unsuccessful?

If the patient’s condition did not improve, that means their pain does not improved +/- raised
inflammatory markers and signs of sepsis, I would again contact my senior surgical colleagues. A loop
of sigmoid, with its blood supply cut off by torsion, is at risk of becoming necrotic if left untreated. I
would prepare the patient for theatre, and request urgent senior review. Preparations would involve a
cross match of 4 units and insertion of urinary catheter. Surgery would be carried out under general
anaesthetic through a lower midline incision. A rectal tube would be used to evacuate sigmoid
contents and then the redundant sigmoid loop would be commonly resected with immediate
anastomosis or colostomy, depending on the fitness of the patient

Supplementary Information

Sigmoid volvulus occurs when the bowel rotates on its mesentery, with rapid development of
strangulated, obstructed bowel. The characteristic abdominal x-ray (shown above) shows ‘coffee-bean
sign’, an inverted ‘U’ loop of bowel. Sigmoid volvulus is most commonly seen in the elderly, constipated
patient (this gentleman has been taking codeine) and those with notable co-morbidites and poor mobility
(he is in residential home). It is responsible for 8% of large bowel obstructions.

Ogilvies syndrome is an acute pseudo-obstruction and dilatation of the colon in the absence of true
mechanical obstruction. It occurs in severely ill patients and is associated with ‘massive dilatation of the
caecum and right colon on abdominal x-ray (>10cm). It may occur following major surgery, serious
infection or metabolic disturbance. It is also associated with drugs such as anticholinergics, which
decrease colonic motility. However, the exact mechanism behind its development remains unclear.

CORE SURGICAL INTERVIEW GUIDE 27


www.surgicalinterview.co.uk
HELPING YOU THROUGH YOUR CORE SURGICAL INTERVIEW 12th EDITION: 2023

Note the use of the NG tube in obstruction. Simply being nil by mouth does not give adequate rest for
the bowel because the intestine can produce up to 9l of fluid/day.

Immediate surgery is indicated in cases of strangulation or ‘closed loop obstruction’. This can mean any
bowel obstruction where the bowel is obstructed at two points. In the small bowel the risk of
strangulation is high with a mortality rate of approximately 25%. It may also refer to the specific case
of large bowel obstruction is associated with a grossly dilated caecum. This occurs when the ileocaecal
valve remains competent and does not allow decompression of the large bowel retrogradely. The
caecum dilates more than the rest of the colon due to its larger diameter and there is a risk of pressure
induced ischaemia of the bowel wall and ultimately, perforation, One should therefore palpate over the
caecum to assess for tenderness.

Advice

Bowel obstruction is a commonly encountered clinical problem and you will be expected to have some
experience of its management. Make sure that you are clear about the structural differences between
small and large bowel obstruction. Small bowel is identified by valvulae conniventes that cross the
whole width of the lumen, while the hautrae of large bowel do not cross the entire bowel width. Small
bowel is usually more centrally located on the AXR whilst large bowel is more peripheral. The key is
to know how you would manage the patient safely in the initial stages, placing a NG tube and giving
IV fluid, as well as arranging suitable imaging is a basic formula for safe initial management if
accompanied by early involvement of senior colleagues and analgesia as necessary.

CORE SURGICAL INTERVIEW GUIDE 28


www.surgicalinterview.co.uk
HELPING YOU THROUGH YOUR CORE SURGICAL INTERVIEW 12th EDITION: 2023

1.9 Bowel Obstruction II


You are the surgical SHO on call. You are asked to see a 56 year old lady who has presented to A&E
complaining of sudden central abdominal pain which started 12 hours earlier. This started shortly
after she had gone to bed and was accompanied by one episode of vomiting. The pains recurred every
few minutes and she vomited green/yellow fluid a number of times. Since the pain began, the patient
has not moved her bowels or passed flatus. In terms of past medical history, this lady has had three
previous spontaneous vaginal deliveries, she is on HRT and had an open appendicectomy through a
paramedical incision 10 years ago for a perforated appendix.

On examination, she has a temperature of 37.4 degrees and is tachycardic at 110bpm. She is clinically
dehydrated. On abdominal inspection, you see the following:

What is your chief differential diagnosis?

This lady has presented with acute colicky abdominal


pain, abdominal distention, profuse vomiting followed by
absolute constipation. These features are all classical for
acute intestinal obstruction. Her history of previous
abdominal surgery, would make obstruction secondary to
adhesions or an adhesive band by far the most likely cause
of her symptoms. Early bilious vomiting with later absolute
constipation is associated with small bowel obstruction.

What radiological investigations would help confirm your diagnosis?

I would request a plain radiograph of this lady’s abdomen. If I was concerned that she may have
perforated viscus as a cause of her pain, I would also request an erect chest x-ray, looking for air below
the diaphragm.

Many radiographers are reluctant to perform plain abdominal x-rays. What are the
other indications for a plain film of the abdomen?

• Bowel obstruction
• Visceral perforation
• Acute inflammatory bowel disease
• Abdominal trauma
• Haematuria
• Renal calculus/renal colic

These guidelines are to reduce the number of unnecessary abdominal radiographs being carried out,
which will not be helpful in reaching a diagnosis or influence management, in order to reduce the

CORE SURGICAL INTERVIEW GUIDE 29


www.surgicalinterview.co.uk
HELPING YOU THROUGH YOUR CORE SURGICAL INTERVIEW 12th EDITION: 2023

radiation dose given to patients. However, in this situation, I feel that an abdominal x-ray would be
wholly justified

This is taken from the current Royal College of Radiologists guidelines on abdominal x-rays. In addition
the following are not acceptable indications:

• Acute gastrointestinal bleeding


• Palpable mass
• Gallstones
• Pancreatitis
• Appendicitis
• Urinary tract infections
• Constipation (other than as detailed above)
• Non specific abdominal pain

What are the classical findings on x-ray in small bowel obstruction? What other imaging
would you like?

A supine radiograph of the abdomen may show dilated loops of bowel. Small
bowel would be centrally located and would have valvulae coniventes which pass
transversely all the way across the bowel. The maximum normal diameter of
small bowel is 3cm. An erect abdominal film may show multiple fluid levels,
giving a ‘ladder’ pattern

I would also consider imaging of the abdomen after injection of water soluble
contrast through a nasogastric tube, an abdominal CT scan and abdominal
ultrasound.

How would you manage this patient?

I would assess this patient’s airway, breathing and circulation and commence resuscitation as
appropriate. Although there is no evidence of blood loss, I would take note of her
diameter
tachycardia and consider other causes for this such as pain and sepsis.

I would reassure the patient and commence appropriate analgesia, and make the patient nil by mouth.
I would pass an NG tube and use suction to empty the distended stomach, review the patient’s blood
results and do a
antibiotic therapy if surgery was to be performed, after review of local protocols and discussion with
microbiology. I would discuss this patient with my senior colleagues and ensure they are prepared for
theatre.

The patient failed to settle on conservative management and was operated on 48 hours
later. Her abdomen was explored through a lower midline incision. Distended loops of
small bowel were immediately encountered. An adhesive band was found across a loop

CORE SURGICAL INTERVIEW GUIDE 30


www.surgicalinterview.co.uk
HELPING YOU THROUGH YOUR CORE SURGICAL INTERVIEW 12th EDITION: 2023

of small intestine and to obstruct it. The band was divided and the patient made a smooth
recovery.

What are the complications of small bowel obstruction?

With proper diagnosis and treatment of the obstruction, prognosis in small-bowel obstruction (SBO) is
good. However, complications of SBO include sepsis and intra-abdominal abscess.

Cases requiring surgical management can be complicated by wound dehiscence, aspiration pneumonia
(due to poor clearance of secretions), short-bowel syndrome (as a result of multiple surgical interventions)
and mortality related to delayed treatment and perforation.

Supplementary Information

The leading cause of small bowel obstruction in the is postoperative adhesions in over half of cases,
followed by malignancy, Crohns disease, and hernias. Surgeries that are more commonly associated
with small bowel obstruction include colorectal and upper GI surgery, appendicectomy and
gynaecological procedures.

A small proportion of patients with SBO will have no obvious abnormality on plain abdominal x-ray,
possibly explained by the loops of distended bowel being completely filled by fluid.

maximum normal diameter


small bowel 30mm
large bowel 60mm
caecum 90mm

Small vowel can dilate to approximately 5cm in diameter before it perforates. If you measure the
diameter of a section of small bowel at 7cm, it probably isn’t small bowel.

Untreated, strangulated obstructions lead to an almost 10% mortality rate. If surgical management is
commenced within 36 hours, the mortality rate decreases to 8%. The mortality rate is 25% if the
surgery is postponed beyond 36 hours. In current practice, the majority of those diagnosed as having
small bowel obstruction are managed conservatively.

Some adhesions from previous operations will loosen, leading to resolution of the mechanical
obstruction. Conservative management involves IV fluids, passage of an NG tube and correction of
electrolyte disturbance. If this route is taken however, the patient must have interval abdominal x-rays
to ensure that the dilatation is not worsening. Clinically, if the patient deteriorates, surgical
intervention may still be necessary. Most cases do resolve with conservative management in under 5
days.

Small bowel obstruction caused by IBD, radiation enteritis or following childbirth is rarely treated
with surgery. The converse is true for small bowel obstruction in an abdomen which has never been
surgically managed. These commonly require an operation to relieve the obstruction.

CORE SURGICAL INTERVIEW GUIDE 31


www.surgicalinterview.co.uk
HELPING YOU THROUGH YOUR CORE SURGICAL INTERVIEW 12th EDITION: 2023

1.10 Burns
You are the surgical SHO on call. You are called to see a 52 year old factory worker who has
sustained burns to the his back and upper arms in an industrial accident.

How would you assess the severity of the injuries?

I would use the ‘rule of nines’ to assess the area affected. This states that the area of the front of the
trunk is 2 x 9% of the body’s surface area, the back of the trunk 2 x 9%, each upper limb 9%, each
lower limb 9%, the head and neck 9% and the perineum 1%

or

I would use the patients open palm and fingers as representative of 1% of total body area and calculate
the area covered by the burns on this gentleman.

This gentleman total burn area was calculated as 14%. How would you initially manage
this patient?

My first step would be to assess this gentleman’s airway, breathing and circulation. My main concerns
would be his airway as inhalational burn injuries can cause laryngeal oedema which can rapidly
compromise an airway. One would look for signs of inhalation, such as soot around the nostrils and
mouth, or stridor. Additionally, I would request regular reassessment of his GCS. Carbon monoxide
inhalation can cause a delayed fall in conscious level. I would contact my registrar and alert him to the
situation.

14% burns area is borderline for intravenous fluid replacement. Current guidelines suggest that IV fluid
resuscitation is needed if the area of the burn is more than 15% of the total body surface area (10% in
a child).

How would you calculate the amount of fluid required?

I would calculate this volume using the Parkland formula:

Fluid replacement in first 24hrs (ml) = 4 x weight of patient (kg) x % area of burn

Half of this volume would be given in the first 8 hours following the burn, and the second half over the
next 16 hours. This would be in addition to the patient’s normal daily fluid requirements (3L of
crystalloid fluid under temperate conditions). My fluid of choice would be Hartmann's solution as it
bears the closest relation to

What is the definition of a full thickness or partial thickness burn?

CORE SURGICAL INTERVIEW GUIDE 32


www.surgicalinterview.co.uk
HELPING YOU THROUGH YOUR CORE SURGICAL INTERVIEW 12th EDITION: 2023

A partial thickness burn leaves part or whole of the germinal epithelium intact, so complete healing
takes place. A full thickness burn destroys the germinal layer and therefore unless it is very small, can
only heal by dense scar tissue.

This gentleman’s burns were found to be partial thickness on his back. The burns over
the arms and his right scapula were found to be full thickness. Would you refer this
gentleman to a burns unit?

I would refer this gentleman to a burns unit as his injuries cover >10% of his body surface area

These areas of full thickness burns were excised the following day and covered with split-
skin grafts taken from the thigh. What types of grafts are in current usage?

A full thickness graft consists of the epidermis and the whole dermal depth. The donor site must be
closed by suturing or split skin graft.

A composite graft contains skin, cartilage or other tissue. This can be used to reconstruct specific areas
with a small surface area and unique structural needs, such as the nasal alar rim.

A split skin graft is taken through the germinal layer of the epithelium, leaving islands of the layer on
the donor site. It is a graft which includes the epidermis and a depth of the dermis. They would be useful
in this case as they can cover large surface areas and their rate of autorejection by the host is low. The
donor site should heal by re-epithelisation from the dermis and should be ready to be a donor site within
6 weeks.

Advice

Information on the management of burns is well covered in the ATLS manual. If you have not yet
completed the course, you will be able to get a copy of the latest manual from your library and it covers
a number of useful topics in some detail. It is important that you know the indication for referrals to a
burns unit, (see Supplementary Information), as well as how to figure out methods for fluid replacement
and calculation of burns surface area.

Supplementary Information

Indications for Referral to Specialist Burns Unit

• Partial thickness burns greater than 10% total body area

• Burns involving the hands, feet, face, genitalia , or major joints

• Third degree burns in any age group

CORE SURGICAL INTERVIEW GUIDE 33


www.surgicalinterview.co.uk
HELPING YOU THROUGH YOUR CORE SURGICAL INTERVIEW 12th EDITION: 2023

• Electrical burns, including lightening injury

• Chemical burns

• Inhalation injury

• Burns injury in patients with co-morbidities that could affect recovery

• Any patient with burns and concomitant trauma (such as fractures) in which the burn poses the
greatest risk of morbidity or mortality.

• Burned children in hospitals without qualified personnel or equipment for the care of children

• Burn injury in patients requiring special social, emotional or long term rehabilitative support.

CORE SURGICAL INTERVIEW GUIDE 34


www.surgicalinterview.co.uk
HELPING YOU THROUGH YOUR CORE SURGICAL INTERVIEW 12th EDITION: 2023

1.11 Slipped Upper Femoral Epiphysis (SUFE)


You are asked to review a 13-year-old girl who is slightly overweight. She presents with her mother
who explains that her daughter has suffered a ‘groin strain’ and thinks that this must be due to playing
netball, which she was playing that afternoon. She first experienced pain in the groin 2 months ago
and has been taking paracetamol, but it has not improved. She is able to weight bear, but does this
reluctantly, she is no longer able to take part in netball training. The urine is negative for betaHCG.

What is your differential diagnosis?

The probable diagnosis in this age group and gender is of a slipped upper femoral epiphysis (SUFE).
However, the injury could be related to a simple muscular strain as suspected by the parent in this case.
Perthes' disease is less likely as it affects those aged between 3 and 11 (typically 4-7) with males more
commonly affected.

Which factors on history and examination would support this diagnosis?

Males are more commonly affected with SUFEs, but it is associated with obesity in 50% of cases. It is
associated with pain in either the hip or the knee and a resultant antalgic ‘waddling gait’ and limp. Hip
motion would be limited, particularly involving internal rotation, the foot would be externally rotated
and there could be apparent shortening of the hip. In 20% of cases, SUFEs are bilateral. The classical
case is in early adolescence, with boys affected from 12-15 and girls from 10-13.

I would perform a detailed gait examination, paying particular attention to external rotation, (which
would be apparent even in early slips of the epiphysis) positioning of the foot whilst at rest and the range
of active and passive motion at the hip whilst lying supine.

Which investigations would you order?

I would order an AP pelvis and frog lateral view of both hips (as 20% of cases are bilateral), as well as
the knee if the patient complained of pain in this joint as well.

What are your treatment options for this patient?

I would start by reassuring the mother and the child and give appropriate analgesia. I may be
necessary to admit the child after discussion with my senior surgical colleague (registrar) and make
sure that the child had a period of bed rest. The gold standard procedure in this case is the fixation of
the slipped upper femoral epiphysis in situ with a cannulated screw. This prevents further slippage and
encourages the physis to ossify and close. The screw should be left in place until skeletal maturity.

Advice

CORE SURGICAL INTERVIEW GUIDE 35


www.surgicalinterview.co.uk
HELPING YOU THROUGH YOUR CORE SURGICAL INTERVIEW 12th EDITION: 2023

Most foundation trainees are unlikely to have extensive firsthand experience of this kind of injury, given
its relative rarity. It is important to know the age group distributions for causes of hip pain in children.
Remember, paediatric surgery is a subset of core surgical training and it is possible for questions
concerning adolescents to be posed. With any questioning involving people under 18, you should
emphasise the importance of good communication with the family and reassurance of the child and the
parents, who are likely to be anxious. Analgesia is also essential. You may not be aware of the surgical
management in a case such as this, but you should have an idea of the ages affected by Perthes' disease
and slipped upper femoral epiphysis.

Supplementary Information

SUFEs involve displacement of the epiphysis slipping infero-posteriorly through the growth plate. It can
often present with referred pain to the knee on the ipsilateral side.

Slips can be classified into acute or chronic, stable and unstable. Although this case does seem to have
an acute precipitant (netball), because of the interval between this occurring and the presentation, it
would be classified as chronic. As the child can weight bear, it would be categorised as stable; if the child
could not weight bear, it would be classified as unstable. In acute slips, for instance after a fall from a
bike or a sporting injury in an otherwise anatomically normal hip, efforts to reduce the femoral epiphysis
and then to fix it are advisable. In chronic slips, attempts to reduce the injury are troublesome and the
risks of avascular necrosis of the femoral head is considerable.

If untreated, SUFEs can result in avascular necrosis of the femoral head or malunion predisposing to
arthritis later in life. Symptoms may be mild, so you should have a low index of suspicion if the patient
is within the correct age group for gender.

CORE SURGICAL INTERVIEW GUIDE 36


www.surgicalinterview.co.uk
HELPING YOU THROUGH YOUR CORE SURGICAL INTERVIEW 12th EDITION: 2023

1.12 Trauma
A 35-year-old man has fallen off a bridge, he is brought into you immobilised on a spinal board, and is
screaming out in pain.

How would you manage him acutely?

I would resuscitate the patient, assessing his airway, breathing and circulation, treating any life-
threatening complications, before assessing him further.

Please explain this in more detail

I would get help and ensure a trauma call has been put out and ensure that the appropriate help is on
its way.

A: secure the airway and triple immobilise the cervical spine

B: look for equal bilateral chest movements, auscultate the chest, measure oxygen saturations and
respiratory rate, mention performing an ABG and arrange a portable CXR as part of a trauma series
of chest and pelvis. Start oxygen (15L, non rebreathe mask)

C: Assess haemodynamic status (heart rate, volume, blood pressure). Feel peripheries, palpate abdomen,
look at JVP, and for any bleeding source,, arrange an ECG and cannulate with two wide bore cannulae,
send off bloods including group and save and cross match and start IV fluids if hypotensive.

D: GCS – split into Eyes, Voice and Movement (see below)

Or for a more rapid assessment - AVPU:

Alert

Responsive to Voice

Responsive to Pain

Unresponsive

BM - DEFG (Don’t Ever Forget Glucose)

E: complete the primary survey whilst avoiding hypothermia

What investigations would you order?

CORE SURGICAL INTERVIEW GUIDE 37


www.surgicalinterview.co.uk
HELPING YOU THROUGH YOUR CORE SURGICAL INTERVIEW 12th EDITION: 2023

As well as the basic bloods, including a cross match, ABG, I would like a trauma CT scan (or trauma
XR series) of this patient to assess for internal haemorrhage and injury. I would like to see an XR of
their cervical spine,

Here is his CXR, what would you do now?

This shows a right sided pneumothorax, with a fluid level, which is most likely suggestive of
haemorrhage given the history. I would reassess his ABCs, and insert a grey cannula into the 2nd
intercostal space, mid-clavicular line, to decompress any possible tension pneumothorax,. Signs of this
include deviation of the trachea away from the pneumothorax, reduced breath sounds, and a hyper-
resonant chest to percussion on the side of the pathology. . He will then require a chest drain. I would
ask for a cardiothoracic surgical opinion to determine the origin of the blood within the pleural cavity.

His GCS drops to 8, how would you proceed?

I would reassess him again using the ABCDE approach, paying particular attention to his airway. If one
has a GCS of 8 or below, they can no longer safely maintain their own airway, and I would ask for
anaesthetic assistance for a ‘definitive airway’ - a secured, cuffed
endotracheal tube. In the meantime, I would manage his airway
with simple manoeuvres and airway adjuncts as appropriate. I
would arrange a CT head, and ask for a

neurosurgical opinion if indicated.

What does the CT head show?

It shows a left sided acute subdural haematoma with midline shift. This is a
surgical emergency requiring rapid intervention by a neurosurgeon. If he
were not in a neurosurgical unit I would arrange rapid transfer of the patient
at the same time as stabilising them haemodynamically.

Advice

Throughout, whenever you ask for an investigation you will be presented with it, so make sure you can
read ECGs – likely to be normal and CXRs – may well have a pneumothorax or haemothorax. The
more familiar you are with the appearance of these the better. Know how to interpret an ABG, a pelvic
Xray, and a CT head quickly and accurately.

CORE SURGICAL INTERVIEW GUIDE 38


www.surgicalinterview.co.uk
HELPING YOU THROUGH YOUR CORE SURGICAL INTERVIEW 12th EDITION: 2023

People do well on this if they are quick at interpreting scans, and can make sensible decisions. It is
testing your ability to think under pressure, and your decision making, and once again, the structure
of your thoughts and answer.

Supplementary information

GCS Eyes Voice Movement

1 No response No response No response

2 Opens to pain Incomprehensible sounds Extends to pain

3 Opens to voice Inappropriate words Inappropriately flexes to pain

4 Open spontaneously Confused Withdraws to pain

5 Orientated Localises to pain

6 Obeys commands

Are you familiar with any current head injury guidelines with regards to performing a
CT head scan?

The NICE guidelines for head injury assessment, investigation and early management have recently
been updated (2014) and clearly describe criteria for when to perform a CT head scan in the context of
a head injury in adults.

Criteria for CT Head scan in adults following a head injury:

If any of the risk factors listed below are present, perform a CT head scan within 1 hour:

• GCS less than 13 on initial assessment in the emergency department.

• GCS less than 15 at 2 hours after the injury on assessment in the emergency department.

• Suspected open or depressed skull fracture.

• Any sign of basal skull fracture (haemotympanum, 'panda' eyes, cerebrospinal fluid leakage
from the ear or nose, Battle's sign).
• Post-traumatic seizure.

• Focal neurological deficit.

CORE SURGICAL INTERVIEW GUIDE 39


www.surgicalinterview.co.uk
HELPING YOU THROUGH YOUR CORE SURGICAL INTERVIEW 12th EDITION: 2023

More than 1 episode of vomiting.

For adults with any of the following risk factors who have experienced some loss of consciousness or
amnesia since the injury, perform a CT head scan within 8 hours of the head injury:

• Age 65 years or older.


• Any history of bleeding or clotting disorders.
• Dangerous mechanism of injury (a pedestrian or cyclist struck by a motor vehicle, an
occupant ejected from a motor vehicle or a fall from a height of greater than 1 metre or 5
stairs).
• More than 30 minutes' retrograde amnesia of events immediately before the head injury.

CORE SURGICAL INTERVIEW GUIDE 40


www.surgicalinterview.co.uk
HELPING YOU THROUGH YOUR CORE SURGICAL INTERVIEW 12th EDITION: 2023

1.13 Hip dislocation


You are asked to assess a 58-year-old gentleman on the ward who is day one post total hip
replacement. They are complaining of pain in the new hip.

How would you proceed?

When taking the call from the nurse, I would clarify the patients name, age and location. I would want
to know their observations, and what the concerns were at present. This allows me to make an initial
assessment immediately of the severity of the case, and the urgency of review.

Once I have ensured they are not acutely compromised by assessing their airway, breathing and
circulation I would take a full history, examine the patient and review the operation note, and the
patient’s notes to get more information. Specifically during examination of the hip I would assess the
position, any deformity, the range of movement and neurovascular status of the leg.

The patient had a right hip replacement operation 6 hours ago, they are now complaining
of severe pain in the right hip. On examination they are haemodynamically stable, but
have a reduced range of movement and their leg is rotated and internally rotated. What
might have happened?

It is likely that they have had a dislocation of their prosthetic hip.

What would you do?

I would inform the orthopaedic registrar on call my consultant after arranging an AP and lateral hip
XR and full length femoral views, and making ensuring that they were nil by mouth, had up to date
blood tests and were otherwise haemodynamically stable. They may need to return to theatre for
relocation of the joint. I would also ensure that the patient was comfortable and had adequate analgesia.

Tell us what you do when you look at an Xray

I firstly check the demographics to ensure it is the correct patient, date, time and side. I make sure that
I have two views of the anatomy of interest, and that ideally, the joint above and the joint below are
included. When inspecting a plain radiograph in a post operative orthopaedic patient, I would look for
fracture in native bone, periprosthetic fracture, change in angulation of the prosthesis or signs of
prosthetic loosening. In this case I would then inspect the AP and lateral images to ensure that the
femoral head remained within the acetabular rim. I then would contact my registrar to report my
findings.

The hip is dislocated, would you reduce it?

CORE SURGICAL INTERVIEW GUIDE 41


www.surgicalinterview.co.uk
HELPING YOU THROUGH YOUR CORE SURGICAL INTERVIEW 12th EDITION: 2023

The hip will require reduction, however, I have no experience of this, so I would contact my registrar.
I imagine reduced. They may well require further surgical intervention, however I am unsure of this
as I have little orthopaedic experience.

Advice

This is word for word how one candidate approached this station last year. He had no orthopaedic
experience therefore felt he had struggled. The feedback he received however was positive. He scored
highly for acting safely and within my abilities and requested help early. They commented that the
candidate ordered all the correct tests, and managed the case appropriately despite obviously finding it
difficult, due to a clear lack of orthopaedic experience.

We include this here to demonstrate that while good clinical knowledge helps, you can still do well by
being safe, and logical in your management. This candidate came 26th in London last year despite
feeling they had not done well in this station.

Supplementary information

Hip replacement is an extremely common procedure with a low complication rate. However
complications can occur. Initially, the patient must be reassured and given appropriate analgesia. The
hip must the be ‘reduced’ or repositioned. This may be under light sedation if tolerated by the patient,
or may be under general anaesthetic in a formal theatre setting. Reported rates of hip dislocation after
primary total joint arthroplasty varies by surgical approach, but varies from 1 to 7% in the literature.
This rate is higher in revision arthroplasty. Of those revision arthroplasties that dislocate, a high
proportion dislocate recurrently. (up to 70% reported in some studies). Dislocation is most common in
the 3 months following primary procedure and the risk declines thereafter. 45% of dislocations are
reported to occur within four weeks of the primary procedure.

Factors which increase the rate of dislocation can be divided into patient factors and surgical factors:

Patient factors include gender (female patients are almost twice a likely to suffer a dislocation than male
patients), weak hip musculature, increased age, obesity and excessive alcohol consumption. Dislocation
is also more likely in patients with neuromuscular disorders and conditions such as congenital hip
dysplasia.

Surgical approach is thought to have a role in incidence of dislocation, with a higher rate using the
posterior approach, when compared with the lateral approach (However, the lateral approach is
associated with a higher rate of other post operative complication such as sciatic nerve palsy,
trochanteric bursitis and increased post operative bleeding). Capsular excision during surgery increases
the risk of dislocation, as does previous hip surgery. It has also been reported that larger femoral head
sizes are associated with significantly lower risk of dislocation.

CORE SURGICAL INTERVIEW GUIDE 42


www.surgicalinterview.co.uk
HELPING YOU THROUGH YOUR CORE SURGICAL INTERVIEW 12th EDITION: 2023

1.14 Breakdown of an anastomosis


You are called to see a patient 69-year-old gentleman on a general surgical ward. He is day 4 post
open low anterior resection with end-to-end anastomosis for adenocarcinoma of the proximal rectum.
He complains of an 8-hour history of severe abdominal pain, which has spread from the periumbilical
region to his entire abdomen. He has a blood pressure of 100/68, with a pulse rate of 128bpm and a
temperature of 38.1 degrees. On examination, his abdomen is rigid and tender, with diffuse guarding
and rebound tenderness on palpation.

What would you do initially for this patient?

I would assess the patient’s airway, breathing and circulation. Given that the patient is showing signs of
haemodynamic instability, I would start with fluid resuscitation of the patient with intravenous fluids,
whilst continuing blood pressure monitoring. I would start appropriate intravenous analgesia using the
WHO pain ladder. I would take a full set of bloods from the patient, including full blood count, CRP,
U&Es (to check for electrolyte imbalance) group & save, venous blood gas. I would contact the registrar
on call to alert them to the case, as well as to the possibility of this patient requiring surgical intervention.
I would make the patient nil by mouth and order an emergency abdominal CT scan. I would also
contact microbiology and start broad spectrum intravenous antibiotics after taking blood cultures. I
would contact the on-call anaesthetist and alert the emergency theatre team as I would anticipate the
need for an emergency surgical procedure to manage this patient.


What would be your differential diagnosis in this case and why?

My leading differential diagnosis would be anastomotic leak with resultant faecal peritonitis. I would
also consider a post operative abscess, which could classically present with a ‘swinging pyrexia’ with or
without a palpable mass. However, anastomotic leak would be my main differential because of the high
leak rate of low rectal anastomosis when compared with intraperitoneal colonic anastomoses. This
presentation has many features of the classic presentation of anastomotic leak; rigid, peritonitic
abdomen, severe abdominal pain, tachycardia, fever and haemodynamic instability. Localised abscess
could be associated with pain and fever, but would not tend to cause the diffuse, rigid peritonism
described in this case


Do anastomotic leaks always present in this way?

No. A proportion present more insidiously, with a post operative failure to thrive, extended period of
ileus or mild pyrexia. These are the patients in whom a leak may be difficult to differentiate from an
abscess.

What are the risk factors for anastomotic leak?

Risk factors can be grouped into patient related factors and operative factors.

CORE SURGICAL INTERVIEW GUIDE 43


www.surgicalinterview.co.uk
HELPING YOU THROUGH YOUR CORE SURGICAL INTERVIEW 12th EDITION: 2023

Patient related factors include age and gender (males and older patients are more likely to suffer
anastomotic leaks), malnutrition, long term steroid treatment, obesity, tobacco and alcohol use,
leucocytosis, diverticulitis and cardiovascular disease.

Operative factors include sepsis, operative time greater than two hours, perioperative blood transfusion,
(which could be associated with an element of perioperative bowel ischaemia) and the level of the
anastomosis. (Low rectal anastomoses are significantly more likely to leak than intraperitoneal colonic
ones.

Surgical error is the final operative risk factor. Inappropriate formation of primary anastomosis, in cases
where faecal diversion in the first instance would have been more appropriate, can lead to increased
incidence of leaks. In addition, poor surgical technique at the formation of the anastomosis are
associated with higher rates of breakdown.

Are there any alternative to CT scanning in the investigation of anastomotic leak?

CT scanning is the gold standard test for the investigation of anastomotic leak. Contrast enema can
have a role in identifying leaks in combination with CT scans, but is not used independently as standard.

How would this case be surgically managed?

This case would likely be managed by extensive washout and drainage with proximal faecal diversion
with either a colostomy or an ileostomy.

Supplementary Information

There is a reported complication rate of up to 18% for low anastomoses such as a low anterior resection.
One third of post operative deaths after colorectal surgery are associated with anastomotic leak and
mortality rates of 7.5 to 36% is reported in the literature. This is increased to a reported mortality rate
of up to 50% in cases of low rectal anastomosis during anterior resection. There is no common diagnosis
for anastomotic leak. One retrospective study into recent reviews found 56 separate definitions of
anastomotic leak. It may be defined clinically, or by the need for a re-operation or on a radiological
basis. It is well recognised clinically, and is characterised by peritonitis, purulent or faecal discharge
from the wound site or drain and the presence of absence or fever. The consequences of leak can range
from intra-abdominal abscess, enterocutaneous fistula and diffuse peritonitis. Anastomotic leak is
associated with increased local recurrence and reduced survival rates after large bowel resection for
colorectal carcinoma. Surgical options for the management of anastomotic leak include faecal diversion
with a loop ileostomy or colostomy (as the candidate recommended in this scenario), or resection of the
anastomosis with a ‘sewing up’ of the proximal stump (Hartmann’s procedure). Faecal diversion allows
the possibility of future restoration of a patent anastomosis, but is only an option when dealing with
patent bowel. In cases of frank faecal peritonitis such as this, the chances of being able to form a
satisfactory anastomosis at a future date is significantly reduced. In cases of anastomotic leak involving
intra-abdominal anastomosis, the Hartmann’s procedure is most effective at controlling the leak, but
has the downside of numerous difficulties in restoring bowel continuity at a later date.

CORE SURGICAL INTERVIEW GUIDE 44


www.surgicalinterview.co.uk
HELPING YOU THROUGH YOUR CORE SURGICAL INTERVIEW 12th EDITION: 2023

Advice

This is an extremely common question in surgical interviews. It tests a basic level of surgical knowledge
and understanding of abdominal surgery. This answer is strong as it gives sufficient detail, whilst still
answering the question fairly succinctly. Whenever you are asked a question about risk factors, a good
candidate will always try to structure their answers rather than just reciting a list. This can be into pre-
operative, operative and post operative; patient related and surgery related or another method that you
may use yourself, but it can act as a useful memory aid and will let the examiner know that you are a
structured and logical thinker. Your answer should make it clear that you recognise the significant
increase in post operative mortality related to the diagnosis and you should mention contacting your
seniors and preparing the patient for theatre at an early stage. Also, stick to your guns. If you know that
CT scanning is the gold standard method of radiological investigation, then say it. Don’t doubt yourself
because of a question such as the one in this case.

You will notice the last answer on surgical management is brief. The panel do not expect you to have
an in depth knowledge of surgical technique at this time, but this answer is succinct and shows an
understanding of the need for bowel rest of the distal colonic segment, as well as the anatomy of the
colon.

CORE SURGICAL INTERVIEW GUIDE 45


www.surgicalinterview.co.uk
HELPING YOU THROUGH YOUR CORE SURGICAL INTERVIEW 12th EDITION: 2023

1.15 Abdominal pain


You are the surgical SHO on call. You are referred a 35 year old gentleman complaining of severe
and unremitting peri-umbilical pain, which started very suddenly whilst at work around 4 hours
previously. He assesses the severity as 10/10. He has felt well recently and has no history of fever,
nausea, vomiting, night sweats or change in bowel habit.

He occasionally suffers from ‘heartburn’, but this is relieved by a glass of milk before bed. He has not
seen his GP for this problem. On this occasion, a glass of milk and an antacid have not given any
symptom relief. He smokes 15 cigarettes/day and consumes 10-12 pints of beer per week.

His observations are as follows: BP: 120/75 P: 115 T: 37.9 deg

What are your differential diagnoses?

Peptic ulcer disease would be my leading differential diagnosis due to his gender, smoking and alcohol
history. His previous symptoms are suggestive of gastric reflux could indicate acid hypersecretion, which
would increase his risk of peptic ulcer formation.

The acute onset of severe central abdominal pain suggests that this patient may have a perforated viscus,
secondary to peptic ulcer disease.

I would of course consider other common causes of abdominal pain in this age group such as
appendicitis, cholecystitis, nephrolithiasis, genitourinary infection or testicular torsion.

How would you proceed with this patient?

I would ensure that the patient was stable, assessing him according to the ALS ABC principles and take
a history from the patient. I would complete a full examination, focussing on the abdominal
examination. Bedside tests would include an ECG (to exclude a cardiac origin for the pain, especially
with a known history of ‘heartburn’ pain), basic observations and urinalysis.

I would order blood tests including a full blood count, urea & electrolytes, liver function tests and C-
Reactive Protein. I would order an erect chest xray to exclude a perforated viscus and a plain abdominal
film to look for radiological signs of abdominal obstruction. I would take an arterial blood gas to look
for a raised lactate and consider further imaging such as a FAST (Focussed Abdominal Sonography in
Trauma) scan or a CT abdomen depending on the results of other investigations.

Explain your rationale for carrying out these investigations?

ABG Lactate – important marker of infection, particularly intraabdominal sepsis, as well as


tissue ischaemia

CORE SURGICAL INTERVIEW GUIDE 46


www.surgicalinterview.co.uk
HELPING YOU THROUGH YOUR CORE SURGICAL INTERVIEW 12th EDITION: 2023

FBC Low Hb from a subacute or chronic GI bleed


Raised WBC in infection

U&Es Indication of renal obstruction


For baseline – especially if considering using contrast for CT

CRP Marker of inflammation/infection

LFTs Indication of biliary obstruction (raised alkaline phosphatase)


Acute hepatitis – raised ALT

Clotting Sepsis can cause deranged clotting

G&S In preparation if patient needs to go to theatre

Urinalysis Blood – UTI/nephrolithiasis


WBC – UTI/nephrolithiasis/pyelonephritis

(You are more likely to be asked about one or two tests but for the purpose of revision this table is
useful)

What would you do if the patient was unable to sit up straight for the erect chest X-ray?

I would always try to get an erect CXR as it is a rapid way to visualise free air on plain film. However,
if this was not possible, a left lateral decubitus film (left side down) could be used to demonstrate free air
between the liver and lateral abdominal wall.

Your examination reveals a diffusely tense abdomen


with guarding in the epigastrium. There is no renal
angle tenderness. Your x-ray has appeared on PACS.

HELPING YOU THROUGH YOUR CORE


SURGICA
What does the X-ray show?

This is an erect chest radiograph, in with the patient is


partially rotated. Free air is seen between the right hemi-
diaphragm and the liver. This image, in combination with the associated history and presentation,
would make a diagnosis of perforated peptic ulcer most likely.

How would you proceed to manage this patient?

CORE SURGICAL INTERVIEW GUIDE 47


www.surgicalinterview.co.uk
HELPING YOU THROUGH YOUR CORE SURGICAL INTERVIEW 12th EDITION: 2023

I would ask the on-call surgical registrar to review the patient as a matter of urgency, highlighting my
concerns regarding the likely diagnosis of perforated peptic ulcer and the current condition of the
patient. I would put high flow oxygen in place. I would ensure that the patient had two wide bore
cannulae in situ. I would start intravenous analgesia, based on the WHO pain ladder, starting with IV
paracetamol, (in the absence of allergy). I would start IV fluid resuscitation therapy, titrated to blood
pressure, with a 500ml gelofusine fluid challenge if the patient was clinically shocked followed by 4-6
hourly ‘normal’ saline or Hartmann’s solution if there was satisfactory response to this challenge. I would
insert a nasogastric tube to decompress the stomach and a foley catheter to enable measurement of
urine output.

I would take blood cultures before commencing broad spectrum antibiotics according to local protocol
for intra abdominal sepsis. In some cases, a central line could be necessary to assess intravascular fluid
status. I would discuss this with the surgical registrar on call. Aware that the patient could require
surgical intervention, I would make the patient ‘nil by mouth’ and ensure that a group and save blood
test had been sent. After discussion with a senior colleague, I would also discuss the patient with the
anaesthetic registrar and theatre staff to alert them to a possible impending emergency laparotomy.
Should my senior think this was necessary, I would consent the patient for the procedure (or provide
the paperwork for the registrar) to ensure no delay to the operation.

Which features determine operative mortality?

Operative mortality in a patient with peptic perforation depends on four major risk factors:

• Length of time from perforation to admission


• Age
• Medical co-morbidity
• Hypovolaemia on admission

Do you know of any national or international guidelines for the management of sepsis?

The surviving sepsis campaign is a programme introduced by the European Society of Intensive
Care Medicine, aiming to increase awareness, understanding and knowledge surrounding the treatment
of sepsis. Its overarching aim is to reduce the mortality associated with sepsis by 25% (from 2009). The
premise is that this can be achieved by early recognition of septic patients, more targeted allocation of
resources and setting clear goals.

These include:

• Time from A&E admission to presumptive diagnosis of severe sepsis <2 hours
• Time from A&E admission a presumptive diagnosis of severe sepsis having a lactate blood test < 4
hours
• Time from A&E admission to appropriate antibiotics less than 4 hours
• Blood cultures taken before the administration of antibiotics - to increase the likelihood of
identifying an organism and therefore appropriately targeting antibiotic prescription.

CORE SURGICAL INTERVIEW GUIDE 48


www.surgicalinterview.co.uk
HELPING YOU THROUGH YOUR CORE SURGICAL INTERVIEW 12th EDITION: 2023

• If hypotensive or if lactate greater than 4.0mmol, immediate fluid resuscitation is started (at least 30
mL/kg normal saline or Hartmann’s solution within one hour)

Supplementary information

Some authors have suggested that in patients with perforation, but without radiological evidence of
pneumoperitoneum, conservative management is indicated as the perforation can be assumed to have
‘sealed off ’ independently. The majority of centres advocate surgical management in cases with both
peritonism and pneumoperitoneum.

Despite strong arguments favouring nonoperative treatment of patients with perforated PUD without
free air, delaying the initiation of surgery more than 12 hours after presentation has been
demonstrated to worsen outcome. Therefore, when indicated, a laparotomy should be performed as
soon as possible. The choice of operative procedure would depend on variables such as the presence
of hypovolaemic or septic shock, the degree of peritonitis and evidence or history of chronic peptic
ulceration. Laparoscopic procedures have been more recently described. However, in the presence of
significant co-morbidity or severe intra-abdominal sepsis, the classical management is repair of the
perforation with an omental (Graham) patch via upper midline laparotomy.

In patients with a prior ulcer history and without significant co-morbidity or systemic upset, a definitive
‘anti-ulcer’ operation could be indicated to reduce recurrence rates. For a perforated duodenal ulcer
(DU), this could include a highly selective (parietal cell) vagotomy, truncal vagotomy with pyloroplasty
or vagotomy with antrectomy. In a stable patient, the ulcer would be excised and sent for frozen section
analysis to exclude malignancy.

For a benign gastric ulcer, a distal gastrectomy with either a Billroth I gastroduodenostomy or a Billroth
II gastroduodenostomy would be most appropriate. This would carry significant morbidity and not be
indicated in the acute setting.

Recurrence Rates

Any procedure that preserves vagal innervation or antral gastrin production can lead to recurrence.
Recurrence rates for various operative techniques are as follows:

• Highly selective vagotomy: 5-20 %


• Vagotomy with pyloroplasty 10-15%
• Vagotomy with antrectomy <2%

Definition of sepsis

(www.survivingsepsis.org)

Documented or suspected infection with one or more of the following:

hypothermia/pyrexia core temperature <36degrees/>38.3 degrees

CORE SURGICAL INTERVIEW GUIDE 49


www.surgicalinterview.co.uk
HELPING YOU THROUGH YOUR CORE SURGICAL INTERVIEW 12th EDITION: 2023

increased heart rate >90bpm

tachyopnoea

altered mental status

oedema or positive fluid balance >20ml/kg over 24hrs

hyperglycaemia in the absence of diabetes plasma glucose >120mg/dl

leucocytosis or leucopenia WBC >12,000/uL or <4,000/uL

raised CRP

raised procalcitonin

increased O2 requirement SvO2 >70%

CORE SURGICAL INTERVIEW GUIDE 50


www.surgicalinterview.co.uk
HELPING YOU THROUGH YOUR CORE SURGICAL INTERVIEW 12th EDITION: 2023

1.16 Projectile vomiting


You are the paediatric surgical SHO on call. You are called by A&E to see a 6 week-old boy who
according to his mother, has been suffering from projectile vomiting following feeds for the last 3 days.

What would be your main differential diagnosis in this child?

This history would be consistent with a diagnosis of (infantile hypertrophic) pyloric stenosis, most
common in boys around one month of age. This boy is younger than the classic age group (6-18 months)
for intussusception, (which would classically present with ‘redcurrant jelly stool and abdominal pain).
Viral gastroenteritis would be the most common cause of generalised GI complaint in the whole
paediatric population, but I would want to exclude more serious pathologies before making this
diagnosis.

What are the most important pieces of information to gather from your history and
examination?

Given the history of vomiting for three days, the most important factor to assess would be the infant’s
hydration status. I would take the child’s blood pressure and observations, and look at his tongue,
mucous membranes and fontanelles. I would ask about whether the number of wet nappies had
changed. I would assess his weight and plot this on his growth chart. Weight loss would be an indicator
of nutrition/hydration status.

I would also look at the child in general; a quiet, floppy baby suggests illness or dehydration. I would
ask the mother about the colour of the vomitus. Pyloric stenosis is classically associated with non-billous
vomiting, as the obstruction is proximal to the pancreatic duct. The child would classically be hungry
and feed eagerly immediately after vomiting. I would also ask about other GI symptoms such as
diarrhoea and systemic features, such as fever and lethargy. I would ask about belching, and crying
(indicating abdominal pain), both of which could accompany pyloric stenosis. I would ask about any
previous similar episodes and his growth and development history to date. A family history of similar
problems may be uncovered.

On examination, I would feel for the abnormal pylorus, which may feel like an olive-shaped mass. I
would inspect the abdomen for visible peristalsis during feeding and just prior to vomiting episodes. The
abdomen may be generally distended.

His observations are as follows:

Temp: 36.5 deg HR: 120 BP: 75/50 RR: 24 weight 6.1 kg

The child is resting in his mother’s arms, occasionally crying. He has sunken fontanelles with dry mucous
membranes. His abdomen is soft and non tender with a mobile, olive shaped mass in the epigastric
region.

CORE SURGICAL INTERVIEW GUIDE 51


www.surgicalinterview.co.uk
HELPING YOU THROUGH YOUR CORE SURGICAL INTERVIEW 12th EDITION: 2023

Would this change your diagnosis and what would you do next?

Whilst diagnosis of infantile pyloric stenosis is primarily from history and examination, the finding of an
‘olive mass’ is close to being ‘pathognomonic’ for the condition, but is often difficult to elicit clinically.
It would reinforce my previous impression.

Importantly this child appears dehydrated. They require fluid resuscitation and senior paediatric input.
I would order a ‘standard’ set of bloods including FBC, U&E and LFT and a venous blood gas. I would
contact the paediatric surgical registrar on call. I would order an abdominal ultrasound scan (USS) to
examine the width and length of the abnormal pylorus. A less commonly used form of imaging option
would be an upper GI series (after giving the child an oral contrast agent).

What would you expect your venous blood gas result to show and how would you
explain this result?

The electrolyte picture most commonly seen in a vomiting patient would be a hypochloraemic
hypokalaemic metabolic alkalosis (with paradoxical aciduria). The hypochloraemia is caused by the loss
of hydrochloric acid in vomitus from the stomach. This causes the patient to become alkalotic. To
compensate for this, hydrogen ions are moved from the intracellular to the extracellular space in
exchange for potassium ions. In addition, the kidneys upregulate the renin-angiotension aldosterone
system, retaining sodium ions and secreting potassium ions. This causes a net reduction in the
extracellular pool of potassium ions, causing hypokalaemia. With the progression of hypokalaemia and
hypovolaemia, the kidneys continue to exchange hydrogen ions for sodium. This explains the increasing
alkalosis and paradoxical aciduria. Dehydration can lead to hyper- or hyponatraemia.

What is the classic finding on abdominal USS in pyloric stenosis?

An USS would show a thickened pylorus. (A thickness of greater than 3mm would suggest the
diagnosis).

Other USS findings can include:

Target Sign: Hypoechoic ring of hypertrophied pyloric muscle around echogenic mucosa centrally on
cross section.

‘Antral nipple sign’: Pyloric mucosa protruding into gastric antrum.

‘Cervix sign’: Indentation of muscle mass on fluid-filled antrum on longitudinal section.

What does the image below show?

This image shows the ‘string sign’ during an upper GI series, which represents the passage of small
barium streak through the stenosed pyloric channel. It supports a diagnosis of pyloric stenosis.

CORE SURGICAL INTERVIEW GUIDE 52


www.surgicalinterview.co.uk
HELPING YOU THROUGH YOUR CORE SURGICAL INTERVIEW 12th EDITION: 2023

How would you manage this patient?

Having contacted senior help (paediatric surgical registrar on-call & the paediatric accident and
emergency team), my initial management would follow the paediatric advanced life support guidelines
for hypovolaemic shock/dehydration.

I would correct the fluid deficit and acid/base imbalance. If the patient showed signs of clinical
dehydration (sunken eyes, reduced skin turgor, lethargy, pallor, dry mucous membranes, sunken
fontanelles), I would obtain IV/IO access and then give a 20ml/kg bolus of crystalloid fluid. I would
ensure that a group and save was taken and discuss the patient with the paediatric theatres and
anaesthetist on call. I would continue to reassess the child’s fluid balance and a nasogastric tube would
be indicated in some cases.

What are the options for definitive management and what are their benefits?

Infantile pyloric stenosis is a surgical emergency. The definitive management is corrective surgery. The
Ramstedt pyloromyotomy is the classical procedure of choice. This was traditional performed through
a right upper quadrant transverse incision, but recent studies have shown comparable results with
circumbilical incision (better aesthetic result) and laparoscopic pyloromyotomy (shorter in-patient stay)

Supplementary Information:

Infantile hypertrophic pyloric stenosis is not truly a congenital disorder. It affects the circular muscle of
the pylorus causing it to elongate and thicken. (see below)

The incidence is 2.4 per 1000 live births in Caucasians, 1.8 in Hispanics, 0.7 in Blacks, and 0.6 in
Asians The cause of this condition is not fully understood. It affects 3-5/1000 live births and affects
boys: girls in a 4:1 ratio. It is most common in first born sons and has a strong genetic component with
a 20% risk of a son being affected if a mother was affected, a 7% risk if a mother was affected, a 5%
risk of a son being affected if a father was affected and a 2% risk to a son if the father was affected.
Feeding can be restarted within 12-24 hours of surgery and recurrence and post operative
complications or mortality are rare.

CORE SURGICAL INTERVIEW GUIDE 53


www.surgicalinterview.co.uk
HELPING YOU THROUGH YOUR CORE SURGICAL INTERVIEW 12th EDITION: 2023

1.17 Difficulty urinating


You are the urological SHO on call in a District General Hospital. You are called by the FY2 in A&E
who refers a 70 year old black gentleman with a 2 month history of progressive difficulty initiating
urination and 7kg of weight loss over the same period. He has no other urinary symptoms and denies
other medical history

What is your differential diagnosis and what further features would you want to fully
investigate in your history?

In a male patient of this age, my primary differential would be of bladder outlet obstruction, most likely
due to prostatic enlargement of a benign or malignant cause. I would consider structural causes,
including urethral stricture, chronic prostatitis, bladder neck stenosis and chronic faecal impaction. I
would look to exclude functional problems, such as bladder neck dysynergia and neurological disease
including MS and diabetic autonomic neuropathy.

I would ask about family history of prostate cancer or benign prostatic hypertrophy, about his dietary
fat intake, and whether he had any systemic symptoms of malignancy such as loss of appetite and
smoking history, bony pain or an occupational history of cadmium exposure (found in cigarettes,
batteries and those working in the welding industry).

How would you assess the patient and what diagnostic tests would you perform?

I would perform a full physical examination, feeling for a palpable bladder, abdominal masses,
prominence of inguinal lymph nodes or other groin lesions. I would perform a digital rectal
examination, feeling for nodules or changes in the texture or asymmetry of the prostate. I would feel for
long bone, spinal and pelvic bony tenderness and perform a full neurological examination, including
assessment of external anal sphincter tone on PR examination to exclude gross signs of spinal cord
compression.

I would perform urinalysis, to exclude urinary tract infection and identify haematuria, and take blood
tests for serum urea and creatinine, PSA and liver function tests including alkaline phosphatase. I would
ask for assessment of post-void residual urine volume. I would contact the urology registrar on call, with
the results of my examinations and investigations. If there was evidence of bladder distention, I would
place a foley catheter to relieve the obstruction in the absence of contraindications.

On examination, you find left leg swelling, with prominence of the inguinal lymph nodes.
His PSA result is elevated. On PR examination, you find normal tone and an indurated,
symmetrically moderately enlarged prostate with no other masses. What is the definitive
investigation?

CORE SURGICAL INTERVIEW GUIDE 54


www.surgicalinterview.co.uk
HELPING YOU THROUGH YOUR CORE SURGICAL INTERVIEW 12th EDITION: 2023

The gold standard is needle prostate biopsy, under transrectal ultrasonography guidance, in cases of
suspected prostate malignancy. This allows staging of the disease using the Gleason score if
malignancy was confirmed.

The biopsy reveals a prostate cancer, which is assigned a Gleason score of 6 (3+3).
Which investigations would you recommend and why?

I would arrange chest radiography, full blood count and clotting, as well as an ECG and group and save
if the patient was to be managed surgically. MRI is helpful for localising cancer within the prostate and
seminal vessels for local staging. CT or MRI are used to identify metastatic deposits. Prostate cancer
most commonly metastasises to the bone, and less commonly the lungs, liver and pleura. PET scanning
can also help to identify these deposits.

What is the relevance of the Gleason score?

Prostate cancer is a very common malignancy and the Gleason score is the major scoring system used
at present.

The Gleason grading system is used to determine prognosis in patients with prostate cancer based on
the histological evaluation of tumour biopsy specimens. It is a measure of the normality of the normal
glandular structure of the epithelium. A grade of 1 indicates a normal or near normal pattern, with a
grade 5 scoring indicating the absence of any recognisable glandular pattern. The predominant and
second most predominant pattern are graded and the sum of these two grades comprises the Gleason
score.

Gleason Score

2-4 Low grade or well differentiated tumour

5-7 Moderate-grade or moderately differentiated tumour

8-10 High grade or poorly differentiated tumour

Patients with a Gleason score of 4 or lower tend to do well clinically, whilst those with a Gleason score
of 6 or higher are likely to have, or to progress to a diagnosis of advanced or invasive cancer.

Models have recently been developed that combine the clinical stage (determined by PR findings),
Gleason score and the PSA level in an attempt to predict the likelihood of local extension, as well as
the time to development of clinically metastatic disease.

CORE SURGICAL INTERVIEW GUIDE 55


www.surgicalinterview.co.uk
HELPING YOU THROUGH YOUR CORE SURGICAL INTERVIEW 12th EDITION: 2023

What are the treatment options for prostate cancer?

In many cases, treatment may not be indicated. This could be


due to the character of the tumour itself (low grade, indolent)
or due to the co-morbidity or frailty of a patient in cases where
surgical management would otherwise have been indicated.

Standard treatments for clinically localised prostate cancer


include radical prostatectomy, radiation therapy (including
brachytherapy and external beam radiation), hormonal
therapy, a combination of the above or active surveillance,
(careful observation of the tumour over time, with the
intention of treatment for cure if there are signs of cancer
progression). For the 50-75% of patients with prostate cancer
that will cause no harm before the man dies from an unrelated
pathology or old age, active surveillance may be indicated.

Hormonal therapy and radiotherapy are reserved for extra-prostatic spread in many cases. However,
radiation therapy may be used for some advanced tumours and hormonal therapy can be used for some
early tumours.

Supplementary Information

Prostate cancer is the most common non-cutaneous cancer amongst males and the second most
common cause of cancer death in males. It is an adenocarcinoma or glandular cancer that is caused by
mutations in the normal semen producing prostate gland cells, most commonly in the peripheral zone.

There is marked variance in incidence with geography. Highest rates are found in North America,
Australia, Northern and central Europe, with the lowest rates in South-eastern and South central Asia
and Northern Africa. 47% of cancers are identified in asymptomatic patients.

Gleason Grade 4 and 5 cancers are associated with adverse pathological findings and disease
progression. Low grade tumours can also be biologically aggressive, though in the main, they do tend
to be associated with better overall outcomes.

The ranges for disease free 10-year survival for early localised disease are as follows:

Treatment Disease free 10yr survival

Radical prostatectomy 80-95%

Brachytherapy and external radiation 80-95%

Watchful waiting’ 50-73%

CORE SURGICAL INTERVIEW GUIDE 56


www.surgicalinterview.co.uk
HELPING YOU THROUGH YOUR CORE SURGICAL INTERVIEW 12th EDITION: 2023

Genetic background may contribute to the risk of developing prostate cancer. Men with a 1st degree
relative (father or brother) with prostate cancer have twice the risk of developing prostate cancer and
those with 2 first degree relatives affected have a 5-fold greater risk compared with men with no family
history. Whilst no single gene has been identified as causative, mutations in the BRCA1 and BRCA2
genes, more commonly associated with breast and ovarian cancer, are two of the genes that have been
found to be associated with prostate cancer.

Prostate cancer, especially the most common low grade forms found in the typical male elderly patient
often is indolent requires no treatment.

CORE SURGICAL INTERVIEW GUIDE 57


www.surgicalinterview.co.uk
HELPING YOU THROUGH YOUR CORE SURGICAL INTERVIEW 12th EDITION: 2023

1.18 A painful leg


You are the orthopaedic surgical SHO at a District General Hospital. You are called to see a 32 year
old gentleman who sustained a closed fracture of the left tibia and fibula in a motorcycle accident ten
hours previously. His fracture has been reduced and put in a cast. However, he now complains of
throbbing left foot pain and numbness.

What would be your differential diagnoses?

My chief concern is the development of acute compartment syndrome. Other differentials include acute
limb ischaemia or a deep vein thrombosis, although this presentation would not fit with the usual time
course for a DVT.

You suspect that the patient has compartment syndrome. What are your initial actions?

Acute compartment syndrome is a limb threatening surgical emergency. I would immediately remove
the lower limb plaster cast and discuss the patient with the orthopaedic surgical registrar on call. I would
examine the limb, inspecting for pallor and skin changes, feeling the temperature of the limb, palpating
for distal arterial pulses (dorsalis paedis and anterior tibial) as well as determining capillary refill time,
and assessing sensation in the limb. I would ask the patient to plantarflex and dorsiflex the toes and
ankle and document the range of movement, and pain on movement. I would also assess pain on passive
movement.

If I could not palpate distal pulses, I may consider the use of a handheld doppler.

If the patient was unconscious, I would


measure pressure in each of the four
lower limb compartments with an
electronic arterial pressure monitor if
one was available.

Distal pulses are intact and easily


palpable. The skin is pink, but
does look slightly ‘shiny’. He is
not able to extend his great toe
and has a loss of sensation in his
1st web space. His anterior
compartment pressures are
38mmHg. Pressures in the other 3
compartments are < 18mmHg. His symptoms have not improved with cast removal.
How would you proceed?

CORE SURGICAL INTERVIEW GUIDE 58


www.surgicalinterview.co.uk
HELPING YOU THROUGH YOUR CORE SURGICAL INTERVIEW 12th EDITION: 2023

This patient would need a compartmental decompression by fasciotomy. I would alert the anaesthetic
registrar on call, as well as theatre staff.
I would elevate the limb to the level of
the heart and give intravenous
analgesia as per the WHO pain ladder
and starting with IV paracetamol. I
would insert two wide bore cannulae
intravenous line and send bloods
(including group and save) and consent
the patient for a fasciotomy. I wound
ensure sufficient IV hydration to
maintain an adequate urine output in
case of rhabdomyolysis.

What compartment do you think is


affected and why?

The loss of great toe extension and


sensation in the 1st webspace would
indicate involvement of the deep peroneal
nerve. This runs in the anterior
compartment.

At what compartment pressure is it considered to be ‘elevated’?

Compartment syndrome is a clinical diagnosis. Various recommendations of compartment pressure


thresholds have been used. Generally, a compartment pressure is considered elevated if it is more than
30 mmHg or when it is within 35 or 40 mmHg of the patients diastolic blood pressure. However, these
numbers are not a hard and fast

What are the ‘classical’ symptoms of compartment syndrome?

The ‘6 Ps’ classically associated with compartment syndrome are:

• Pain (out of proportion to what would be expected)


• Paraesthesia (may be in the cutaneous distribution of a nerve in the affected compartment)
• Pallor/pale
• Pulselessness
• Pressure

CORE SURGICAL INTERVIEW GUIDE 59


www.surgicalinterview.co.uk
HELPING YOU THROUGH YOUR CORE SURGICAL INTERVIEW 12th EDITION: 2023

• Paralysis (late finding)

Of these, only the first two (pain and paraesthesia) are reliably diagnostic. Pain is almost universal and
commonly described as severe, deep, constant and poorly localised. Pulses are frequently retained, as
the pressures that result in compartment syndrome are not usually sufficient to occlude the arterial
‘inflow’ pressure.

Supplementary Information

Compartment syndrome is defined as the compression of nerves, blood vessels and muscle inside a
closed space (compartment) within the body. Deep fascia envelops the limbs and other fascial planes
divide the limbs into compartments. The forearm has two compartments, the thigh has three
compartments and the lower leg has four compartments (see above). Compartment syndrome can affect
compartment of the forearm, lower limb, upper arm, abdomen and buttock, but the forearm and lower
leg are most commonly affected.

Two distinct types of compartment syndrome have been recognised. The first type is associated with
trauma to the affected compartment, as seen in open/closed fractures or muscle injuries. The second
form, called exertional compartment syndrome, is associated with repetitive loading or microtrauma
related to physical activity. Thus, compartment syndrome may be acute or chronic in nature.

Pressure increase within a confined space causes tissue necrosis, nerve injury and muscle infarct within
6-10 hours from a lack of microvascular inflow within compressed tissues. Pressure-induced functional
deficits are likely caused by decreased tissue perfusion rather than a direct mechanical effect. Therefore,
the amount of pressure a limb can tolerate depends on limb elevation, blood pressure, haemorrhage,
and arterial occlusion. In addition to local morbidity caused by muscle necrosis and tissue ischaemia,
cellular destruction and alterations in muscle cell membranes lead to the release of myoglobin into the
circulation. This circulating myoglobin results in renal injury. Advanced compartment syndrome may
result in rhabdomyolysis, and conversely, rhabdomyolysis may result in compartment syndrome.
Mortality is usually due to renal failure or sepsis from difficult wound management, hence the
importance of adequate intravenous hydration as soon as the diagnosis is suspected.

CORE SURGICAL INTERVIEW GUIDE 60


www.surgicalinterview.co.uk
HELPING YOU THROUGH YOUR CORE SURGICAL INTERVIEW 12th EDITION: 2023

1.19 A neck lump


You are the ENT SHO asked to review a 40-year-old lady who has presented to A&E with a neck
mass. She complains of a 2-month history of ‘tightness’ in her neck.

She denies pain or breathlessness, but feels that over the previous fortnight, she can feel a more palpable
mass on the front of her neck.

What questions would aid your differential diagnosis and how would you examine her?

A neck masses is a common presenting complaint. The most common causes are masses of the thyroid
or parathyroid, neoplastic origin (primary or secondary) or an infectious cause such as an abscess or
infectious lymphadenopathy. The differential diagnosis can vary considerably based on patient
demographics and the location of the mass. Specific questions from the history include; symptoms of
thyroid disease such as temperature intolerance, lethargy and unintentional weight gain or weight loss.
Risk factors for malignancy include; radiation exposure, iodide deficiency, smoking, alcohol use and
unintentional weight loss. I would ask about symptoms of systemic upset, such as fever and nausea,
which could be a sign of an infectious cause. I would also ask about a family history or personal of other
endocrine disorders, which would increase my suspicion of thyroid or parathyroid cause.

After reviewing baseline observations and ensuring the patient was stable, I would fully examine the
lump. This would include; checking for attachment to underlying structures, pain on palpation and
movement of the lump on extrusion of the tongue and on swallowing. I would examine the edges of the
mass for regularity and symmetry and look for other systemic manifestations of thyroid disease, such as
eye signs (exophthalmos and lid lag), tremor and pre- tibial myxoedema.

The patient reports no PMH, FH or DH. She has felt lethargic and tired over the past 6
weeks and has had 7lbs of unintentional weight gain.

Your examination is as follows:

Temp: 36.8 deg BP: 115/70 HR: 85 No exophthalmos or other thyroid eye signs.

4 x8 cm firm, painless irregular mass in the anterior midline. It moves with swallowing. No dominant
nodules with no other cervical masses, no cervical, abdominal or inguinal lymphadenopathy. No
myxoedema. NS: Mildly globally hyporeflexic.

What would be your next step?

The history and examination correlate with diffuse thyroid enlargement. This case would need to be
discussed with the ENT registrar on call, as well as the endocrinology registrar on call. Liaison between
the medical and surgical team in this case would be essential for optimal management.

CORE SURGICAL INTERVIEW GUIDE 61


www.surgicalinterview.co.uk
HELPING YOU THROUGH YOUR CORE SURGICAL INTERVIEW 12th EDITION: 2023

I would take thyroid function blood tests, including serum thyroxine (T4), triiodothyronine (T3) and
thyroid stimulating hormone (TSH) levels and would check calcium and parathyroid hormone (PTH)
levels to exclude parathyroid involvement.

This patient is found to have elevated TSH and a depressed T4 level. Additional blood
tests are sent and she tests positive for antithyroglobulin antibodies and
antimicrosomal antibodies. What is the most likely diagnosis?

The elevated TSH, depressed T4 and presence of specific antibodies leads to a diagnosis of Hashimoto’s
thyroiditis. However other causes of hypothyroidism should not be immediately excluded.

AMAs are 99% positive in Hashimoto’s, 80% in graves, but the latter is associated with increased T4.
Hashimoto’s is the most common cause of hypothyroidism in UK and US. Weight gain, plus lethargy
plus antibody results would make this the most likely diagnosis

How would you investigate this mass?

This patient required triple assessment with a history/examination, ultrasound imaging and a fine
needle aspiration for histology.

FNA shows diffuse parenchymal infiltration by lymphocytes and metaplasia of the


normal cuboidal cells. No malignant cells are identified. Is there a role for surgical
management of this lump at this stage?

If the FNA excluded the presence of malignant cells, there would be no indication for surgery at this
time, so long as there is no local compression of the trachea, oesophagus or superior vena cava. Surgery
would be indicated if there was evidence of malignancy, radiological or symptomatic evidence of
tracheal deviation or compression, continued growth of the goitre despite hormonal therapy or
according to patient request due to significant cosmetic deformity.

What is your management plan?

This condition is commonly associated with underlying hypothyroidism and bouts of hyperthyroidism.
This patient would be managed primarily by the endocrinology team in the absence of an indication
for surgery, with levothyroxine (thyroid replacement therapy) and regular assessment of thyroid
hormone status.

Supplementary Information

Imaging in thyroid pathology:

CORE SURGICAL INTERVIEW GUIDE 62


www.surgicalinterview.co.uk
HELPING YOU THROUGH YOUR CORE SURGICAL INTERVIEW 12th EDITION: 2023

Historically, radionuclide scanning was the mainstay of assessment. Nuclear imaging could be used to
describe a nodule as hot (autonomously functioning), warm (normal thyroid function) or cold (hypo- or
non-functioning thyroid function). 5-8% of warm and cold nodules are found to be malignant on FNA.

Ultrasonography is now more commonly used to determine the shape and size of an enlarged gland.
However, by itself, it is not able to differentiate between benign and malignant disease. US-guided
FNAB may be preferable to palpation-guided FNAB and image-guided FNAB may be particularly
helpful in the assessment of nonpalpable or small nodules.

CT or MRI scanning is generally not cost-effective in the initial evaluation of solitary thyroid nodules.
Such studies may be useful in the assessment of thyroid masses that are largely substernal. FNAB is
highly accurate, with mean sensitivity higher than 80% and mean specificity higher than 90%. The
accuracy of FNAB in diagnosing thyroid conditions depends on the cytopathologist's expertise and
experience and the technical skill of the physician performing the biopsy. FNAB is cost-effective
compared with traditional workups that heavily depended on nuclear imaging and ultrasonography.
Routine use of FNAB in the evaluation of thyroid nodules can reduce the need for diagnostic
thyroidectomy by 20-50% while increasing the yield of cancer diagnoses in thyroid specimens by 15-
45%

CORE SURGICAL INTERVIEW GUIDE 63


www.surgicalinterview.co.uk
HELPING YOU THROUGH YOUR CORE SURGICAL INTERVIEW 12th EDITION: 2023

1.20 Shin pain


You are the orthopaedic SHO. You are asked to review a 14 year old boy who complains of a dull,
but constant pain in his left shin that has been worsening over the past 4 days. 10 days ago, he fell,
scraping his left knee whilst playing football in a field near his house, but the wound has now healed.
He presents with his parents. He is otherwise fit and well.

What is your differential diagnosis?

I would want to exclude a fracture, given the history of trauma and pain. The association with fever
and recent trauma could be suggestive of an infectious process such as an infected haematoma or
bursitis. I would also consider a primary bony malignancy with associated inflammation.

How would you initially manage and investigate this patient?

I would start with a full set of observations (including BP, HR and temperature), as well as a history
(weight loss and other systemic symptoms) and examination of the painless and then the painful limb. I
would also complete a general examination, including abdominal palpation to look for occult masses,
and the cervical, abdominal and inguinal region for lymphadenopathy.

I would give appropriate analgesia as per the WHO pain ladder, using the paediatric BNF for assistance
on dosing. I would take bloods, including a full blood count (FBC), an ESR, liver function tests (LFTs),
especially alkaline phosphatase, and blood cultures, if i suspected an infectious cause for the pain. I
would order a plain film of the affected limb, including AP and lateral views of the proximal and distal
joint. I would particularly inspect these films for evidence of metaphyseal bony destruction (cortical
lucency) and periosteal reaction. However, there is potential for radiological features to lag being the
clinical presentation).

On examination

Observations: Temp 35.5 degrees HR: 79 BP: 124/75

Abdominal examination unremarkable, no hepatosplenomegaly. Right lower extremity unremarkable


with good ROM.

Left antero-lateral shin with 4 x 5 cm area of erythema and tenderness; no fluctuance or underlying
mass, no inguinal lymphadenopathy. Left knee: slight swelling, no erythema, non tender, full ROM.
Bloods: elevated ESR, slightly elevated WCC, blood cultures and left knee fluid aspirate negative for
organisms. Left tibia/fibula x-ray: no sclerotic changes, no osteolytic lesions or fracture site, periosteal
elevation in mid-distal tibia.

How would this change your differential diagnosis and your subsequent investigations?

CORE SURGICAL INTERVIEW GUIDE 64


www.surgicalinterview.co.uk
HELPING YOU THROUGH YOUR CORE SURGICAL INTERVIEW 12th EDITION: 2023

Given the history of trauma, fever, ESR/WCC elevation and cray changes, I would be suspicious of
osteomyelitis. Ultrasonography could highlight the presence of a fluid collection adjacent to the bone
without intervening soft tissue, as well as periosteal thickening and elevation. However, a ‘three phase’
bone scan could reinforce the diagnosis. Focal hypoperfusion, hyperaemia and increased focal
metabolic activity are diagnostic in the absence of a fracture.

Open bone biopsy with histopathologic examination and culture is the diagnostic gold standard, but
may not be necessary if blood culture is positive and there are radiographic changes.

Do you know the names of any scoring systems for osteomyelitis?

There are multiple classification/staging systems for osteomyelitis, including the Gordon, Kelly and
Cierny-Mader classifications, which help predict the prognosis and guide timing of surgical
management should it be required.

How would you manage this patient?

I would admit the patient and start supportive therapy, including paracetamol (anti-pyretic) and
intravenous (IV) fluid. I would also discuss the case with the orthopaedic surgery registrar on call and
ensure pain was sufficiently controlled. antibiotic therapy would be started based on the identification
of pathogens from bone cultures at the time of bone biopsy or debridement. Parenteral antibiotic
therapy would be started after blood cultures were taken, and this could be changed based on sensitivity
results. Traditionally, treatment would consist of a 4-6 week course and should be according to local
policy after discussion with microbiology.

The parents are very concerned about their son’s condition. They have read extensively
about bone pain on the internet and worry that this could be bone cancer. How would you
counsel them?

Firstly, I would consider whether at my level, I would be the most appropriate person to speak to the
parents. I would discuss this with my senior registrar and/or consultant. If I was in a position where
no senior help was available, I would ensure that I took the parents to a quiet area or the ward or
A&E department, after having first thoroughly reviewed the history or results of any investigations and
asking the permission of the patient to discuss his case with his family.

I would start by checking how much information the parents would want to know about the case, as
well as their current level of understanding to ensure that my explanation and advice was appropriately
complex or simplified. I would explain that acute osteomyelitis is a bony infection, common in children
and commonly spread in the bloodstream from a distant focus, such as a small break in the skin, often
from innocuous trauma, such as experienced by their son.

I would explain that the fever and bone pain experienced by their son are common presentations and I
would reassure the parents that they had done the right thing in presenting with their child at this stage.
I would explain that antibiotics are the first method of treatment, but that if there were a failure of
conservative management, surgical therapy could be indicated.

CORE SURGICAL INTERVIEW GUIDE 65


www.surgicalinterview.co.uk
HELPING YOU THROUGH YOUR CORE SURGICAL INTERVIEW 12th EDITION: 2023

I would address the concerns of the parents specifically with regards to bone cancer and explain we had
no suspicions of this at this stage. Regardless of this reassurance, I would explain that complications
could include the possibility of metastatic infection at distant sites, as well as spread into the joint,
resulting in septic arthritis, the development of chronic infection, pathological fracture and failure of
medical management, necessitating surgical management.

I would give the parents literature concerning osteomyelitis diagnosis, investigation and management
and direct them to relevant online materials which could aid understanding. Given the fact that family
members often would not retain all the information given to them at times of illness of a friend or
relative, I would give them contact information for a specialist nurse who would be able to give more
information at a later date.

Supplementary Information

Responsible organisms usually include

• Staph aureus (80% of cases aged 4 years to adults)


• Strep pyogenes
• Haemophilus influenzae
• Gram negative organisms

The Cierny-Mader staging system is commonly used

I. Disease involves medullary bone and is usually caused by a single organism.

II. Disease involves the surfaces of bones and may occur with deep soft-tissue wounds or ulcers

III. Disease is an advanced local infection of bone and soft tissue that often results from a
polymicrobially infected intramedullary rod or open fracture. Stage 3 osteomyelitis often
responds well to limited surgical intervention that preserves bony stability.
IV. Osteomyelitis represents extensive disease involving multiple bony and soft tissue layers. Stage 4
disease is complex and requires a combination of medical and surgical therapies, with postsurgical
stabilisation as an essential part of therapy.

Stage 1 and 2 disease usually does not require surgical treatment, whereas stage 3 and 4 respond well
to surgical treatment.

CORE SURGICAL INTERVIEW GUIDE 66


www.surgicalinterview.co.uk
HELPING YOU THROUGH YOUR CORE SURGICAL INTERVIEW 12th EDITION: 2023

1.21 Penetrating trauma


You are the surgical SHO on call at a District General Hospital. Whilst reviewing a non-urgent
patient in A&E, a team of paramedics present with a 19 year old with a stab wound to the right lower
quadrant. It is reported that he was in a knife fight where he was hit in the head with a heavy wooden
object and then stabbed in the abdomen. He is shouting loudly and is agitated. You can smell alcohol
on his breath.

How would you proceed in this case?

I would put out a ‘2222’ trauma call to ensure that experienced help was on its way. It is likely that
senior A&E help would be nearby. If this was the case, I would ask for help immediately whilst making
my way to the patient, aware of the limitations of my own competence, and my role as part of a team
to ensure optimum patient care.

I would manage this patient according to ATLS principles. This patient would be most effectively
managed by the whole ‘trauma team’, including A&E staff, anaesthetists, surgeons, etc. Depending on
the seniority of those involved, I could be involved in more of an auxiliary role in the initial management
of this patient, but as the first clinician on the scene, if I felt competent to do so, I may ‘lead’ the ‘trauma
call’ and co-ordinate the rest of the team.

The patient would likely be distressed at this point, and would need to be reassured. I would start with
a primary survey and the assessment of airway, breathing and circulation. Airway management would
include visual assessment of patency of the upper airway and removal of vomitus, blood and other
foreign bodies in a position to cause possible airway obstruction. I would assess the patient’s rate and
pattern of respiration, and position of his trachea. Assessment of his circulatory state would involve
measurement of blood pressure, attachment to a cardiac monitor and measurement of heart rate and
rhythm through palpation of a central pulse.

More generally, given the mechanism of injury, I would thoroughly inspect and examine the abdomen,
back and limbs for other penetrating injuries, take the patient’s temperature and examine his pupils.

A team member would be responsible for obtaining intravenous access, through which crystalloid
fluid/blood would be given according to the level of hypovolaemic shock noted on primary survey. I
would request a full set of bloods, including FBC, U&E, clotting and a G&S. I would also make the
patient nil by mouth. At this stage, I would also give analgesia according to the WHO pain ladder,
including IV morphine for its analgesic and anxiolytic properties.

His observations are: Temp: 36.6 Deg 146/80 HR: 116 Sats: 100% GCS: 15. What would
your next steps be?

I would continue to reassess his airway, breathing and circulation as per ATLS protocols. I would try
to determine what type of weapon was used in the attack, if the patient was sufficiently composed to
give a coherent history. This is to determine the likely depth of the wound, as well as the likelihood of
retained fragments. I would request an erect chest x-ray to look for free subdiaphragmatic air or possible

CORE SURGICAL INTERVIEW GUIDE 67


www.surgicalinterview.co.uk
HELPING YOU THROUGH YOUR CORE SURGICAL INTERVIEW 12th EDITION: 2023

rib fractures. A higher wound, (above the level of the umbilicus) would also prompt radiological
examination for pneumothorax. Plain film could also be useful for the identification of a radio-opaque
foreign body.

It is unclear at this stage whether this patient would have occult internal injury, with or without ongoing
haemorrhage. Diagnostic peritoneal lavage (DPL) has now been superseded by FAST scanning in A&E
departments.

You repeat the observations, five minutes after your previous test. The patient is still
very vocal, but his speech is now confused. He is increasingly combative. He complains
of worsening pain in his right upper quadrant. On palpation, he becomes aggressive and
tries to strike members of the clinical
team

Observations: Temp: 36.6 Deg 135/70 HR:


123 Sats: 97% GCS: 14 FAST scan: An
anechoic strip is seen in Morison’s pouch.

Given the change in clinical picture,


what form of radiological imaging do
you think would be most appropriate?

Further imaging at this stage would NOT be


appropriate. This patient is young, with
significant biochemical reserves. Despite this,
he is showing signs of developing
haemodynamic instability. An urgent
exploratory laparotomy would be indicated. As the SHO, I would ensure that O negative blood was
available, pending the arrival of blood from the cross-matched sample and would alert theatre staff to
the imminent arrival of the patient.

Supplementary Information

The focused assessment with sonography for trauma, commonly abbreviated as the FAST scan, is a
rapid bedside ultrasound examination which has become a mainstay of assessment of the management
of intra-abdominal penetrating injury and the identification of haemoperitoneum. A positive FAST scan
is characterised by fluid collection in dependent areas of the peritoneum. In the RUQ (classically hepatic
injury), this typically appears in Morison’s pouch, between the liver and the kidney. In the left upper
quadrant, this collection is in the perisplenic space. In the pelvis, fluid collects in the retrovesical space.

FAST scanning has superseded DPL is many centres as it does not involve irradiation and is less
invasive. A positive FAST scan with haemodynamic instability should be managed by emergency
laparotomy +/- proceed. Abdominal CT scan may be used if the patient is stable, but this decision
should be taken at a senior level.

CORE SURGICAL INTERVIEW GUIDE 68


www.surgicalinterview.co.uk
HELPING YOU THROUGH YOUR CORE SURGICAL INTERVIEW 12th EDITION: 2023

1.22 Post CABG complications


You are the cardiothoracic SHO on call. You are asked to review Mrs Jones, a 76 year old lady who is
30 hours post coronary artery bypass grafting (CABG) for three-vessel coronary artery disease. She
has a history of hypertension and AF, for which she was on long-term warfarin pre-operatively. This
was reversed before her procedure and her pre-op INR was 1.0.

She has had a good post-operative course and was extubated 5 hours previously. The nursing staff are
concerned because of her low blood pressure. Her CVP and radial arterial line remain in situ. Her
ionotropic demands have increased over the past 4 hours. A left pleural and mediastinal chest drain
remain in situ. Both have negligible output in the last 2 hours.

Her observations are as follows: BP: 95/60 (from 120/85 one hour previously), HR: 110, RR: 19 sats
96%. Cool peripheries. GCS: 14 (confusion). Urine output: 20 and 10 mls/hour in the previous 2
hours.

What would be your concerns and how would you further assess the patient?

My main concern in a post-operative cardiac patient would be cardiogenic shock secondary to


tamponade or early graft failure resulting in myocardial ischaemia. I would also be concerned about
hypovolaemic shock, tension pneumothorax and cardiogenic shock, as a result of excessive fluid
administration or a primary cardiac event. Whilst pulmonary embolism could fit with the clinical
picture, it would be unlikely given the heparisation of cardiac patients perioperatively.

Post-operative cardiac tamponade is a surgical emergency. I would contact the cardiothoracic registrar
as a matter of urgency. I would order an ECG and assess the features of ‘Becks Triad’. I would assess
the trend in the patient’s blood pressure over the preceding few hours, noting her history of hypertension
and the fact that her current reading could signal a severe relative hypotension on a hypertensive
background. I would auscultate her chest and inspect her neck for signs of jugular venous distension, as
well as inspecting the CVP trace. Pulsus paradoxus would further confirm my diagnosis and I would
ask the patient to take a deep inspiratory breath whilst monitoring the BP if the patient was able to
complete this.

Quiet or muffled heart sounds or a drop in inspiratory BP by > 10mm Hg would support the diagnosis
of cardiac tamponade. Deviation of the trachea from a central position could support an alternative
diagnosis of tension pneumothorax, although the time-course of symptoms progression would not
correlate with this.

I would take an arterial blood gas sample from her arterial line. This would give me information about
arterial oxygenation and Haemoglobin (Hb). A low arterial saturation would not be wholly
discriminatory, and would in some cases correlate with occult blood loss, pneumothorax or pulmonary
embolism. However, if pulse oximetry, or the arterial gas suggested hypoxia, I would start oxygen via
nasal cannulae or a facemask. A serial drop in Hb would correlate with cardiac tamponade or other
occult post-operative blood loss.

CORE SURGICAL INTERVIEW GUIDE 69


www.surgicalinterview.co.uk
HELPING YOU THROUGH YOUR CORE SURGICAL INTERVIEW 12th EDITION: 2023

The patient has muffled heart sounds on auscultation of the chest. There is still no
output from her chest drains and her blood pressure does not improve despite an initial
250ml gelofusine bolus given by the Intensive Care nursing staff just before your
arrival. Her CVP measurement remains high.

Which investigations would you order for this patient and how would you proceed?

I would take a full set of bloods from the patient, including a coagulation screen. I would check the
validity of the previous crossmatch. Creatine kinase and troponin-I measurements would be of no
benefit at this stage as they would be elevated in the post-operative patient.

I would continue fluid resuscitations with blood or 250ml gelofusine boluses in the absence of a known
history of poor left ventricular function. Further ionotropic medications could be considered. I would
also consider ordering a mobile plain chest x-ray if I felt that the patient was stable enough for a delay
whilst waiting for this investigation. I would discuss all of these measures with the cardiothoracic or
intensive care registrar on call.

What would an echo show in cardiac tamponade?

An echocardiogram could be used to visualise ventricular and atrial compression abnormalities during
the passage of blood through the chambers of the heart or diastolic collapse. I would contact an
appropriate senior member of staff to get either a transthoracic echocardiogram (TTE) or a
transoesophageal echo (TOE) if the expertise was available. Despite the value of these investigations,
the diagnosis of cardiac tamponade is a clinical diagnosis and I would ensure that I had senior surgical
and anaesthetic input from the outset.

How would this patient be managed?

Non-surgical cardiac tamponade is managed via a pericardiocentesis. However, this post operative
patient would likely have a pericardial collection due to haemorrhage, clot formation in the pericardial
sac and blockage of the outflow into the drains. If tamponande was suspected clinically, this patient
would need to be returned to theatre for a re-do sternotomy and removal of clot in the pericardial sac.

Supplementary Information

Cardiac tamponade is pressure on the heart which occurs when fluid collects in the pericardial space,
impairing the ability of the chambers of the heart to contract and to stretch normally and impairing
cardiac filling in diastole. There are a number of non-operative causes including hypothyroidism,
penetrating trauma and pericariditis. It is a challenging diagnosis, often due to the complex nature of
the patients and ITU environment and remains a clinical diagnosis. If untreated, tamponade will lead
to arrest, with PEA the likely presenting rhythm.

Imaging in tamponade

CORE SURGICAL INTERVIEW GUIDE 70


www.surgicalinterview.co.uk
HELPING YOU THROUGH YOUR CORE SURGICAL INTERVIEW 12th EDITION: 2023

Chest x-ray may show cardiomegaly, with a ‘water bottle


shaped heart’, pericardial calcification or evidence of
chest wall trauma. It could also highlight a widened
mediastinum or a large pleural collection.12 lead ECG
can highlight features that are suggestive, but not
diagnostic of pericardial tamponade. These include sinus
tachycardia, low voltage ECG complexes and PR
segment depression.

The key to successful management of cardiac


tamponade is early recognition. If you have a suspicion
about this diagnosis, especially in a post-operative patient, speak to a senior colleague urgently.

CORE SURGICAL INTERVIEW GUIDE 71


www.surgicalinterview.co.uk
HELPING YOU THROUGH YOUR CORE SURGICAL INTERVIEW 12th EDITION: 2023

1.23 Swollen Testicle


You are the general surgical SHO. You are called to see a 27-year-old man who has presented to
A&E complaining of a painless swelling in his right testicle which he noticed in the shower 4 hours
previously. He is a keen footballer and has noticed an intermittent ‘dragging sensation’ in his scrotum
over the past 3 months. He denies fever, weight loss or bony pain. He is very anxious.

What diagnoses would you consider? Which of your differential diagnoses would be
most concerning?

I would consider testicular torsion, although the presentation of painless swelling does not fit with this.
This is a surgical emergency and so would need to be excluded immediately. Given the presentation
and age of the patient, malignancy is an important differential diagnosis to consider early. Other
differential diagnoses include hydrocele, varicocele, spermatocele, haematoma, epididymitis and
inguinal hernia

The key to diagnosis would be a complete history and bimanual examination of the scrotum, abdomen
and chest, inspecting particularly for evidence of metastatic disease and systemic features of hormone
secreting tumours such as gynaecomastia.

The left testicle is normal and non tender. The right testicle has a 2cm spherical rubbery
mass at the apex. The testis and epididymis separately definable. The mass is non tender
and does not transilluminate. The swelling is confined to the scrotum.

How would you examination findings refine your differential diagnosis?

Testicular tumour would remain my working diagnosis given the findings of a discrete, rubbery, non
tender testicular mass,
Testicular torsion can be excluded in the absence of pain and definition of the testes and epididymis.
Similarly, epididymo-orchitis is classically associated with pain and tenderness. Importantly, if I had any
doubt about the diagnosis of testicular torsion, I would request immediate review from my registrar as
early intervention in torsion is essential. A hydrocele would transilluminate and the testes and
epididymis would not be easily defined on palpation. The swelling of inguinal hernia would not be
localised to the scrotum and would not be associated with a discrete mass as in this case.

At the end of your examination, the patient becomes tearful. He explains that a close
friend of his has recently been diagnosed with testicular cancer and he is ‘convinced’
that is the cause of his lump. He urges you just to ‘be honest with him’ How would you
deal with this?

I would ensure that we were in a quiet and private environment and would try to calm him. I would
ask if he wanted me to call any family members or friends. I would try to assess the level of his current
understanding and ask how much information he would like about his case, before explaining the fact

CORE SURGICAL INTERVIEW GUIDE 72


www.surgicalinterview.co.uk
HELPING YOU THROUGH YOUR CORE SURGICAL INTERVIEW 12th EDITION: 2023

that we were not yet at the point of a confirmed diagnosis and that we would need to complete more
investigations before we would be at that stage.

However, I do think it would be appropriate to address his ideas and concerns at this stage regarding
testicular cancer itself, especially if he had heard or read misleading information from friends or the
internet, or believed for example that testicular cancer was incurable. I would assure him that we would
be honest when we did have a confirmed tissue diagnosis, whilst reassuring him that there were a
number of therapies available to him, even if his worst suspicions were confirmed.

I would check his understanding of our discussion and make sure that I came back later to check that
he had been reassured and answer any other questions he may have.

How would you investigate the mass?

I would order a baseline set of bloods (including FBC and basic metabolic bloods), as well as serum
tumour markers specific to testicular cancer; alpha fetoprotein, beta HCG and LDH.

My radiological investigations would start with a scrotal ultrasound, which would evaluate the location,
size and characteristics of the lump. This patient would need to be discussed both with the surgical and
oncology specialists and would likely be referred for specialist care if malignancy was suspected.
However, CT scanning of the abdomen and pelvis would have a role in evaluating the presence of
metastatic disease, particularly in the retroperitoneum. Chest x-ray would highlight pulmonary
metastases.

Baseline bloods: Normal. AFP: Normal LDH: 253 bHCG: 8000 mIU/ml, (normal:
<5mIU/ml). Scrotal USS shows a 2cm solid mass in the right testes.

CT: 6 mm pelvic retroperitoneal lymphadenopathy with no evidence of visceral


metastases.

CXR: No pulmonary nodules.

How would a definitive diagnosis of testicular cancer be made?

A confirmed diagnosis would require histology from a tissue sample. This would be obtained from the
surgical excision of the entire testes along with the epididymis and spermatic cord.

Would you consider obtaining a tissue sample from a biopsy? What are the additional
benefits of radical inguinal orchidectomy?

No. A biopsy could cause seeding of the cancer cells from the testes itself into the scrotal sac. Radical
inguinal orchidectomy provides both histological identification, but also local tumour control.

Pathology results confirm a well differentiated seminoma.

CORE SURGICAL INTERVIEW GUIDE 73


www.surgicalinterview.co.uk
HELPING YOU THROUGH YOUR CORE SURGICAL INTERVIEW 12th EDITION: 2023

What treatment plan would be started by the oncology team?

After the removal of the affected testis by radical inguinal orchidectomy, early stage seminoma would
be treated by radiation therapy directed towards the para-aortic lymph nodes. Cure rates are reported
to be in the region of 95%, with recurrence rates of 5-15%. Alternatively, surveillance with close follow-
up may be selected for small <3cm seminomas with favourable histologic features and negative post-
operative serum tumour markers. For higher stage tumours, radiation therapy would still be considered,
though chemotherapy would be indicated for bulky, recurrent or advanced disease.

Supplementary Information

Testicular cancer is the most common malignancy in young men with the highest incidence in
Caucasians in Northern Europe or the U.S. There are 1400 new cases per year in the UK. Peak
incidence in the UK is at age 25 for teratomas and 35 for seminomas. There is 95% 5 year survival for
disease localised to the testis with no metastasis. Due to advances in chemotherapy, treatment of
testicular cancer has been labelled one of the ‘success stories of modern medicine’. Cure rates approach
85% in all cases, with better than 95% for localised disease and 80% for metastatic disease- the best
response for any solid tumour.

Classical presentation is with painless mass or swelling, but pain can occur. Gynaecomastia is caused by
bHCG production. Seminomas metastasise to para-aortic lymph nodes and produce back pain, whereas
teratomas are blood borne and spread to the liver, lungs, bone and brain.

Royal Marsden staging of testicular tumours:

I No evidence of disease outside the testis.


IM - as above but with persistently raised tumour markers.

II Infradiaphragmatic nodal involvement


IIA - maximum diameter <2 cm.
IIB - maximum diameter 2-5 cm.
IIC - maximum diameter >5-10 cm.
IID - maximum diameter >10 cm.

III Supradiaphragmatic and infradiaphragmatic node involvement


- subclassifications A,B & C based on diameter of abdominal node as above
M+ - mediastinal nodes involved
N+ - neck nodes involved

IV Extralymphatic metastases
- subclassifications A,B & C based on diameter of abdominal node as above
M+ - mediastinal nodes involved

CORE SURGICAL INTERVIEW GUIDE 74


www.surgicalinterview.co.uk
HELPING YOU THROUGH YOUR CORE SURGICAL INTERVIEW 12th EDITION: 2023

N+ - neck nodes involved

Lung involvement:
L1 <3 metastases
L2 multiple metastases <2 cm maximum diameter
L3 multiple metastases >2 cm in diameter

H+ Liver involvement

CORE SURGICAL INTERVIEW GUIDE 75


www.surgicalinterview.co.uk
HELPING YOU THROUGH YOUR CORE SURGICAL INTERVIEW 12th EDITION: 2023

1.24 Paediatric hernia


You are the paediatric surgical SHO on call. You are called to see a 6 month old girl of West African
origin who has been brought in by her parents as they are concerned about a ‘bulge’ in their
daughter’s mid-abdomen when she cries or coughs. She was born at 36 weeks, is otherwise healthy,
and is meeting growth and development targets. The ‘bulge’ always goes down when she is calm or
sleeping. She is eating well and there has been no change in her bowel habit.

On examination: 115bpm. Respiratory rate is 30 breaths per minute. BP 90/55. She is afebrile

How would you proceed?

I would ensure that the child is stable using an ABCD approach. Given that all of her observations are
normal for her age, I have no concerns at present and would reassure the parents of this. I would take
a full history and proceed to examination of the child.

On examination:

The child is playful and in no apparent distress or pain. After being


placed on the examining table, the child starts to cry. You note a
2cm x 2cm peri-umbilical mass protruding on crying, most easily
reducible on gentle palpation when the patient is calm. There is a
1cm x 1cm palpable umbilical defect at rest.

What is your diagnosis?

The history and examination would support a diagnosis of umbilical hernia.

What are the risk factors for the development of this condition?

Congenital umbilical hernia is a present in up to 30% of babies. It is caused by a failure of the umbilical
ring to completely close over. Boys and girls are equally affected. Risk factors for this include
prematurity, low birth weight, African origin and Down's syndrome.

Amongst adults, it is three times more common in women than men, with a slight female preponderance
amongst infants. [In adults, an acquired umbilical hernia can be caused by increased intra-abdominal
pressure due to a chronic cough, multiparity, obesity or straining during heavy lifting].

What are your differential diagnoses?

Umbilical hernia would fit with the presentation. It would need to be differentiated from paraumbilical
hernia in adults. Cysts of the vitello-intestinal duct could also present similarly in some cases.

CORE SURGICAL INTERVIEW GUIDE 76


www.surgicalinterview.co.uk
HELPING YOU THROUGH YOUR CORE SURGICAL INTERVIEW 12th EDITION: 2023

In a child of this age, what would be the recommended course of management?

90% of small (1-2cm) umbilical hernias close within 3 years. They have a low risk of incarceration or
strangulation. In the absence of abdominal distention or constant pain indicating visceral compromise,
observation is recommended.

You see a similar patient in a well child clinic. She is 18 months old and was advised
about conservative management of umbilical hernia at 5 months of age. The parents
are very concerned about the persistence of this lump when she coughs and strains and
are wondering if surgical management is necessary. What would you advise?

I would reassure the parents. The diagnosis and recommended management would be unchanged at
this age. The exact age at which operative management is indicated is disputed. Operative management
is often delayed until at least 3-4 years of age. At this stage, more than 85% will have spontaneously
closed. Around this age, the chance of spontaneous closure would decrease, with an increase in the
likelihood of incarceration. At 18 months of age, continued observation would be reasonable.

What causes this condition?

Paediatric umbilical hernia is caused by a failure of timely closure of the umbilical ring, leaving a central
defect in the linea alba. This fascial defect allows visceral protrusion. Adult umbilical hernia is a different
pathological entity.

Supplementary Information

Omphalocele is a differential diagnosis for umbilical hernia in the newborn. It is a congenital midline
abdominal wall defect at the base of the umbilical cord insertion with visceral herniation from the foetal
abdomen. This would be classically associated with other gastrointestinal abnormalities and up to 50%
of these patients would have associated cardiac abnormalities. It is associated with significant morbidity
and mortality

Operative closure of this abdominal wall defect should be performed shortly after diagnosis as it can
lead to significant fluid loss. Primary closure of the defect may be problematic after repair due to bowel
oedema in a relatively small abdominal cavity, therefore a system of ‘staged repair’ is often used.

CORE SURGICAL INTERVIEW GUIDE 77


www.surgicalinterview.co.uk
HELPING YOU THROUGH YOUR CORE SURGICAL INTERVIEW 12th EDITION: 2023

1.25 Epigastric pain


You are asked to see a 59-year-old woman who has attended Accident & Emergency with severe
epigastric pain spreading to her back pain and difficulty in breathing. She has been vomiting for the
last hour and denies any recent change in her bowel habit. She has a history of excess alcohol
consumption and has been experiencing difficulty in swallowing over the last few weeks.

What would be your differential diagnosis and what other aspects of the history would
you clarify?

Severe epigastric pain following several episodes of vomiting is characteristic of Boerhaave syndrome
(oesophageal rupture following vigorous vomiting). The classical features of this include excruciating
retrosternal chest and upper abdominal pain, ondynophagia, tachypnoea, dyspnoea and signs and
symptoms of shock. Her history of excess alcohol predisposes her to gastritis, gastric ulcer disease and
Barretts Oesophagus, risk factors for Boerhaave syndrome.

Other differentials include pancreatitis, peptic ulcer disease and oesophageal spasm. I would take a
detailed history about the content of the vomitus. Haematemesis may suggest a Mallory Weiss tear,
upper GI bleed or a vascular malformation of gastrointestinal tract, such as bleeding gastric or intestinal
varices. I would also consider an atypical presentation of a cardiac aetiology, such as an acute coronary
syndrome.

How would you manage this patient?

Taking an ABCDE approach I would sit the patient upright and apply oxygen via a face mask and gain
IV access with a large bore cannula. I would take bloods including an FBC, U&E, amylase, clotting and
G&S. I would immediately resuscitate the patient using colloid or crystalloid solutions if I believed her
to be clinically shocked. I would then contact my registrar for advice as this patient is unwell. I would
involve critical care early by contacting HDU to assess the number of beds available in case the patient
needed to be escalated swiftly.

I would make the patient nil by mouth, and commence antibiotics if necessary.

What investigations would you arrange?

I would organise a portable erect CXR to look for pneumoperitoneum that could highlight a
perforated viscus. I would look for pneumomediastinum and pleural effusions which are usually left
sided in oesophageal rupture. Although not common in the acute scenarios these patients can also
present with a pneumothorax and soI would check the CXR for this. The gold standard investigation
is a water soluble contrast such as gastrograffin to delineate the defect in the gastrointestinal tract.

CORE SURGICAL INTERVIEW GUIDE 78


www.surgicalinterview.co.uk
HELPING YOU THROUGH YOUR CORE SURGICAL INTERVIEW 12th EDITION: 2023

What does this image show? Does it support any of your differential diagnoses?

This image possibly from a gastrograffin x-ray shows extra-luminal contrast arising from a left,
posterolateral tear of the oesophagus. It supports a diagnosis of oesophageal rupture.

What other investigations would you order?

A CT should follow to help further assess the defect but may also identify underlying malignancy that
may have precipitated the oesophageal rupture. An endoscopy can identify where the defect is but
should be performed with caution as it may cause further disruption of the defect.

What is the definitive management for this diagnosis?

The definitive management is the surgical closure of the oesophageal defect. Small oesophageal defects
in a haemodynamically stable patient can be managed conservatively with serial haemodynamic
assessment and radiological monitoring. Surgical techniques include repairing the defect, insertion of a
stent over the defect, resection or insertion of a drain from the defect to the external environment.

Supplementary Information

Oesophageal rupture is due to iatrogenic causes such as endoscopy in 75% of cases. Boerhaave
syndrome is the spontaneous rupture of the oesophageal wall often secondary to volatile vomiting. This
is usually after excess alcohol consumption. Other causes include ingestion of caustic agents or foreign
bodies that interrupt the oesophageal wall. Oesophageal rupture tends to affect the left posterolateral
wall of the oesophagus. These patients can present with an array of symptoms which can make it
confusing to isolate the diagnosis. It is usually, but not exclusively found in men.

Macklers triad consists of three key symptoms that occur in oesophageal rupture, chest pain, vomiting
and subcutaneous emphysema. Often these symptoms overlap closely with cardiovascular and
respiratory pathology and so a thorough history is important. Remember to ask about predisposing
factors such as a history of excess alcohol intake, Barrett’s oesophagus and GORD. It is important to
start the patient on antibiotics early as often hours to days later patients can develop mediastinitis.

90% of all tears occur in the left posterolateral wall of the distal oesophagu

Advice

This is a surgical emergency and must be taken seriously. The


patient can rapidly develop hypovolemic shock and serious
airway problems as oesophageal contents leak into the thoracic
cavity. Your aim should be to initially resuscitate the patient and
immediately escalation to a senior as in some cases the lifesaving
treatment is urgent surgery by an experienced surgeon.

CORE SURGICAL INTERVIEW GUIDE 79


www.surgicalinterview.co.uk
HELPING YOU THROUGH YOUR CORE SURGICAL INTERVIEW 12th EDITION: 2023

1.26 A child with a fracture


You are the orthopaedics SHO on call. You have been asked to see a 6 years old boy in the Accident
and Emergency Department (A&E) who sustained a right elbow fracture after falling from a height of
9 feet. On arrival in A&E, you see the boy with his parents in a bay. He is crying and there is an
obvious deformity of his right elbow.

What would be your approach to management of this patient?

I would introduce myself and confirm the patient's and his parents' identity. I would then assess the
patient according to the ATLS protocol.

Firstly I would assess his airway making sure it is patent and that he has adequate C-spine stabilisation.
He is crying hence it is re-assuring that he has a patent airway. If there were any signs of airway
obstruction or concerns of impending obstruction I would maintain his airway with a jaw thrust and
consider simple airway adjuncts such as a Guedel airway if necessary. I will also seek help from the
anaesthetist immediately.

I would then assess his breathing by clinically assessing his breathing efforts. I would look for signs of
respiratory distress such as tachypnoea, intercostal recession, use of accessory muscles, grunting,
gasping, noisy breathing and central cyanosis. I would also measure his oxygen saturation using a pulse
oximetry and examine his chest to listen for any reduced, asymmetrical or bronchial breath sounds and
other added sounds such as wheeze or stridor. If there were any signs or concerns of compromised
breathing I would start him on high-flow oxygen via non-rebreathing mask and call the anaesthetist for
help immediately.

After establishing that this child has a good airway and breathing well, I would assess his circulation to
ensure he is haemodynamically stable and well perfused. I would check his capillary refill time, pulse,
blood pressure and look for clinical signs of dehydration which include reduced skin turgor, dry mucous
membrane, reduced urine output and thirst. This child will need intravenous access and urgent blood
samples to be taken. If there were any signs of shock I would start this child on fluid replacement regime.

Next I would examine his neurology status by determining his GCS level or using AVPU scale and
checking pupillary reflex. I would also ensure his blood glucose levels are normal. After this I would
perform examine the right elbow and then perform a full examination of this child to rule out presence
of other injuries. I would explain what I am doing as I go along to the patient and his parents.

While assessing this patient you find that he has a respiratory rate of 22, heart rate of
130 and a blood pressure of 100/50. He also has dry mucous membranes and
complaining of thirst. He already has intravenous access and you decide to fluid
resuscitate him.

CORE SURGICAL INTERVIEW GUIDE 80


www.surgicalinterview.co.uk
HELPING YOU THROUGH YOUR CORE SURGICAL INTERVIEW 12th EDITION: 2023

How would you decide how much of fluid you are going to administer and which type of
fluids to prescribe? (Patient’s weight = 22kg)

In a situation when a child is acutely dehydrated, he will need both maintenance fluids and
rehydration. In this scenario, this child is moderately dehydrated which equates to 5-10% dehydrated.

To calculate fluids for rehydration:

Percentage dehydration x weight in kg x 10

5% x 22 x 10 = 1100 mL

To calculate for maintenance fluids:

First 10kg give 100mL/kg/day = 4mL/kg/hr

Next 10kg give 50mL/kg/day = 2mL/kg/hr

Above 20kg give 20mL/kg/day = 1ml/kg/hr

First 10kg:

10 x 100 = 1000 mL/day

10 x 50 = 500 mL/day

2 x 20 = 40 mL/day

Total maintenance fluids = 1000 + 500 + 40

= 1540 mL/day

The volume for rehydration in addition to the maintenance fluids should be given over 24 hours.

Total fluid to be given over the 1st 24 hours = Fluids for rehydration + maintenance fluids

= 1100 + 1540

= 2640 mL/24hours

Assuming this child has normal U&Es, I would prescribe dextrosaline mixture of 5% glucose and 0.45%
saline with 10mmol of potassium chloride in each 500mL bag. If there are any electrolyte disturbances
present I would liaise with my paediatrician colleagues prior to deciding on the type of fluids to
administer.

CORE SURGICAL INTERVIEW GUIDE 81


www.surgicalinterview.co.uk
HELPING YOU THROUGH YOUR CORE SURGICAL INTERVIEW 12th EDITION: 2023

How will you assess this child's right elbow injury?

I will take a full history in particular mechanism of injury and previous injury and fractures this child
has had. I will then examine his upper limbs, assessing the neurovascular status and range of
movements of the joints of the upper limbs. I will also examine this child fully looking for any other
injuries that may be due to trauma but also keeping in mind the possibility of non-accidental injuries.
After completing my clinical assessment I would obtain radiographic imaging of the limb(s) affected
which include an AP and lateral view. I will ensure that I record my findings carefully in the patient's
notes.

Further examination of this child revealed linear whip marks and finger mark bruising
over his torso.

What are your concerns and how will you deal with it?

My main concern is the presence of non-accidental injuries. I would enquire the history of how this
child obtained these injuries both from the child and the parents in a non-judgemental manner. I
would also assess the appearance and behaviour of this child and the interaction between the child
and his parents. Based on my findings I would consider my level of concerns regarding child abuse
and risk of any immediate harm to the child. I would document my concerns in detail in the patient’s
medical notes and inform a senior colleague and the paediatrics registrar or consultant on-call prior to
discussing this sensitive issue with the parents or social services.

HELPING YOU THROUGH YOUR CORE SURGICAL INTERVIEW

CORE SURGICAL INTERVIEW GUIDE 82


www.surgicalinterview.co.uk
HELPING YOU THROUGH YOUR CORE SURGICAL INTERVIEW 12th EDITION: 2023

1.27 Post operative confusion


You are the general surgery SHO on-call at night. You have been called by the nurses on the general
surgical ward regarding a 68 year old patient who has been confused since late afternoon. She has
been wondering around the corridors and refuses to return to her bay. She gets very agitated when the
nurses try to persuade her to go back to bed and the nurses want you to prescribe medications to calm
the patient down.

What would you do?

Whilst on the phone I would make an initial assessment by asking about her observations and ensure
that the patient is clinically stable. I’d go to the ward to assess the patient at the first opportunity I had,
and certainly before prescribing any medications. Once there, I would recruit the expertise of the
nursing staff to help bring the patient to a safe place. If the patient was very aggressive I would seek
advice from the registrar on-call and get help from security at an early stage.

Once safe I would assess her, clinically and try to find out from her what is causing her to be distressed.
Assuming her ABC’s are stable (and if not I would measure and treat as per ATLS) I would assess her
confusion, by assessing her GCS. A BM measurement is essential at this stage.

Once I’m convinced she is safe I would also go through her medical notes, recent blood tests and other
investigations to find out whether her confusion was new and if she has had any predisposing factors
such as recent surgery, critical care admission or a catheter that could have increased the possibility of
an infection. I would gather all the information, make my own assessment then discuss the case with my
registrar.

Once a reversible cause for her confusion is found she will need to be started on treatment for it. If there
were no obvious causes and there were no surgical causes for her confusion this lady will require input
from the medical team for ongoing care.

She may need medication temporarily to keep her calm and I would seek advice from the medical
registrar on-call regarding medications suitable to prescribe for this lady.

You examine this lady and find that she is pyrexial at 38.5 degrees Celsius, has a heart
rate of 130, blood pressure of 90/60 and respiratory rate of 22 breaths per minute. ECG
shows fast atrial fibrillation and her known past medical history includes
hypertension, ischaemic heart disease and type 2 diabetes. You also noted that she is
day 5 post-operation. She had a low anterior resection for a malignancy and her
surgical wound has pink fluid oozing from it. She is very tender on palpation of her
abdomen. Nurses reported to you that she has not been eating and drinking much and
has not open her bowels yet.

How are you going to manage this situation?

CORE SURGICAL INTERVIEW GUIDE 83


www.surgicalinterview.co.uk
HELPING YOU THROUGH YOUR CORE SURGICAL INTERVIEW 12th EDITION: 2023

I will manage this lady first according to the ALS guidelines. This lady has signs of shock likely secondary
to post operative abdominal sepsis. This lady needs fluid resuscitation and I will initially give her a bolus
of intravenous fluid to assess her response. If she responds well she will require maintenance fluid and
if not she will need further boluses of intravenous fluids. I will also perform an arterial blood gas to assess
her metabolic status and respiratory function. Then I will immediately inform my registrar about her
as she is unwell and as per my registrar’s advice, arrange for an urgent abdominal CT scan. I would
seek microbiology advice regarding empirical antibiotics and ask the critical care outreach team to assess
her. I would also ask the nurse to perform hourly observations, and assess her myself again regularly.

The investigations results are as follows

ABG:
pH 7.29
PaO2 11.7 kPa
PaCO2 3.9 kPa
HCO3- 10 mmol/L
BE -4
Lactate 7

Bloods
Hb 120
WBC 18
Platelets 700
Neutrophils 8.5

Na+ 143
K+ 4.2
Urea 17 (baseline ~ 6)
Creatinine 190 (baseline ~ 70)
CRP 186

What do the results show?

This lady has raised inflammatory markers showing likely infection and she has developed a metabolic
acidosis secondary to sepsis and is trying to compensate by increasing her respiratory rate. She is in
acute renal failure.

Her CT scan showed intra-abdominal collection. You inform your registrar regarding
this however he is currently scrubbed in theatre for another emergency case and is
unable to review the patient but asks you to consent the patient for an emergency
laparotomy and discuss with HDU for a bed post-operatively. You are not trained to

CORE SURGICAL INTERVIEW GUIDE 84


www.surgicalinterview.co.uk
HELPING YOU THROUGH YOUR CORE SURGICAL INTERVIEW 12th EDITION: 2023

take consent yet and the hospital policy is for at least a registrar review for any patient
to be referred to HDU and ICU.

What would you do?

I’m not competent to take consent and therefore have to inform my registrar of this. Instead I will try
to facilitate the surgery by booking the case onto the emergency list and informing the anaesthetic and
theatre team. I will aid the consent process by getting the consent form ready and inform the patient
and family about her current clinical situation.

Regarding the consent, this lady appears confused but ideally her wishes and her next of kin and family’s
views should be consulted. In emergency situation like this one we would treat her in her best interests.
The patient will probably require a consent 4 form if she is unable to understand, weigh, retain the
information or communicate her decision.

I would also ensure the patient is optimised for surgery, and contact the critical outreach team for their
support. I would escalate to the consultant on-call immediately as this patient is critically unwell and
will need urgent senior review. If I am unable to get in touch with the consultant on-call, I will then
contact the consultant who is responsible for this patient’s care to ensure patient care is not delayed.

CORE SURGICAL INTERVIEW GUIDE 85


www.surgicalinterview.co.uk
HELPING YOU THROUGH YOUR CORE SURGICAL INTERVIEW 12th EDITION: 2023

1.28 Wounds and dressings

How would you classify a wound?

One can classify a wound by type, thickness, and age.

Type describes how the wound was created. For instance; abrasions (superficial), incision (from a
surgeon’s knife), laceration (often confused, but an irregular wound caused by a sharp object on soft
tissue), crush (heavy blunt objects, such as a falling tree on your leg), stab, puncture, bullet, bite and
burns (thermal, chemical or electrical).

Depth of the wound, like the classifications of burns can be separated into superficial, partial thickness
and full-thickness. Superficial wounds affect the epidermis and the upper dermis. Partial thickness
wounds contain damaged tissue up to the subcutaneous tissue. Full thickness wounds involving all layers
of the skin and subcutaneous tissue, down to fat and bone. Wounds can also extend down to or through
organs.

Classification by age as clinical significance as older wounds are less amenable to primary surgical
closure, and susceptible to contamination or tissue necrosis.

How might you classify wound healing?

There are three types of wound healing. Healing by first intention is where the wound is closed by
opposing the tissue margins surgically, most commonly with sutures, clips or steristrips or by using tissue
grafts from other sites. Healing by secondary intention is where the wound is left open so that
granulation tissue can gradually fill the intervening gap and close the wound without surgical
intervention. Healing by tertiary intention represents delayed closure, where the wound, usually because
of contamination is deliberately left open to heal, and then closed at a later date once sufficient
granulation tissue has occurred and the infection resolved.

What are the stages of wound healing?

The phases of wound healing include an inflammatory phase, a proliferation phase and a maturation
phase.

What factors affect wound healing?

Both intrinsic and extrinsic factors affect wound healing.

Intrinsic factors include the type, depth, age, size and anatomical location of wound. Tibial lacerations
for instance are partially susceptible to slow wound healing due to thin tissue layers and poor vascular
supply. Local infection of the wound is perhaps the most common reason for poor wound healing.

CORE SURGICAL INTERVIEW GUIDE 86


www.surgicalinterview.co.uk
HELPING YOU THROUGH YOUR CORE SURGICAL INTERVIEW 12th EDITION: 2023

External factors include the patient’s clinical and nutrition status, their age, the presence of co-existing
systemic infection or disease and the therapy they are receiving. Steroids and radiotherapy are
particularly detrimental to wound healing. Co-morbidity such as diabetes mellitus, heart disease,
smoking, and vitamin deficiencies C and K also can delay wound healing significantly.

What operations are clean and what are considered contaminated?

Clean operations are non-traumatic operations that don’t enter the respiratory, gastrointestinal or
genitourinary tracts. Clean contaminated operations are non-traumatic operations which involve
respiratory, gastrointestinal or genitourinary tracts. Contaminated operations include fresh traumatic
wounds from a clean source or significant involvement of the respiratory, gastrointestinal or
genitourinary tracts. Dirty wounds are traumatic wounds from an unclean source or traumatic wounds
with delayed closure.

Why might one debride a wound?

To ensure adequate wound healing, the wound edges and base need to be clean and have an
appropriate vascular supply to ensure an adequate supply of nutrients, cells, and allow growth and
immune factors to enter the wound. If the wound has become infected, necrotic or non-viable tissue
then this will inhibit good wound healing. Surgical debridement aims to remove this infected, necrotic
or non viable tissue to allow optimal wound healing.

Why do we use wound dressings?

Wound dressings protect the wound from contamination, help alleviate pain, and reduce moisture
loss.

What is the ideal characteristics of a wound dressing?

This is relatively similar to the ideal characteristics for suture material, essentially it should be inert, not
promoting inflammation or allergic reaction. It should ensure adequate protection from contamination
from bacteria and dirt, keep the wound moist, adherent but allows mechanical movement, absorbs
exudate, pus and odour from the wound and be cost effective and relatively simple to apply.

What are the different types of wound dressings and what different types of wound
dressings do you know?

Wound dressings can be synthetic (most dressings), non-synthetic (skin grafts), temporary (most
dressings) or permanent (skin grafts).

Types of wound dressing include:

CORE SURGICAL INTERVIEW GUIDE 87


www.surgicalinterview.co.uk
HELPING YOU THROUGH YOUR CORE SURGICAL INTERVIEW 12th EDITION: 2023

• Cotton gauze

• Impregnated gauze

• Hydrocolloids

• Silicones which act similarly to skin in allowing adequate mechanical movement and wound
protection but little absorptive capacity.
• Barrier films

• Tissue adhesives (such as tissue glue found in emergency departments for superficial wounds)

• Hydrogels which contain significant percentage of water, useful in dry and necrotic wounds to
keep the healing area moist.

• Hydrocolloids which are good at absorbing fluids

• Alginates that are useful in the most exudative of wounds, for instance following incision and
drainage of an abscess.

• Vacuum dressings, which are usually film dressings with the addition of a pump to create a
negative pressure vacuum around the wound site, with the aim of encouraging the excretion of
exudate and pus to speed wound healing.

CORE SURGICAL INTERVIEW GUIDE 88


www.surgicalinterview.co.uk
HELPING YOU THROUGH YOUR CORE SURGICAL INTERVIEW 12th EDITION: 2023

1.29 Epigastric Pain II


You are telephoned to see a 65-year-old man presents to A&E, as the general surgery SHO on call.
A&E report that on examination, the patient looks unwell. He is pale and clammy, with marked
epigastric tenderness. He has vomited once. He is normally fit and well, although is troubled
occasionally by mild upper abdominal pain after eating. He smokes socially and admits to drinking
more than he should, four or five pints most evenings after work.

What are your differentials?

Using a systems' approach, my differentials would be as follows: gastrointestinal causes might include
peptic ulceration with or without perforation, pancreatitis, appendicitis or bowel obstruction. Of these,
pancreatitis or peptic ulceration seems most likely, because of the history of epigastric pain and alcohol
excess. Biliary causes include gall stones and related complications. Urological causes include renal colic
or a pyelonephritis. Cardiovascular causes would include an abdominal aortic aneurysm or myocardial
infarction, which would be important to exclude given the smoking history. Finally, in the respiratory
system, a lower lobe pneumonia could cause referred pain in the abdomen, however this is unlikely as
there is no mention of respiratory symptoms.

Advice

Always structure your answer when asked to list potential diagnoses and give the more obvious ones
first. Provide simple justifications for your answers - one sentence, or two, just enough to show that you
have taken the history and examination into account, rather than only hearing "epigastric pain".

The suspicion is of pancreatitis. How would you approach the patient initially?

Over the phone I would ask for the initial observations and take a quick history so that I could ascertain
whether immediate escalation was required. Once with the patient, I would assess his airway, breathing
and circulation according to ALS principles, while simultaneously initiating resuscitation.

Airway - I would ensure no comprise, e.g. from vomiting, and apply high flow oxygen. Breathing - I
would looks for signs of respiratory distress, including assessing oxygen saturations, respiratory rate
and auscultation of the chest.

Circulation - I would look for signs of haemodynamic instability. I would gain intravenous access and
send off haematological investigations as appropriate, following this I would start aggressive fluid
resuscitation with crystalloid boluses, aiming to achieve a urine output of at least 0.5ml/kg/hr.

Disability - I would regularly assess the patient's consciousness level, ensuring they had adequate
analgesia according to principles of the WHO analgesic ladder, and I would monitor blood sugars.

CORE SURGICAL INTERVIEW GUIDE 89


www.surgicalinterview.co.uk
HELPING YOU THROUGH YOUR CORE SURGICAL INTERVIEW 12th EDITION: 2023

Exposure and everything else - I would palpate the abdomen, looking for signs of peritonism or bruising.
With a diagnosis of pancreatitis in mind, I would start intravenous pantoprazole, while keeping the
patient nil by mouth.

I would pass a nasogastric tube and insert a urinary catheter. I would organise timely investigations,
including erect chest x-ray, ultrasound and abdominal CT.

I would liaise with my seniors and my intensive care colleagues as this patient needs at least HDU
care.

Advice:

The text above is written as 'Airway - Breathing - ' etc. for ease of reading. When answering in real life,
make the transition between each smoother. For example, "moving on to assess breathing, I would...".

The clue is the question - "the suspicion is of pancreatitis". Do not be afraid to tailor your answer to
this. As always, coherent, concise, un-rushed presentation is crucial. You want the examiner to feel
that the calm they see before them is the level of calm you would demonstrate in a pressured situation
like the one described.

Tell me in more detail what investigations you would consider?

I would divide investigations into four categories; simple, bedside tests, haematological, radiological and
more specialised tests.

At the bedside I would ensure a urine dip was performed, looking particularly for blood, protein or
sign of infection. I would ask for basic observations, a blood glucose and a 12-lead ECG.

Among my blood tests I would include an FBC, U&Es, LFTs, amylase, a blood gas (pH, calcium,
lactate), coagulation, group and save. These would enable me to score the severity of the pancreatitis, if
this turns out to be the diagnosis.

Regarding imaging, I would order an urgent erect CXR to look for air under the diaphragm, but also
to exclude a pneumonia. Depending on my examination findings, I would consider an abdominal
radiograph to look for dilated bowel loops or stones.

Further, specialised test might include a FAST scan, to look for free fluid within the abdomen, or indeed
a formal ultrasound assessing the biliary tree and pancreas. I would consider a CT KUB for renal stones,
or with pancreatic protocol to look for pancreatitis.

Advice

It might seem odd to structure your investigations - but again, it gives you a fall-back if your mind
goes blank. It also demonstrates a logical train of thought that the examiners will find appealing.
Never forget an erect CXR in abdominal pain, likewise - a pregnancy test in female patients.

CORE SURGICAL INTERVIEW GUIDE 90


www.surgicalinterview.co.uk
HELPING YOU THROUGH YOUR CORE SURGICAL INTERVIEW 12th EDITION: 2023

Investigations are as follows:

HR 120, BP 90/60, temp. 37.7, RR 30

Hb 130, WCC 17, CRP 85, urea 17, creatinine 120, amylase 700, bilirubin 23, AST 150,
albumin 35 Pa02 9, PaC02 4.1, lactate 3, glucose 7, calcium 1.9

Q. What scoring systems do you know for assessing severity of pancreatitis?

Glasgow, Ranson.

Q. What is this patient's Glasgow score and how did you obtain it?

The patient’s Glasgow score is 4.

The patient scores 1 for each of age > 55, WCC > 15, urea > 16, calcium < 2.

Scoring is as follows (PANCREAS):

• P02 < 8kpa

• Age >55

• Neutrophils (WCC) >15

• Calcium <2 mmol/l

• Renal: Urea >16 mmol/l

• Enzymes: AST>200 IU/l, LDH > 600 IU/l

• Albumin <32 g/dl

• Sugar: Glucose >10mmol/l

Q: What are causes of pancreatitis?:

• Gallstones

• Ethanol

• Trauma

CORE SURGICAL INTERVIEW GUIDE 91


www.surgicalinterview.co.uk
HELPING YOU THROUGH YOUR CORE SURGICAL INTERVIEW 12th EDITION: 2023

• Steroids

• Mumps and other viruses

• Autoimmune, including SLE

• Scorpion sting

• Hyperlipidaemia

• ERCP

• Drugs (Mnemonic - ‘GET SMASHED’ - think it, but don't say it!)

Advice:

Don't simply reel off the list, specify that the first two are the most common, but that there are other,
rarer causes. You then may mention one of two of the rarer causes, as directed by any clues in the
clinical scenario. Simply going through the list verbatim gives the impression of someone who is at
medical school level and can learn facts, but hasn’t yet learnt to filter what information is important
and relevant. In addition it wastes time, and can lead to unnecessary questions which you don’t know
the answers to. One favourite (sadistic) examiners’ trick is ask candidates who mention scorpion stings,
which particular species of scorpion has venom which causes pancreatitis, then where they can be
found, what time of year it is active, whether the male or female is venomous etc…

By making your answer relevant, you can avoid such pitfalls!

Q: What is the likely cause of the low calcium?

This is most likely caused by fat necrosis occurring as a result of released enzymes leads to the release
of triglycerides. These combine with calcium in a process called saponification, leading to
hypocalcaemia.

Q: What eponymous signs may be associated with pancreatitis? Describe them.

Grey-Turner's sign is bruising on the flanks secondary to retroperitoneal haemorrhage tracking from
the pancreatic area.
Cullen's sign is peri-umbilical bruising secondary to pancreatic enzyme tracking to the anterior
abdominal wall and subsequent tissue digestion.

Advice:

Sometimes it’s useful to know a few choice facts about the commonly encountered conditions, because
even if you aren’t asked directly, adding them into your answer looks good, and may also come up in
your MRCS examinations.

CORE SURGICAL INTERVIEW GUIDE 92


www.surgicalinterview.co.uk
HELPING YOU THROUGH YOUR CORE SURGICAL INTERVIEW 12th EDITION: 2023

1.30 RTA
You are the surgical SHO on-call and you get a trauma call informing you that a 35 year-old
gentleman involved in a motorcycle road traffic accident travelling at 35 mph will arrive in 10 min.

What would you do?

Prior to the patient arriving I would go to resus in A+E and introduce myself as the surgical SHO on-
call. I would ascertain who the other members of the trauma team are and clarify that I will be leading
the trauma call. Each member of the team would be given a specific role to facilitate timely assessment
and resuscitation of the patient. Each member of the team would wear apron and gloves, and a final
equipment check will be performed.

What would you do once the patient arrives in resus?

I would perform a ‘ABCDE’ primary survey according to ATLS principles. The first step would be to
perform airway maintenance with cervical spine protection. I would ascertain airway patency by talking
to the patient whilst rapidly assessing for signs of airway obstruction.

Informed by the examiner the patient is able to communicate verbally.

As the patient is communicating verbally, the airway is unlikely to be in immediate danger. However,
frequent repeated evaluation of the airway should be performed during the primary survey I would
then immobilise the C-spine with 3-point in line immobilisation using collar, blocks and tape.

I would assess breathing to ensure adequate ventilation and oxygenation. Starting with inspection, I
would carefully expose the neck and chest to assess for equal and bilateral chest expansion, jugular
venous distension, use of accessory muscles and any signs of injury. I would then determine the
patient’s respiratory rate. I would palpate the trachea and chest wall to assess for tracheal deviation
and equal chest expansion. I would then percuss the chest and auscultate the chest bilaterally. During
my chest examination, I am actively assessing for injuries that may severely impair ventilation and
require immediate treatment. These include tension pneumothorax, open pneumothorax, massive
haemothorax, flail chest and pulmonary contusion.

Informed by the examiner the patient does not have any of the above injuries.

I would administer high flow oxygen, 15L/min, through a non-rebreathing bag and attach a pulse
oximeter to the patient to ensure adequate oxygen saturation. I would then ask a member of the team
to perform an arterial blood gas. Before moving onto circulation, I would reassess the patient’s airway
and breathing.

CORE SURGICAL INTERVIEW GUIDE 93


www.surgicalinterview.co.uk
HELPING YOU THROUGH YOUR CORE SURGICAL INTERVIEW 12th EDITION: 2023

Informed patient’s airway is patent and nil change in the patient’s breathing status.
Patient’s RR is 25 and saturations 97%.

The next step is circulation with haemorrhage control. I would assess the patient’s skin colour, pulse,
and capillary refill time, and ask for a blood pressure reading.

Informed the patient is tachycardic at 120 with a BP of 110/90, and a CRT of 4 sec.

The patient is currently in at least class II shock. I would insert 2 large bore cannulas into the
antecubital fossa and simultaneously obtain blood for FBC, U&E, LFTs, clotting and G&S, and a
venous gas for a lactate level if an arterial blood gas has not been obtained. I would also cross-match 4
units of blood. I would initiate intravenous therapy by prescribing a 1L bolus of warmed Hartmann’s
solution. I would identify and control any source of external bleeding by applying direct pressure to
the external bleeding site.

Informed no external source.

I would then consider the presence of internal bleeding.

What are the major areas of internal haemorrhage?

The major sites of internal bleeding are the chest, abdomen, retroperitoneum, pelvis and long bones.

The abdomen is slightly distended and tender.

I would then reassess the patient by assessing airway, breathing and circulation.

Informed the patient had a transient response to the initial bolus with nil changes to the
patient’s airway and breathing.

I would prescribe another 1L bolus of warmed Hartmann’s solution and request 1-2 units of type-
specific blood to be transfused to the patient.

What bedside test could be performed in this situation?

A FAST scan can be performed at the bedside as a tool to detect intra-abdominal bleeding.

To complete the primary survey, I would perform a brief neurological examination to assess the
patient’s disability including patient’s GCS, pupil size and reaction, and any lateralizing signs. I would
then expose the patient by completely undressing them but avoiding hypothermia, perform a log roll
and a digital rectal exam. If a urethral injury were not suspected, I would ask for a urinary catheter to
be inserted so that the patient’s fluid status and response can be monitored. I would now reassess the
patient by repeating the ABCDE examination

Would you inform a senior?

Once I have completed the primary survey and initiated resuscitation I would urgently inform my
senior registrar about the patient.

CORE SURGICAL INTERVIEW GUIDE 94


www.surgicalinterview.co.uk
HELPING YOU THROUGH YOUR CORE SURGICAL INTERVIEW 12th EDITION: 2023

Would you send the patient for a CT scan of the abdomen now?

The patient may need a CT scan in the future but is currently not haemodynamically stable enough
to go for a CT scan. At this stage I felt I was doing well in the interview and had developed a rapport
with the examiners. I ended the question by saying “I do not want to send the patient to the doughnut
of death”. The examiners liked the joke and laughed in response.

CORE SURGICAL INTERVIEW GUIDE 95


www.surgicalinterview.co.uk
HELPING YOU THROUGH YOUR CORE SURGICAL INTERVIEW 12th EDITION: 2023

1.31 Post operative assessment


You are the surgical SHO and a nurse calls you about a post-operative patient whose PAR score had
deteriorated.

What would you do?

As the nurse initially called me, I used this opportunity to demonstrate my knowledge and prior use of
the SBAR communication framework. I started by saying I would use the SBAR framework to
communicate with the nurse and learn about the patient’s condition.

What does SBAR stand for?

It stands for Situation, Background, Assessment, and Response/Recommendation. What is the


situation?

The situation is that a 19 y/o man, day-3 post open appendicectomy has become
unwell. The background is that the patient was admitted 4 days ago with RIF pain. He
underwent an open appendicectomy the next day.

Any PMH or previous surgery?

He has no previous PMH.

Assessment?

On assessment he is tachypnoeic with a respiratory rate of 24, tachycardic with a HR of


122 and has a temperature of 38.9°C. What is your response/recommendation?

Does the patient have any allergies?

No

Can you please start some intravenous paracetamol and IL of Hartmann’s solution? I am on my way
to assess the patient.

Once I am on the ward, I would assess the patient using the CCrISP system of assessment. Immediate
management would be an ABCDE assessment of the patient. I would initially assess airway patency
by asking the patient ‘How are you?’. An adequate response would inform me the patient’s airway is
currently patent.

The patient is talking

CORE SURGICAL INTERVIEW GUIDE 96


www.surgicalinterview.co.uk
HELPING YOU THROUGH YOUR CORE SURGICAL INTERVIEW 12th EDITION: 2023

I would then administer high flow oxygen, 15L/min through a non-rebreathing bag and assess the
patient’s breathing by inspecting, palpating, percussing and auscultating the patient’s chest. I would
determine the patient’s respiratory rate and ask the nurse to attach a pulse oximeter to calculate the
patient’s oxygen saturation. Is there anything wrong with the patient’s chest?

Apart from a RR of 24, patient’s breathing is normal with nil acute Tindings

I would then assess the patient’s circulation and perfusion by palpating both central and peripheral
pulses and determining capillary refill time.

The patient has a bounding pulse with a HR of 120 and CRT<2sec. Their temperature is
38.9°C

I would secure at least 1 large cannula, minimum 18G calibre, and take blood cultures, a full set of
bloods including FBC, CRP, U&E, LFTs and clotting, and a VBG for a lactate. I would then ask the
nurse to administer 1L Hartmann’s STAT and intravenous antibiotics as part of the sepsis 6 bundle.

What is included in the sepsis 6 bundle?

The sepsis 6 bundle includes high flow oxygen, blood cultures and a full set of bloods, intravenous
antibiotics, intravenous fluids, lactate and monitoring urine output. I would then determine the
patient’s neurological status and expose the patient. During this part, I would carefully examine the
patient’s abdomen by inspecting the wound site for any infection or discharge. I would assess for
abdominal distension and palpate the abdomen to assess for any tenderness or peritonism.

Patient has severe tenderness around the wound site and right side of the abdomen with
signs of local peritonism.

Once I have completed my initial assessment I would re-assess the patient to ensure my resuscitation
measures have improved the patient’s condition or to identify further deterioration and need for
immediate senior help. At this stage, if the patient were stable, I would perform a full patient
assessment. This would include reviewing the patient’s observation chart, drug chart, blood test results
and operative notes.

What would you look at in the patient’s operative notes?

I would look at the patient anaesthetic chart to see if there were any peri-operative issues, the
medication administered peri-operatively and the surgeon’s operative notes. I would look at the
operative finding’s to see if any signs of infection was noted and it so whether it was contained or
spread through out the site/abdomen. The findings would help classify the surgical procedure as
clean-contaminated, contaminated or dirty.

Would you inform your senior?

I would inform my senior registrar early as the patient may require imaging in the form of an
ultrasound scan or CT abdomen and pelvis, and further surgical intervention

CORE SURGICAL INTERVIEW GUIDE 97


www.surgicalinterview.co.uk
HELPING YOU THROUGH YOUR CORE SURGICAL INTERVIEW 12th EDITION: 2023

SECTION 2:
Portfolio Station

CORE SURGICAL INTERVIEW GUIDE 98


www.surgicalinterview.co.uk
HELPING YOU THROUGH YOUR CORE SURGICAL INTERVIEW 12th EDITION: 2023

2.1 Portfolio station


Introduction

Your portfolio is a key piece of the interview process. It is like an extended CV, a chance to demonstrate
your achievements during your medical school, foundation years and beyond. Surgical applications at
CT1 level are increasingly competitive and you will be competing against your peers who will have
similar aims.

Your portfolio station is worth 33.3% of the total marks for your interview as a whole. In 2015 a pilot
leadership skills presentation was run. This is expected to be formally included in 2019. Its a great
opportunity to showcase yourself - as long as you are properly prepared.

The role of the portfolio station is to assess your achievements to date. Assessment is threefold; through
the answers you gave in the application form, through assessment of your portfolio and through your
answers to the interviewers questions. Compared to previous years there is now a much heavier focus
on the drop down questions from your application. Essentially the interview is tasked with assessing the
evidence you have to back up those achievements.

Before the start of the interview, portfolios are collected so that the interviewers can mark them against
a standardised scoring sheet. This mark combined with a score generated from your portfolio and your
answers to the questions in the station give a total score.

The job of the interviewer is a difficult one as they have a very limited amount of time to go through
your portfolio and your task is to help them give you the marks. A well-structured and organised
portfolio allows the interviewer to quickly find the relevant information. We provide a guide to the
portfolio below, which you should take time to review. Candidates who attend our interview course will
get the chance to see how the portfolios of the top-ranking candidates from previous year s were
structured, and have their portfolio personally assessed by the same candidates. There’s always room
for improvement, so if you don’t get a place on the course, make sure that you show your portfolio to a
senior surgeon, and to a good trainee who has recently been through the interview.

The questions asked during your portfolio station are designed to allow you to show the interviewers
the evidence they might not have found themselves.
In 2015 a leadership presentation pilot was run. Each applicant was required to present a pre-prepared
(non powerpoint) 2 minute leadership biography This was followed by 3 minutes of questioning by the
panel. In 2019 it is expected (at the time this went to press) that this pilot would become a formal part
of the station for 2019.

It is essential that you know your portfolio well and can turn to the relevant section quickly. During the
course, you should be prepared to do this under pressure during both the small group sessions, and the
mock interviews.

Here’s one candidates experience of the portfolio station last year.

CORE SURGICAL INTERVIEW GUIDE 99


www.surgicalinterview.co.uk
HELPING YOU THROUGH YOUR CORE SURGICAL INTERVIEW 12th EDITION: 2023

“During my interview, there were 2 very friendly interviewers which helped calm my nerves. They had
already gone through my portfolio and marked some components on their marking sheet. One of the
first questions they asked me was ‘’I see you have attended a number of courses such as the BSS, ALS
and ATLS which are very good but why have you not done the CCrISP course?’’. Not really a question
I was expecting so I have not rehearsed an answer for this. I answered truthfully and explained how I
did try to attend the course however was let down by many institutions as core surgical trainees were
given priority to attend the course:

‘’I understand the CCrISP course is a very useful course to attend for surgical trainees as it trains trainees
on how to deal with sick surgical patients efficiently which is an essential skill to have. Hence I contacted
many institutions to book a place for myself on the course however at most I was put on the waiting list
as priorities were given to core surgical trainees and I was a foundation trainee when I applied. Places
were also very limited on the CCrISP course. Although I did not manage to get myself onto the course,
I have borrowed the CCrISP manual from the hospital library which I have found very useful. I also
plan on attending the course as soon as I start my training as a core surgical trainee.’’

The interviewers responded in a very understanding manner. They said that this is a very common
problem faced by foundation doctors who are interested in attending the CCrISP course and that the
college should think of a way to overcome this. I was then asked:

‘’You have many done audits and presentations which are good. But how and what do you think you
could improve in your CV?’’

At that particular moment, as a foundation year 2 trainee who has just recently figured out what I
would like to do for the rest of my life i.e. surgery, I could think of a million ways of how to improve
my CV.

I thought about the question for a while and decided to answer this question in a similar way I would
talk through my CV which is in order of the mnemonic CAMP (clinical, academic, management,
personal) but modifying my answer to suit the question at hand:

‘’There are many ways I can improve my CV. As a trainee pursuing surgery, clinically, I will try my
best to spend more time in the operating theatre to gain more experience and increase the number of
my logbook entries. At the same time, I will ensure that my clinical duties in the ward are not
compromised and that there is adequate cover while I am in theatre. I will also get as many work-
based assessments completed as possible with constructive feedback from my assessors so that I am
able to reflect on my clinical practice and ensure progression.

I also plan on expanding my academic interest. Although I have been involved in teaching medical
students a few times, I would like to be more involved in regular formal teachings for junior doctors as
well and journal club meetings as this not only helps me consolidate my own learning but also helps my
colleagues and I keep ourselves up to date with the advancements in surgery.

I have previously led audit projects and completed four audits however I have yet to complete an audit
cycle. This is something I am very keen to pursue as I will be able to make a bigger contribution to my
department with a completed audit cycle which results will give a better indication on how to improve
current clinical practice.

CORE SURGICAL INTERVIEW GUIDE 100


www.surgicalinterview.co.uk
HELPING YOU THROUGH YOUR CORE SURGICAL INTERVIEW 12th EDITION: 2023

Personally, I will ensure I continue to be supportive and understanding with my colleagues and other
healthcare professionals at work as I would like to continue having good feedback on my multi-source
feedback assessments and hopefully even more encouraging and positive feedback. Out of medicine, I
would like to continue with my diving activities and obtain an advanced diver license. These are a
number of ways that I have planned to both improve my CV and help with my professional and personal
progression. I will outline my objectives in the coming year with my educational supervisor and layout
a clear course of action to achieve these.’’

I was then asked to talk about my research experience, so I talked through my research experience both
as a medical student and the 2 research projects I was involved in during my foundation training. I
described each research project briefly and outlined my role in these projects. I also elaborated on the
research-related and general skills I learnt, the outcomes and the presentations from these projects. On
top of that, I included the journal club meetings I attended, papers I have presented during these
meetings and expanded further on the critical appraisal course I attended.

This was followed by question on which surgical field I would like to pursue my career in. I did not
commit myself to one field for this question as this was true. I explained that I was interested in both
ENT and Neurosurgery as a career, explained how I developed my interest and gave the pros and cons
of both fields. I also pointed out the evidence I have in my portfolio as proof that I have taken steps to
further my experience in both surgical specialties which I think helped me to answer the question a lot
as it reminded me of what I have in portfolio to expand my answer. It is possible that I was asked this
question as my CV especially during my foundation years were mainly ENT- and neurosurgery-
oriented. I also used this chance to point out why core surgical training would be ideal for me as I would
be given more time and experience both surgical specialties.”

7 top tips from previous candidates

1. When preparing for the station, it is a good idea to look through your portfolio and write a list
of possible questions you may be asked on what you have and do not have in your CV. Know
everything in your portfolio. Interviewers can select anything to discuss. For example, if you
have not done any teaching previously, it may be possible you get asked why you were not
involved in teaching. During the Core surgical interview course we will show you how to
organise your portfolio in a way that helps to direct the interviewer towards the achievements
you want to speak about, rather than to that tiny audit you did 2 years ago that if difficult to
discuss.
2. Have a system on how to answer various questions so that even if you are asked a question you
have not practiced before you still have something to refer back to. Several techniques are
mentioned in the guide and we’ll put you through your paces, in the fun, but high pressured
environment of the small group sessions.
3. Practise both the standard questions for interviews and questions personalised to your CV.
Prepare for as many questions as you can. It really helps and makes a difference. We provide a
lot more questions than are required below based on the principle that the more chance you
have to practice the better.
4. Always remain calm! Don’t panic when you are asked a question you did not expect. Stay calm
and think of how you would roughly format your answer systematically. Have a mental image
on how you would arrange your answer in simple bullet points and then expand on these in a
structured manner.

CORE SURGICAL INTERVIEW GUIDE 101


www.surgicalinterview.co.uk
HELPING YOU THROUGH YOUR CORE SURGICAL INTERVIEW 12th EDITION: 2023

5. Do not rush to answer a question. Take some time to make sure you understand what the
question is asking for and think of how to answer it.
6. Always smile, appear confident and be friendly. Do not be too confident as this may come across
as arrogance and do not be too rigid and monotonous in your speech. It is a formal interview
but the interviewers would like to know they can enjoy working with you for the next few years.
7. Enjoy the interview. It is possible. Surprisingly.

We aim to walk you through the worry and the stress associated with the creation of an effective
portfolio, through to the point where you produce a summary of your work that is clear, coherent and
ultimately impressive.

CORE SURGICAL INTERVIEW GUIDE 102


www.surgicalinterview.co.uk
HELPING YOU THROUGH YOUR CORE SURGICAL INTERVIEW 12th EDITION: 2023

2.2 How to Structure Your Portfolio

Presentation

Presentation of your portfolio can be as daunting as the interview process itself, but it is an excellent
way to review your achievements, and find gaps in your CV which you may still have a chance to fill.
It will also get you thinking about questions items in your portfolio may raise and you can start to
formulate answers to these. The majority of candidates use a simple A4 ring binder with file dividers.
This allows your work to be clearly categorised and helps the panel navigate your portfolio at will.
Remember that the panel have to look at hundreds of these over a few days. Commonly, the portfolio
interview panel is made up of three interviewers. Two will conduct each interview, whilst the third reads
the portfolio of the incoming student. An impression of good presentation skills and organisation are
important for any trainee and will get your examiners onside from the start. Buy a brand new ring
binder, of a neutral colour, and some high quality printing paper. Use the paper for your CV and the
key pages you want the panel to flick to at the very least. Investing in good quality dividers and file
pockets is definitely worth it, as you will give the impression that you are serious about your career and
that you really want to get in. WHSmith and Rymans both have a range of these. We recommend
buying the standard “glossy” file pockets in which you put an individual sheet (plus photocopies hidden
but accessible behind it if required) and the “open slip in” file pockets which can be used for things you
would like the interviews to take out and read, such as PDFs of your publications, or your CV.

Contents

Your portfolio should start with a clear page of contents. Whether you use individual pages or sections
divided by plastic folders or section dividers, each section should be clearly numbered.

One of our editors used her CV at the front of her portfolio in lieu of a contents page, so that the panel
immediately had an overview of her achievements. She then reprinted the relevant parts of her CV as
a front page for each ‘chapter’ in her portfolio.

Obviously, the order of the contents will depend on what you have achieved throughout your training.
The following is a suggestion of a possible format for your portfolio:

Formal Documentation

• Curriculum Vitae
• Application form
• References
• Attainment of FY1 competences
• Passport copies and passport photographs
• MBBS/MBChB
• BSc certification
• MRCS A/B (if applicable)

CORE SURGICAL INTERVIEW GUIDE 103


www.surgicalinterview.co.uk
HELPING YOU THROUGH YOUR CORE SURGICAL INTERVIEW 12th EDITION: 2023

Prizes and Awards

• Including essay prizes, audit prizes, year-group prizes, presentation prizes

Employment

• Including jobs, hospitals, consultants

Clinical Skills

• Surgical Logbook --> sign up to ISCP now. It shows that you are aware of what is used in core surgical
training (and beyond) and that you are already engaged with the process.
• Basic Surgical Skills
• CCrISP course
• ATLS
• ALS
• Other courses attended (as applicable)

ePortfolio

• Summary of e-portfolio
• FY1 & FY2 DOPS Summary
• FY1 & FY2 Case Based Discussion Summary
• FY1 & FY2 Mini CEX
• FY1 & FY2 Mini PAT Summary (Choose a few of your best work based assessments,; the scores were
actually looked at and noted during interviews)
• Clinical Encounters – Reflective practice
• Evidence of Attendance at surgical teaching sessions 202

Publications

• Summary of your publications including your role in the work


• Colour PDFs in booklet form that is easily accessible (best work first)

Presentations

• Summary of your presentations including your role in the work


• Colour powerpoint presentation (6 slides per page) or PDF of poster (best work first)

Audit

CORE SURGICAL INTERVIEW GUIDE 104


www.surgicalinterview.co.uk
HELPING YOU THROUGH YOUR CORE SURGICAL INTERVIEW 12th EDITION: 2023

• Summary of your audits including your role in the work


• Colour powerpoint presentation (6 slides per page) or summary sheet of each audit (again best work first)

Teaching

• Evidence of teaching courses attended (e.g.: Training the Trainers)


• Evidence of teaching sessions given
• Example paper based or electronic feedback (as applicable)

Management and Leadership

• Examples: Foundation School (FY1/FY2) representative


• Mess President
• Involvement in the British Medical Association Junior Doctors Committee or other professional organisation
• Involvement in rota design

Extracurricular

• Sporting achievements
• Charitable pursuits
• Musical interests
• General lifestyle interests (if can be applied to competence in surgical career)

Formal documentation will include your General Medical Council (full and provisional) registration
document (with 2 copies) and medical school certification (plus any other degree certificates that you
may possess) (each with 2 copies).

You will also need to provide 2 original forms of photo ID and one original documentation providing
proof of address OR one original form of photo ID and 2 original documents providing proof of address.
This is in addition to 2 passport photographs with names printed clearly on the back. The attainment
of FY1 competences will come in the form of a completed 5.1 form. This should be signed off by your
foundation school at the end of your FY1 year.

If you have completed your Royal College Membership exams, you should include the original
certificate and 2 copies at this stage. It is a good idea to have two photocopies of each certificate in your
portfolio in case they ask for it as evidence.

Overseas graduates will need to include evidence of their undergraduate training in English, as well as
an original (plus two copies) of the English Language Testing Certificate. This is in additional to a
current Immigration Status page (including visa stamp) and supporting documentation from the Home
Office (where applicable).

*In cases where original documentation is outstanding, you will be required to supply originals within
48 hours (excluding weekends and bank holidays) following your interview between the hours of 9am

CORE SURGICAL INTERVIEW GUIDE 105


www.surgicalinterview.co.uk
HELPING YOU THROUGH YOUR CORE SURGICAL INTERVIEW 12th EDITION: 2023

and 5pm. Failure to provide the missing documentation within 48 hours (excluding weekends and bank
holidays) and in person would result in your application being withdrawn.*

We have included a more detailed section on all the important categories and what you might be asked
about below.

CORE SURGICAL INTERVIEW GUIDE 106


www.surgicalinterview.co.uk
HELPING YOU THROUGH YOUR CORE SURGICAL INTERVIEW 12th EDITION: 2023

2.3 Curriculum Vitae


You should include at least three copies of your CV for all members of the interview panel, and make
it easy for them to remove them from your portfolio.

Your CV should include a brief pre-university educational history and then your university
qualifications. If you received any prizes at university for finishing in the top 5% or 10% on a particular
firm, or end of year prizes, include these here, along with the percentage of your year group that attained
the particular distinction.

If you graduated from medical school with a Merit, Honours or Distinction, this should be obvious.
This is not to say a double size font with an underline, but it should be clear to the reader that you are
not a run-of-the-mill candidate by placing it in a prominent position.

If you have completed a BSc, BA or BMedSci, include this here, along with your degree classification.

Any further degrees prior to entry to medical school should be clearly mentioned. In the case of further
work on PHD or Masters qualifications, you may want to deal with this separately, especially if your
work was substantive and would have a direct impact on your suitability for surgery. Make sure that
you link your education experiences back to your aptitude, as well as interest in a surgical career.

At FY2 level, your employment history is likely to be fairly generic. You should give your job
information starting from your most recent post and working backwards to the start of FY1. If this is
not your first time applying for CT1, you may have completed an FTSTA job or another lateral non-
training surgical post. Instead of trying to hide this amongst the mass of other things in your CV, use
this as a strength! You will have experience that other trainees will not have had and you should take
advantage of this. At your eventual interview, you should emphasise this additional surgical experience
is in itself a proof of commitment to specialty.

Be careful to clarify the relevance of all the experiences mentioned in your CV to a surgical career and
do not downplay your achievements. Keywords like ‘organised’, ‘led’, ‘designed’ and ‘proposed’ are
almost universally preferable to ‘participated’. Be careful however, to show that you are a team player
through your featured experiences. Many surgical candidates are overly keen to always appear to be
the alpha-male or female. This ignores the fact that the vast majority of your career will be spent working
as part of a team. Use this to your benefit when mentioning achievements in sports or music. Emphasis
the fact that you have experience of working with others in a high pressured environment towards a
common goal and show how this will make you a better trainee.

Application Form

It is advised that you include a copy of your application form in a section of your portfolio. Make sure
you know everything you have included in your application intimately. For instance, they often will ask
about a particular audit and then for your actual results. Potential questions such as, “describe your best
audit” which was asked last year, are discussed in a separate section below

CORE SURGICAL INTERVIEW GUIDE 107


www.surgicalinterview.co.uk
HELPING YOU THROUGH YOUR CORE SURGICAL INTERVIEW 12th EDITION: 2023

Courses

Be sure to include verification of any courses that you have mentioned in your application form. This
includes the main surgical courses, such as ATLS, BSS and CCRISP, but also ALS, communication
skills courses, teaching skills and other courses towards specialist training. Any courses, meetings or
conferences that you have attended however, can be used to show additional commitment to specialty.
Make sure you include two photocopies of the course certificate in each file pocket so that they can take
them if necessary.

Be prepared to answer the question “what did you learn from this course?” It has been made clear by
the London Deanery that courses and conferences shouldn’t just be attended to put on your CV, instead
you should always think “what am I bringing back from this course to my hospital/every day work?”

Clinical Skills

Don’t worry if it you do not have hundreds of operations in there, but you should include a logbook, as
logging the cases you do is an important part of being a surgical trainee and they will expect to see it.
eLogbook lets you print out a summary sheet of operations performed which you can include at the
front of your logbook, with the individual operations printed after. At this early stage in your career for
the majority of cases you will have assisted (A) rather than performed. Supervised trainer scrubbed
(STS), means that you performed part of the operation with your consultant or registrar scrubbed to
assist you, supervised trainer unscrubbed (STU) means you were the senior surgeon in the operation
and did not need any assistance, but that it would have been available if you had needed it. Performed
(P) is when you are capable, on your own, of performing the operation and of dealing with potential
complications. Your consultant therefore does not need to be available to assist you. This is where you
are aiming when you are a registrar rather than an SHO, so be wary of saying you performed an
operation as the interview panel may well question this, and ask you about critical steps and how you
would deal with complications.

Don’t despair if your logbook is not detailed. The panel are looking for any other evidence that you
have an interest in pursuing a surgical career, not proof that you have vast surgical experience. If you
happen to have had good surgical experience in the past, then highlight this in your portfolio, but the
role of core surgical training and beyond is to deliver quality generic surgical skills.

Prizes and Awards

If you have mentioned prizes or awards in your portfolio, you should include relevant documentation,
with an extracurricular prize or award, you may be asked the relevance of your award to your future
surgical practice and you should think of some ways in which your award will make you a better team
member, better leader or more effective teacher for example. You may be asked what your greatest
achievement is. If so, then make sure you refer to something impressive in your portfolio if possible,
and make it clear why you are proud of this achievement. It could be a prize, some teaching you did,
a presentation, or something outside of medicine. The marks will focus on why this is your greatest
achievement, so it is important to reflect on the effort you put in, what you learnt from it, and how it is
relevant to your future career in surgery.

CORE SURGICAL INTERVIEW GUIDE 108


www.surgicalinterview.co.uk
HELPING YOU THROUGH YOUR CORE SURGICAL INTERVIEW 12th EDITION: 2023

2.4 ePortfolio

ePortfolio

There are different opinions on the best way to display your eportfolio. I personally opened this section
with the table of contents printed off from the eportfolio website. This allows the interviewers to navigate
the eportfolio section at a glance. You will have a number of case based discussions (CBDs) and mini-
CEXs which could be displayed. You will not have the space to display them all, and the panel will
equally not have the time to read them individually. I got around this by displaying groups and
assessments (e.g.: FY1 CBDs, FY2 CBDs, FY1 mini CEXs and so on) together. If you are using plastic
file dividers, remember that they may not take all the sheets of paper out of the file so it is a useful tip to
front each section with a good assessment, with plenty of qualitative feedback so this can be easily read
by the examiner. I prominently displayed my mini PAT feedback forms and evidence of reflective
practice. Such pieces are more likely to be noticed by the panel as words will always tend to tell more
about a candidate’s manner and clinical knowledge that quantitative data from a CBD, especially if the
assessor has not given any written feedback.

Some trainees have included ‘thank you letters’ or cards from patient’s or patient’s relatives. Whilst this
may not seem altogether important, a satisfied patient is a useful metric against which your clinical
verbal and non-verbal communication can be measured.

Last year people were asked:

Do you think CBD and CEX assessments are useful?

To answer this question well, once again structure is important. One suggestion is to explain why work
based assessments (as they are referred to in CST) are useful to i) the individual, ii) the trainer and iii)
the Deanery. We would encourage you to think around management questions such as this and begin
to develop your own structures to hang your answers on to.

Whatever you truly think about these assessments, it is important to realise their purpose. The deanery
is responsible for training surgeons and has to have a way of monitoring this. One of the most common
methods used through out the UK is a Case Based Discussion (CBD) and Clinical Examination (CEX).
Done properly these are excellent educational tools that allow you to have one on one time with a senior
who knows a lot more about the clinical condition of the patient you are seeing than you do, and receive
detailed feedback. The score reflects how well you are doing, and therefore gives the deanery a method
of assessing you without having to set a formal exam and most importantly, shows that you are
improving and progressing. They show that you are continuing your medical education, and engaging
with the syllabus, and also that you are generally well organised, and able to communicate well enough
to get them done.

The interviewers know that many doctors think the assessments are imperfect, so be honest but
professional in your criticisms. For instance, whilst being an excellent tool when done properly, often
there is insufficient time to complete them fully with a consultant. The marks given are not
representative of how well you have done, but rather, how generous a marker the consultant is, or how

CORE SURGICAL INTERVIEW GUIDE 109


www.surgicalinterview.co.uk
HELPING YOU THROUGH YOUR CORE SURGICAL INTERVIEW 12th EDITION: 2023

well you get along with them. The interviewers are looking for an honest balanced answer which
demonstrates that you take responsibility for your own medical education.

It is important to note that different deaneries across the country ask their trainees to complete different
numbers of these throughout the year; look up how many are required by your first choice deanery.
They are generally embraced by CST deaneries; the London Deanery (for example) expects at least 80
work based assessments per year, with 50% completed by consultants.

This information would suggest to a savvy candidate that they should err on the side of approval of this
method of assessment/monitoring of education and progression. One way of giving a fully developed
answer is to say why they are good, what a criticism of them might be, and how you intend to overcome that
problem whilst you are in your CST years.

A lesson for life - never just go to someone with problems, always bring possible solutions with you.

CORE SURGICAL INTERVIEW GUIDE 110


www.surgicalinterview.co.uk
HELPING YOU THROUGH YOUR CORE SURGICAL INTERVIEW 12th EDITION: 2023

2.5 2-minute leadership biography

’Please present a 2 minute leadership biography, based on skills you have acquired as a
leader and how you will be able to apply these to core training.’’

Throughout my life I have acquired and exemplified good leadership qualities through the positions I
have held. At school I was appointed as Head Girl, as an undergraduate I captained the University
Hockey Team and lead a charity raising expedition up Mount Kilimanjaro, whilst at a postgraduate
level I was appointed Lead Clinical Teaching Fellow for Homerton Trust.

In the latter, I was extremely successful as a team leader. I took initiative appropriately and identified
changes needed to be made to the teaching programme. I discussed and communicated these changes
effectively between Consultants, fellows and students, negotiating between differences of opinions
from various groups. From a personal point, I was consistently well organised throughout and was
described by peers as reliable, flexible and very approachable. Importantly I was extremely
enthusiastic and motivating, exampled by encouraging other fellows to take on greater roles within the
various projects and future roles the following year. I know my own limits and I sought and listened to
my colleagues opinions when outside of these limits. The excellent verbal and written feedback
received from consultants, students and teaching colleagues alike confirms this.

Specifically, whilst leading the climb up Mount Kilimajnaro I had to be positive and motivating at all
times. I had to recognise and utilise the best qualities of each of the individual members to ensure a
successful expedition.

I will apply similar qualities to my core surgical training. This could be in the operating theatre where
good communication between the team is paramount, to on the ward where I will be a reliable senior
to the junior colleagues, listening and acting on all their concerns and encouraging them to take on
greater responsibilities. I will be extremely motivated in my work, identify areas of training to be
improved and showing initiative in tackling these problems.

I believe I have acquired all the appropriate skills to be an excellent leader.

Advice

This new station allows you to show off your greatest achievements from your portfolio. It will be
followed by 3 minutes of questioning. You might be asked for evidence of leadership skills you haven’t
mentioned or to clarify the evidence for a particular skill. Leaders tend to be articulate and confident
in themselves, so make sure you practice several times beforehand. Get as much feedback as possible,
both on the content and on your delivery. To make it sound as natural as possible split the 2 minutes
into three general points plus a concluding remark. Commit the message you are trying to convey in
each section to memory, rather than the words themselves to avoid looking too rehearsed. We will
practice this station extensively during the course.

CORE SURGICAL INTERVIEW GUIDE 111


www.surgicalinterview.co.uk
HELPING YOU THROUGH YOUR CORE SURGICAL INTERVIEW 12th EDITION: 2023

2.6 Publications, presentations, and audit

Publications

In your application form you were asked about publications that are searchable on the PUBMED
database. It is a good idea to include a summary page with each of your publications listed and briefly
summaries in approximately one paragraph. State the publication, the aims, main results and
conclusions and your role in the work. Often putting your name in bold within the authors helps the
person reading your portfolio/CV. Include any work that is submitted or in progress that you may not
have put in your application form. You should then include a colour PDF print out of your published
work in your portfolio, ideally in a slip out file pocket so that it is easily accessible to your interviewers.
Put the publication you are most proud of earlier in your portfolio and describe it in more detail in the
summary page to increase the chances that you will be asked about it. If the panel have a chance to see
the piece of work, they may be more liable to spend time discussing it. Make sure that you know the
methods you used and the results, and be prepared to discuss them.

Presentations

Again here, we would recommend that you include a summary sheet summarising all the presentations
that you have done briefly and highlight the role that you played. It serves to draw the interviewers eye
where you would like it to go, and also is an excellent way of reminding yourself of presentations that
you did years ago. Include the actual presentation if you can, normally as a printed colour handout,
with 6 slides per sheet. Powerpoint presentations are normally colourful and therefore make your
portfolio look more aesthetically pleasing. Like above, know your presentations inside out.

Audit

A summary sheet here, with a paragraph on each audit is


very helpful as it allows you to highlight the role you played,
which is what they will be interested on finding out.

Include a powerpoint or one page summary of each audit


including what you did, the clinical problem, the standard
used, how data was collected and analysed, with relevant
results. Mention any change you made, and results of any
re-audit. If possible include the data collection sheet and
any implementation such as a new pro forma that you came
up with.

Using the words in the diagram to describe the stages you were involved with can help clarify your role,
and

CORE SURGICAL INTERVIEW GUIDE 112


www.surgicalinterview.co.uk
HELPING YOU THROUGH YOUR CORE SURGICAL INTERVIEW 12th EDITION: 2023

2.7 Teaching
Teaching

Include any certificates of teaching courses that you have. You should include paper or electronic-based
feedback on your teaching. If you don’t have any feedback, get it now. Get in contact with medical students
you have taught on the ward or given lectures to and ask them to fill out a simple feedback form. When
discussing the teaching you have done, be clear about your role. Use 'I' instead of 'we' and if you have
been involved in the inception, rather than just participation in a teaching course, mention this. Be clear
about the difference between a local and regional teaching programme. It is interesting to note that
regional teaching is defined as teaching students from more than one hospital, and a teaching
programme is defined as more than one lecture or session with the same students. Use these definitions
to your advantage; write a formal teaching schedule rather than leaving valuable bedside teaching as
informal/ad hoc. Assessors will be looking for evidence that your teaching had an lasting impact, both
on those receiving your tuition and beyond. If you have produced a manual, perhaps a guide to your
old firm, or other teaching materials, this is evidence of your possession of more developed teaching
methods and should be included in your portfolio.

Your answer is further strengthened if you possess a formal teaching qualification and there are a
number of available courses for doctors looking to gain teaching qualifications. The most well known of
these is 'Training the trainers', which looks to teach novel teaching methods to trainees. There are
alternative options based at local hospitals and regional centres, or online. Completing formal training
gives evidence of your interest in teaching and should improve the effectiveness of your clinical
communication skills in general. Whatever teaching you have done, make sure you have either a
certificate or written feedback as the interviewer will expect to see it in your portfolio. The same is true
for anything you have put on your CV or application form. Evidence is essential, so if you have
mentioned achieving Grade 8 in the piano and have mentioned that it has helped developed your
manual skills and dexterity, make sure the documentation is included.

In past years candidates were asked:

What is your experience of teaching?

One candidate’s answer:

"I have always enjoyed teaching. I was professionally employed as an A level tutor during my medical
student years, which allowed me to develop important skills that I have continued to use throughout
foundation training, and have developed them further by taking part in the introduction to teaching
course. I helped to set up a 4 month long teaching course for medical students during F1, and led a
mock OSCE course within the trust. In addition, I regularly teach medical student informally on the
wards and have received excellent feedback to date"

Other Information

CORE SURGICAL INTERVIEW GUIDE 113


www.surgicalinterview.co.uk
HELPING YOU THROUGH YOUR CORE SURGICAL INTERVIEW 12th EDITION: 2023

Your extracurricular pursuits are valid and should form part of your portfolio. They show that you are
a rounded individual and that you recognise the importance of work life balance. Remember that this
all forms part of an interview process and that your portfolio will be read by the panel. If you have
performed at a high sporting level or achieved an impressive distinction in music or the arts, include
evidence of this. This could be a certificate or a programme from your last performance. Remember
that your portfolio should look professional and that the information that you are presenting will form
part of an overall impression of what you will be like as a surgical trainee and ultimately as a potential
future colleague.

Allocate sufficient time to getting your portfolio together, it takes a lot longer than you think. Its probably
best to start by searching for all the certificates, posters and power point presentations that you wish to
include, and seeking out the inevitable missing ones. Next, put the different sections together as we have
suggested, using an old file, but one that will not cause any damage to your documents. Once a section
is complete, order it with the most impressive first. At this stage you can draft a summary sheet for the
sections like publications, and a sub contents page for the others sections. Once your portfolio is starting
to look near completion, then take a trip to WHSmiths or equivalent after calculating what you need
and buy the high quality stuff. This portfolio will soon become your most prized possession, sad but
true, more valuable than your laptop, girl/boyfriend combined. You do not want to lose it. So, keep it
somewhere safe, where someone else isn’t going to pick up and drop spill tea on etc and try to keep it in
perfect condition until the big day. Photocopy everything and keep the copies in a separate place -
believe us, once you’ve spent hours tracking everything down, you’ll never want to have to do it again!

CORE SURGICAL INTERVIEW GUIDE 114


www.surgicalinterview.co.uk
HELPING YOU THROUGH YOUR CORE SURGICAL INTERVIEW 12th EDITION: 2023

2.8 Commitment to Speciality


Why Surgery?

Surgical training is hard, but rewarding. It commonly will last up 8 years or more and this part of the
interview is designed to ensure that you have fully thought about your choice of a surgical career. All
applicants will be keen to convince the panel of their dedication, but what can you do to set yourself
apart from other candidates? Most of the evidence for this will come from your portfolio and it is
essential that you have included all the experiences which could show your dedication.

When answering this question it is important to both explain why you would like to be a surgeon and
why you are the ideal candidate for surgery:

Why do you want to be a surgeon?

The interviewers are looking for a well structured answer which shows dedication to speciality and
motivation for core training. It is important to give a range of reasons, with a personal example, of why
you enjoy it and why you are suited to surgery. Your answer should be enthusiastic but focused and
unique to you.

Perhaps you enjoy using your hands to make an immediate difference to peoples’ disease, or enjoy
making decisions under pressure. Surgery is fast moving when looking after acutely unwell patients in
an emergency setting, which appeals to many people. It is also allows you to look after chronic disease,
where a personal relationship is built up with your patients.

Whatever the reason you have chosen surgery, it is important to use a personal example, and if possible
a reflection/achievement related to this example. For instance here is John's answer from last year.

"I enjoy the challenge of using the practical skills I have developed, whilst I enjoy watching my
consultant carrying out larger cases, I really love performing skin biopsies myself. I can feel myself
becoming more dextrous each time, and improving my technique in small ways. Whilst this is a simple
procedure, it has confirmed my love of surgery. I enjoy seeing a patient beforehand, and explaining the
operation and potential risks before carrying out the biopsy and reviewing them afterwards. Both
patients and my consultant have commented positively on my clinical manner and on how quickly I am
improving technically. I am looking forward to doing this for more complicated procedures later in my
career."

His answer is a good one as the example is generally applicable to core surgical training. It is specific
and personal and is appropriate to his current level, an F2. He also manages to mention a small
achievement, a consultant's interviewing him would like to have as their trainee. He comes across as
enthusiastic and knowledgeable about the need to care for patients before, during and after surgery.

He makes a strong statement at the end of his answer; “I am looking forward to doing this for more
complicated procedures later in my career." It gives a good impression to make confident statements
such as this. Don’t say “if I get into core surgical training’, say “when I get into core surgical training.”

CORE SURGICAL INTERVIEW GUIDE 115


www.surgicalinterview.co.uk
HELPING YOU THROUGH YOUR CORE SURGICAL INTERVIEW 12th EDITION: 2023

The rest of his answer can follow a similar format:

• Reason why you want to be a surgeon


• A personal example
• Your reflection on what you / why you like it
• +/- an Achievement which positively reflects on you as a potential surgical trainee

It is a good idea to mention a variety of reasons why you want to be a surgeon. These can be research
opportunities, the fact it is challenging, the interaction with other specialties, being able to interpret
images and then do something about the surgical problem, and using advanced technology to name just
a few.

You can end by mentioning the particular subspecialty you are interested in going into, "I hope to
develop these skills and then apply for ENT, a specialty that combines my passion for ...."

Where do you see yourself in 10 years time?

“I would certainly hope to have completed my clinical training in 10 years time and have become a
consultant. My hope is to become a vascular surgeon. I worked on a vascular surgery firm in my first
foundation year and made a presentation to a national vascular conference during this time. I enjoy the
technical nature of the work, and enjoyed the variety in the types of operations performed, from the
more lengthy and complicated bypass procedures to the minimally invasive venous procedures, where
advances are constantly being made.

I have enjoyed working in large teaching hospitals so far in my career, though I look forward to having
experiences of District General Hospitals during my training. I have an interest in teaching and have
organised 2 surgical teaching courses for 3rd year students at Warwick medical school. I feel that
working as a consultant at a teaching hospital would give me more access to current teaching materials
and allow me to develop this interest further.

I have a strong interest in research. This was first developed during my BSc, where I managed to
complete a project on plasmodium falciparum, which I followed through to publication. I published the
results of a surgical audit that I completed in June on surgical complications in orthopaedic surgery and
I would certainly hope to have a breadth of research experience in 10 years time. I am interested in
pursuing a PhD over the course of my specialty training. I have discussed this with a number of my
seniors who have completed their own research and I realise that it can increases one's understanding
of a subject and improve your clinical practice. I have considered the fact that this would lengthen my
training, but still think that it would be worthwhile. However, I do not think that I would want to end
up in a purely academic role as I would miss daily patient interaction.

I have played rugby at a high level throughout my school and university years and continue to play for
a local side. I am under no illusion that surgical training can be arduous with long and sometimes
irregular hours. However, I believe that having some work-life balance is essential. I would do my
utmost to stay involved with my rugby team on an occasional basis and as much as my work

CORE SURGICAL INTERVIEW GUIDE 116


www.surgicalinterview.co.uk
HELPING YOU THROUGH YOUR CORE SURGICAL INTERVIEW 12th EDITION: 2023

commitments would allow and would hope that I would still be running out on the odd Saturday if my
knees would allow it!

I was involved in an expedition to Nepal giving medical aid to deprived and remote areas on my medical
elective and am currently involved in raising funds for a charity sending medical supplies to deprived
communities in South Africa. As I become more senior, I would hope that my effectiveness in such
charitable pursuits would also increase.”

What approach should be taken in answering these questions?

The difficulty of both of these questions are in their simplicity. They were among the most commonly
asked questions in recent interviews; therefore write an answer for both and learn before your interview.
The easy solution here would be to give a generic answer of for example 'I would like to be a consultant
in x specialty working in Manchester', or ‘I want to be a surgeon as I enjoy practical things’. This would
be score a poor mark, despite answering the question posed.

With this sort of question, it is useful to think of how to structure your answer. Such a wide ranging
question merits a wide ranging answer, but you must be careful that you maintain concentration on
structure.

A common structure would be to divide your answer into clinical, academic and personal reasons.
Candidates are often reluctant to discuss personal aspirations in an interview, but of course these
considerations will have a bearing on your progression as a clinician and should be considered, at least
briefly.

Clinical

You must show that you are aware that a core training scheme is not a specialty programme, however,
at this point, it would be appropriate to discuss your specialty aspirations, if you have one. This could
be backed up by evidence from your portfolio showing that you had given some thought to building a
career focussed in that particular direction. This could be in the form of papers, presentations or taster
weeks for example.

Are there any specialties that you would like to develop? Many specialties are so sub-specialised that
you may want to be an orthopaedic surgeon, but want to have an interest in arthroscopic procedures
involving the knee for example. This sort of detail shows that you have given some though to your
potential career path. If you do go into such details, be sure that you can give adequate reasoning behind
your interest if questioned more thoroughly.

Clearly the eventual preferred destination for most clinicians is a consultant post. Would you like to
work in a large, cosmopolitan environment or a more rural destination, a DGH or teaching hospital.
Whilst you may not have thought this far ahead and your decision may change in future, such
considerations show a maturity of thinking that will set you apart from your peers. Again, make sure
you can explain this answer. For example, you may want to work in a large teaching hospital because
of your experience in teaching. If you can show evidence of a teaching course you have attended or a
course you have set up for medical students in your portfolio, this will strengthen your answer.

CORE SURGICAL INTERVIEW GUIDE 117


www.surgicalinterview.co.uk
HELPING YOU THROUGH YOUR CORE SURGICAL INTERVIEW 12th EDITION: 2023

Academic

Are you interested in research? Would you be wanting to pursue an MD or PhD in future? If so, discuss
how you think this will impact on your career and chosen specialty. Are you currently undertaking an
MSC or other further qualification? What have been your experiences of this so far and how do these
experiences affect your plans for future research involvement?

Do you have an interest in teaching? How do you plan to develop this?

Personal

Would you like to be part of a large or a smaller team? You could discuss your experiences of
participating in different types of team at this stage to show your understanding of the multidisciplinary
team, but also of good communication with seniors and other colleagues.

You could also discuss your extracurricular pursuits. If you are a keen sportsman/woman, would you
hope to be playing rugby, lacrosse or netball in 10 years time. How do you feel this would impact on
your surgical career and are there any changes that you will have to make in the intervening period to
ensure a good work life balance?

Would you hope to be participating in any expeditions abroad or volunteer work? Surgery gives a lot
of opportunities abroad and if you would be keen to take advantages of these, you should make this
clear. You could again link this to past experiences on student elective or current volunteer projects.
This is an opportunity to show that you are a well qualified, but also a well rounded individual.

You also need to show good commitment to a surgical career to date. This will include your attendance
at surgical courses, such as the Basic Surgical Skills course, Advanced Trauma Life Support (ATLS),
STEP Core, STEP Foundation courses and START surgery courses. Of these, BSS and ATLS are the
most favoured by the interview panel as they are compulsory during core training, and show that you
are ahead of the game already. Course show an interest in specialty and can differentiate you from less
driven candidates.

There are a number of associations for junior surgeons, which can be joined by trainees at foundation
level. These include the Association of Surgeons in Training (ASiT). This was founded in 1976 as a
forum for surgical trainees to discuss training matters and now has over 2,700 members. It arranges
several courses, prizes and awards for surgical trainees throughout the year, as well as an annual two
day conference including oral and poster presentations of delegates work. Membership of such
associations goes towards highlighting an interest.

Many hospitals will hold surgical teaching sessions during generic foundation year training. If you can
include documented evidence of attendance at such non compulsory sessions, this can show a dedication
to higher learning and educational achievement, both of the utmost importance for surgical trainees.
Taster weeks are another useful way for foundation year doctors to highlight their surgical interest. They
can often be negotiated after discussion with your educational supervisor and involve you joining
another specialty of your choice for a determined period. This will be of most benefit if you request a
documentation from the surgical consultant verifying your voluntary attendance for surgical cases,

CORE SURGICAL INTERVIEW GUIDE 118


www.surgicalinterview.co.uk
HELPING YOU THROUGH YOUR CORE SURGICAL INTERVIEW 12th EDITION: 2023

clinics and MDT meetings. It will not only show commitment to specialty, but also organisation and
good communication skills as on occasion, such intra-firm transfers can involve delicate negotiation with
senior members of both teams.

Further example questions from previous years:

What are your plans for improvement over the year ahead?

I hope to develop myself further in several key themes.

Firstly, I wish to develop my clinical skills further - not only in theatre, but also in assessment of the
acutely unwell surgical patient. I will attend relevant courses, such as CCrISP, and use my free time to
attend theatre lists in order to become more familiar with the surgical environment. In particular, I
hope to attend a course on laparoscopic surgery as I feel that it is important to get to grips with this
technique early on, given how many operations may now be performed laparoscopically. I hope to
become a proficient surgical assistant, confident with the basic principles of surgery, so that I may be
invited to take on certain procedures mostly independently; for example, aiming to perform a
laparoscopic appendicectomy under supervision by the end of the year.

Academically, I wish to enhance my surgical knowledge by completing my MRCS exam. I will make
time to study for this and take the most of the learning opportunities afforded to me at work, by both
asking my seniors if they might test my knowledge and also asking for feedback on my practical skills in
theatre.

I wish to become more involved in teaching of juniors - having experienced some excellent teaching as
a London trainee so far, I understand its importance. I have developed a poster that is being used in
A&E for teaching purposes, and I hope to organise supporting sessions with foundation trainees in order
to develop myself as a clinical teacher.

On a personal level, I hope to become more confident in my role as a trainee, taking on more decisions
independently and being someone that the juniors feel they can come to for advice and support. I am
looking to enrol on a leadership course to develop my skills further. Outside of medicine, I plan on
travelling to South America this summer, where I hope to spend time trekking. I am excited to learn
more about another culture, which will generate an understanding that can only be beneficial in my
career as a surgeon.

Advice:

Look at your portfolio - where are the gaps? How can you fill them? Do a little research into relevant
courses that might help you. Think about how you feel in the surgical environment. Are you perhaps
lacking a little confidence in theatre? Do you find it tricky when a patient becomes acutely unwell? If
so, mention that you are keen to spend more time in theatre, or that you are keen to attend the CCrISP
course to develop your skills in looking after very sick surgical patients. Tailor your answer to your
preferred surgical speciality if you have one, but don't forget that basics are important too. Use a
structured approach. Talk confidently and enthusiastically - the aim here is not to highlight your

CORE SURGICAL INTERVIEW GUIDE 119


www.surgicalinterview.co.uk
HELPING YOU THROUGH YOUR CORE SURGICAL INTERVIEW 12th EDITION: 2023

weaknesses, but to show that you are driven, ambitious and have actually thought ahead on how to
better yourself.

Why do you think it is important to keep a portfolio?

I feel there are several reasons for keeping a portfolio. Firstly, it provides hard evidence of what I have
achieved so far in my career. I can not only talk about things I have done - but show you certificates I
was awarded for doing them. Not only can I talk about my skills in theatre and my communication skills
- but I can show you letters of support from consultants, and letters of praise from patients. In essence,
it is the proof to support and illustrate that what I say I have achieved it true.

However, I have also found that keeping a portfolio is important in another way - it enables me to look
back and highlight gaps in my progress. For example, I look might look at my presentation section and
feel pleased with how many I have done, but looking closely I can see that one of my national
presentations was related to sexual health medicine, so I might then endeavour to present a more
surgically-orientated project nationally.

In summary, I feel that my portfolio helps me to identify not only what I have achieved so far, but what
I have yet to achieve in the future.

Advice

The best answers discuss more than one perspective - so the obvious answer here is that a portfolio is
something used at interview to prove your worth. But the trick here is in identifying another reason why
it might be useful. Again, draw on specifics from the portfolio (use it as an opportunity to flash a
project/national presentation) and talk enthusiastically.

Take us through your CV

Tip: it may be helpful to break this down into sections in your mind so that you don’t
miss out important areas and to ensure a well rounded answer.

Clinical

Both as undergraduate and postgraduate, I have been highly committed to a surgical career reflected
by my choice of surgical rotations. I have been able to gain exposure by enthusiastically seeking
opportunities available to me in order to strengthen my surgical experience.

By prioritising tasks I have managed to attend theatre regularly thereby assisting, observing and
performing in over 100 validated operations. My seniors have valued my eagerness to participate in
theatre and this has been facilitated by my attendance to the Basic Surgical Skills Course. As a result,
they have been happy to delegate more responsibility to me and have supervised me performing a wide
local excision of vascular lesion, excision of fibroadenomas, dynamic hip screws as well
incision/drainage procedures.

CORE SURGICAL INTERVIEW GUIDE 120


www.surgicalinterview.co.uk
HELPING YOU THROUGH YOUR CORE SURGICAL INTERVIEW 12th EDITION: 2023

Additionally, I value my on call commitments as they have allowed me to build my confidence and
competence in dealing with acutely ill surgical patients. This is reflected in my CBDs and Mini-CEXs
where my seniors have praised my communication, clinical decision-making and ability to recognise
acutely ill patients as well as requesting senior input appropriately.

My regular attendance to multidisciplinary team meetings has allowed me to gain understanding and
develop a holistic approach to the patient care. I have kept this awareness in mind whilst participating
and reviewing patients in the outpatient clinics.

Academic

As an undergraduate, I have researched extensively in the field of oesophageal cancer which has resulted
in a local presentation and BMedSci degree. Additionally, as a foundation trainee I have led and
organised a local surgical syllabus providing one-to-one teaching to 5th year medical students and more
recently I have delivered regional teaching at St. Georges Hospital. This has been facilitated by my
attendance to several courses including clinical presentation skills, teach the teacher course and
management and leadership.

Management

I have also been responsible for completing the WHO checklist during theatre lists and this has increased
my awareness of surgical safety and has motivated me to lead and organise regular theatre list meetings
to ensure an effective patient care pathway.

Understanding and appreciating the roles of my team at large have allowed me to develop as a holistic
and conscientious practitioner. Furthermore, in my previous rotation I took the initiative to update and
edit the trainees guide to ensure smooth handover as well as inspire a high quality of care amongst
future trainees.

Personal

I have been fortunate to have worked under some great mentors and human beings, by observing and
reflecting on their good practices I have tried emulating such attributes into my own development. I
believe my aspirations to pursue my surgical career will be strengthened by successful completion of
MRCS Part A and the ATLS course.

You appear to have a vast number of cases in you log book but have only performed a
few, why is this?

I believe the nature of the foundation programme is to ensure exposure to all fields of medicine and
surgery in order to build a solid foundation for junior doctors as well as acknowledge their own
limitations. However, my regular attendance to theatre shows commitment to the specialty as this has
been difficult at times given ward based commitments. Therefore, to be able to perform just several
procedures under consultant supervision during my foundation training is a great achievement for me.

What defines a leader?

CORE SURGICAL INTERVIEW GUIDE 121


www.surgicalinterview.co.uk
HELPING YOU THROUGH YOUR CORE SURGICAL INTERVIEW 12th EDITION: 2023

A leader is an individual who sets a vision, questions current practices and has the courage to make a
change. They then provide an environment whereby individuals are able to develop. By means of
organisation, planning and appropriate delegation of tasks the vision becomes more apparent and goals
can be achieved.

Advice

This question may appear difficult at first, however, breaking it down into more manageable sections
can help you to identify how much you have actually achieved, and any areas which need more thought.
It is important to reflect on all your experiences and remember you probably have done a lot more than
you think. It is worthwhile making a note of your achievements and gaining evidence for these well in
advance in order to place into your CV. If you think that you are lacking certain areas for the portfolio
station make your other achievements stand out. It is also strongly advised to look at the person
specification for core surgical training whilst you are thinking about your answer, this will give you a
good idea on how you should communicate your achievements.

CORE SURGICAL INTERVIEW GUIDE 122


www.surgicalinterview.co.uk
HELPING YOU THROUGH YOUR CORE SURGICAL INTERVIEW 12th EDITION: 2023

2.9 Research and Audit


What are the differences between research and audit?

“Research creates or tests new knowledge, whilst audit tests clinical practice against established
standards. Research asks ‘What is the right thing to do?’, whilst audit asks ‘Are we doing the right
thing’”. This has been a common question over the years, and it is important to have a clearly thought
out answer.

Both improve the quality of care provided by a healthcare service, involve answering a specific question,
careful sampling, questionnaire design and analysis of findings. However, there are some key differences
between the two. Research is a ‘systematic and rigorous investigation of materials undertaken to
discover/establish facts or relationships and to reach conclusions using scientifically sound methods.

Clinical audit is the ‘comparison of current standards of service provision against either local or national
guidelines, highlighting deviation from best practice and maintaining and improving clinical practice’.
This table sets out difference between the two which you can use to embellish your answer.

Research Audit
Creates new knowledge and determines best practice Answers the question ‘Are we following best
practice?’

Based on a ‘null hypothesis’ Measures against a set, predetermined standard

May involve experimentation or novel treatments Usually does not involve deviation from standard
practice

Extensive statistical analysis The norm is simple statistical analysis

Results can be generalised and are therefore Results are usually relevant to the local setting, but
publishable may be publishable in the interest of a wider
audience

Always requires ethical approval Rarely requires ethical approval

Usually large scale and long duration Usually small scale and short duration

What is an audit?

“An audit systematically compares current clinical practice against an established standard. It consists
of several stages:

• Identification of a clinical problem

CORE SURGICAL INTERVIEW GUIDE 123


www.surgicalinterview.co.uk
HELPING YOU THROUGH YOUR CORE SURGICAL INTERVIEW 12th EDITION: 2023

• Standard Setting

There are multiple definitions of clinical audit. Pick a simple one that you can easily remember. It is
good to show your knowledge of the different stages of the audit cycle.

What is your own experience of clinical audit?

This question will clearly depend on what you have done. If you are reading this in the run up to an
interview and have not completed an audit, it is imperative that you complete one before your interview
date. Audit is indispensable in the modern health care, and most foundation trusts insist that you
complete at least one a year.

In previous years you got marks for the number and complexity of your audit, however the key to top
marks is the role you played. Saying that you “designed, initiated, lead and presented” an audit, will get
you more marks than just “carried out” an audit, so if possible, make sure you use these terms.
Completing the audit cycle and a resultant significant clinical change will score more points.

It is essential to know your audits inside out. Last year candidates were asked what the standard they
used was, and the specific results. Remember, the interviewers will have your portfolio open in front of
them, and therefore will be able to turn to your audit and ask you very specific questions on it. It is easy
to be thrown if you don’t go over all of your audits carefully before the interview.

Describe your best audit

“I have complete 5 audits, in one I noticed that theatre time was being lost due to absent pre operative
investigations, such as ECG and group and save. I consulted with the senior surgeons in the hospital to
define a local standard for time an operation is delayed, designed a computer database to exact
information from the theatre log and analysed and presented it at the trust wide clinical governance
day. I suggested a preoperative checklist that was approved in the entire trust, and used by all surgical
SHOs. A recent re-audit that I carried out demonstrated an improvement from an average of 25 minute
delay to 18 minutes per operation. It is my best audit as it produced a tangible change in practice on a
Trust wide scale.”

You need to emphasise your role, so ideally choose an audit that you have taken a leading role in. If
you can describe involvement in all phases, and discuss a significant impact that your audit had, you
should score well.

CORE SURGICAL INTERVIEW GUIDE 124


www.surgicalinterview.co.uk
HELPING YOU THROUGH YOUR CORE SURGICAL INTERVIEW 12th EDITION: 2023

2.10 Paper Critique


How do you Structure a Paper Critique?

Advice

This question may be asked theoretically. Some deaneries provided the candidate with a short review
or report to read and then asked for a critique. It may be useful to practice this with a colleague or a
senior before your interview, however the trend in surgery has been to move away from this type of
question. Questions change from year to year, so we thought it prudent to include a sample question in
the guide.

When reviewing a paper, I would concentrate on four broad areas. These are:
• Aims
• Methods
• Results
• Discussion

When considering aims, one must consider the central research question. What is the paper looking to
establish and how will this add to the current body of scientific work? The question should be succinct,
relevant and based on a thorough review of current literature.

Methods: Ethical considerations must come first here. Laboratory techniques or patient treatments
deviating from accepted practice must be passed by a local, regional or national ethics committee.

The study design needs close inspection. What design was used and how was the population selected?
Was there any potential bias in patient selection or was it truly random? Was the paper retrospective or
prospective and is this selection valid for the study aim? What was used as a comparative group and is
it truly comparative?

Results: How was the data analysed? What methods were used? What is the significance of the statistical
findings? Does the sample size limit the strength of the conclusions that can be drawn, regardless of the
significance of the results? P values of greater than 0.05 are generally not considered to be significant.
If the authors have claimed significance at a less strict threshold, there must be an extremely strong
statistical reason for this.

Are there are funding sources or conflicts of interest that could hint at a potential bias in the study
conception. The presentation of the data must be considered with graphs correctly labelled and easy
to comprehend.

Discussion: Are the inferences drawn in accordance with the data? Any conclusions stated by the author
need to be substantiated by the study design, sample size and population and statistical analysis. Has
the influence of design flaws and limitations been considered? Who is the author, where was it published
and what is the journal’s impact factor? Could any results be explained by chance or confounding
factors? Your answer should be completed by a summary statement concerning all of the above. This
should be a short paragraph which gives your assessment of the overall strength of the paper.

CORE SURGICAL INTERVIEW GUIDE 125


www.surgicalinterview.co.uk
HELPING YOU THROUGH YOUR CORE SURGICAL INTERVIEW 12th EDITION: 2023

SECTION 3:
Management Station

CORE SURGICAL INTERVIEW GUIDE 126


www.surgicalinterview.co.uk
HELPING YOU THROUGH YOUR CORE SURGICAL INTERVIEW 12th EDITION: 2023

2021

3.1 Management Station


Introduction

The management station is worth 33% of the marks at interview.CT1 will see you take on more
responsibility and whilst you will be still very much in the infancy of your training, you may encounter
more non-clinical challenges that you would have done earlier in your practice; from ethical
considerations, to difficulties with colleagues and involvement in clinical governance.

In this station, you will score highly by again giving confident and structured answers. However, these
must be tempered by knowledge of the law, pillars of medical ethics under which we work and must
show an awareness of the limitations of your capabilities as a junior member of the team and a
willingness to ask for help when necessary.

We include some questions asked in this interview station in the last two years as a guide to what to
expect at your interview panel.

Questions vary geographically and over time, but we


hope to not only give you examples of strong answers to
some of these difficult questions, but also to help you start
to think about experiences that you have had personally
that can be used as examples.

There may be a temptation to exaggerate past


experiences or difficulties to try and say what you want
the examiner to hear and to ‘fit’ with the question. Don't.
Your answer will be much more reflective, coherent and
thoughtful if it is something that you have actually gone
through. Some people struggle to prepare for this station
and some think that it will be the easiest of the three.

It is important to remember that the management station holds a third of your marks, and therefore
does require revision. A good answer requires familiarity with the structure of the NHS, complaints
procedures, and ethical principles which you may not have had a reason to engage with during your
ward work in FY1 and FY2.

Think before you answer, patient safety is an important starting point in answering many management
questions, remember to say you would ask for help from more senior members of the team and to
structure your answer. These tips are the basis of success in this station.

CORE SURGICAL INTERVIEW GUIDE 127


www.surgicalinterview.co.uk
HELPING YOU THROUGH YOUR CORE SURGICAL INTERVIEW 12th EDITION: 2023

3.2 Consultant appears drunk at work


You are a surgical SHO on the ward. Your consultant arrives to start the afternoon ward round. You
can smell alcohol on his breath, he is visibly off balance and he is slurring his words. How do you
proceed?

‘This situation would need to be dealt with away from the middle of the ward. I would need to take the
consultant away from the clinical areas to a side-room to deal with the issue. I would need to find out
more about the issue by asking some questions. I would suggest that he go home and would ask a
supportive registrar or consultant to cover his clinical duties’.

Advice

A classic interview question that is often repeated and is a candidate differentiating station, in that those
who have prepared for something like this are able to score near full marks, whereas, those who haven’t
considered it score poorly. When there are only 3 stations, doing well in each is essential.

This clearly would be a difficult situation which would require careful handling. Your answer must
highlight the fact that your first concern would be for patient safety. You could not be sure that the
decisions made by a consultant in this condition would be in the best interests of the patient and you
would be failing in your duty of care if you allowed him to make patient contact.

As an SHO, even at an early stage you may not be in a position to deal with this yourself so it might be
best to enlist the help of a supportive registrar or consultant. If there is anything you can do to resolve
the situation then it is a good idea to take the initiative and do so, for instance, you could suggest that
he goes home and that he ask the registrar to lead the ward round.

If you are unable to resolve the situation the next step to ensure patient care is to escalate. Given the
fact that there is a potential for patient harm here, you would be justified in enlisting the help of a senior
consultant, or the clinical head of the unit. This may be difficult, given the fact that this person may be
a friend or close colleague of your consultant. If you felt unable to contact another consultant within the
department, you would be justified in contacting the clinical director. In the interview they specifically
ask for this chain of command - registrar - consultant - head of department - clinical director - medical
director. Once you have senior support the management will normally be taken out of your hands, but
they may ask the next steps you would take.

Whilst your first duty of care is to the patient, you also have a duty of care to your consultant. It is
important that he has the support he needs, these include ensuring his work is covered, that he has
people to talk to about the incident and surrounding alcohol issues. The situation would need to be
discussed with your consultant and he would have to be sent home. All patients previously seen by him
would have to be reviewed by another consultant as the decisions made whilst in this condition would
have to be assumed to be unsafe.

You also have a duty of care to the hospital and to yourself in this case. It would be imperative that you
clearly documented all the encounters mentioned above. In this event, i.e. one involving a senior

CORE SURGICAL INTERVIEW GUIDE 128


www.surgicalinterview.co.uk
HELPING YOU THROUGH YOUR CORE SURGICAL INTERVIEW 12th EDITION: 2023

colleague, if you informed a senior colleague or the clinical director, you would have a duty to ensure
that appropriate action had been taken, even as a junior member of staff.

If you still think that the action taken had been insufficient, you would be justified in escalating to the
National Clinical Assessment service and following that, the General Medical Council (GMC), however
you should make it clear that you would ask senior colleagues in the department for advice before taking
such action.

For any question like this a similar approach can be taken:

• Ensure patient safety, always your first priority


• Maintain professionalism by taking the situation away from patients,
• Offer your colleague support
• Find out more and discuss the situation, the initiative and act to resolve it if possible
• If resolution is not possible, then you need to escalate the situation to an appropriate level and
discuss with the MDU.

CORE SURGICAL INTERVIEW GUIDE 129


www.surgicalinterview.co.uk
HELPING YOU THROUGH YOUR CORE SURGICAL INTERVIEW 12th EDITION: 2023

3.3 Conflict Resolution


You are an SHO assisting in an elective minor procedures theatre list, however your consultant would
like you to complete the list as he has an urgent meeting to attend to.

When the consultant leaves, the theatre team are apprehensive about you completing
the list and advise they have been finishing late everyday for the past week and would
like to leave on time. What you do?

This scenario highlights a case of patient safety. As a prospective core surgical trainee I hold this with
high regard and from the outset I would reassure the team of my surgical competence for the procedures
scheduled on the list by showing a validated logbook that has been signed by my consultant.

I would then seek to understand why the theatre team are apprehensive about me completing the
theatre list and would be receptive to their opinions.

There may be several reasons why they may be feeling this way including the fact that they have
recently been finishing late for the past week. In this instance, I would empathetic to their concern as I
understand that they may have families to attend to and plans outside the work place and have every
right to leave within their contracted working hours.

However, at the same time I would also advise that patient safety is paramount and that I am willing to
continue with the theatre list in the event that it overran and would arrange for the on-call theatre staff
to take over if the day team wished to leave on time.

This would help ensure that patient care is not affected and patients’ needs are prioritised. Cancelling
cases on the theatre list may tarnish relationships built on trust with one’s patients. This may further
lead to patient dissatisfaction and complaints especially if a patient as been kept nil by mouth and no
justifiable explanation is provided for the cancellation.

Additionally, I would take the initiative to see if there were any other theatre lists running in the
adjacent theatres and speak to the consultants in charge of those list to see if the workload could be
shared between theatres. Overall, as a prospective core surgical trainee, it is essential to emulate
characteristics of leadership and management and to work in collaboration with ones colleagues as
these are individuals you will have to work with on a daily basis. By means of effective communication
and being receptive to team members, difficult situations as above can be resolved without hindering
the team spirit and patient care.

Furthermore, with establishment of the European Working Time Directive it is difficult to get
opportunities whereby seniors have confidence in you to complete a theatre list and such opportunities
should be sought after and maximised. I would therefore continue with the theatre list and
communicate the above initiatives with the consultant in charge and advise the theatre team that any
further concerns/suggestions can be formally discussed at the theatre-debriefing meeting.

CORE SURGICAL INTERVIEW GUIDE 130


www.surgicalinterview.co.uk
HELPING YOU THROUGH YOUR CORE SURGICAL INTERVIEW 12th EDITION: 2023

3.4 Dealing with Colleagues

How would you react if one of your registrars refused to treat a patient because they were
a known paedophile?

GMC guidance on this directs that if “carrying out a certain procedure or giving advice about it conflicts
with your religious or moral beliefs, and this conflict could affect the treatment that you would provide,
you must explain this to the patient and tell them that they have the right to see another doctor.’ You
must also ensure that the patient has sufficient information to enable them to exercise that right. If it is
not practical for the patient to see another doctor, you would have a duty to ensure that another suitably
qualified colleague was available to take over the role”

In this situation, you have a duty to the patient and to your colleague. Ask one of the other registrars or
consultants to see the patient. Find out more about your colleague’s views, and discuss them with him
if appropriate. If they had strongly held feelings on a particular issue, these could be related to past
personal experiences or traumatic events. You would have a responsibility ensure they have appropriate
support and could advise a referral to occupational health.

You suspect one of your fellow surgical SHOs is suffering from stress. How would you
proceed?

Here there may be a risk to patient, depending on how your colleague is able to manage his stress. If a
colleague can’t carry out their clinical duties due to stress then patient care will suffer. You should discuss
it with the colleague concerned and offer to help them with some of their tasks as long as your own work
wont suffer. Sensitively use examples of when you feel that patient care may have been compromised
and make sure it is understood that your role is supportive, rather than accusatory. Find out what is
happening to cause them stress and if you can suggest solutions such as taking some time off, organising
their day differently, changing teams, or that they take on less work. Suggest that they seek help, either
from the consultant involved or from occupational health. If your colleague refused to seek help, you
may have to escalate this to a registrar or consultant if you feel they cannot carry on working. Make
sure your colleague knows that you are planning to do so first as any inappropriate involvement of a
senior team member may cause problems within the team that will be difficult to remedy.

CORE SURGICAL INTERVIEW GUIDE 131


www.surgicalinterview.co.uk
HELPING YOU THROUGH YOUR CORE SURGICAL INTERVIEW 12th EDITION: 2023

3.5 WHO checklist

The WHO surgical checklist has played a major role in improving surgical outcomes in
recent years. What do you know about it?

It is a checklist designed to improve surgical safety. It identifies three phases of an operation, each of
which corresponds to a specific period. Before the induction of anaesthesia; the ‘sign in’, before the
incision of the skin; ‘time out’, before the patient leaves the operating room; the ‘sign out’. In each phase,
a checklist coordinator must confirm that the surgical team has completed the listed task before it
proceeds with the operation.

How would you confirm the site of the operation?

I would check the patient name, hospital number and date of birth, comparing it to the consent form,
WHO checklist and patient band. I would check which site and side the patient has been consented
for, check for a mark indicating side on the patient and confirm this with imagine. This would be done
by the surgeon, anaesthetist and scrub nurse.

Who completes the sign out?

Normally the theatre scrub nurse

What device is used for patient warming?

CORE SURGICAL INTERVIEW GUIDE 132


www.surgicalinterview.co.uk
HELPING YOU THROUGH YOUR CORE SURGICAL INTERVIEW 12th EDITION: 2023

Supplementary Information

The WHO Surgical Checklist was introduced in 2008. It aims to increase the safety of patients
undergoing surgery. It aims to ensure that all staff are identifiable, and that errors involving patient
identity, site/side of operation and the type of procedure performed are avoided.

Before Anaesthesia

The patient is asked to confirm their identity, the consent form and the site of operation are checked,
and the procedure confirmed. The side of operation must be marked. Patient allergies are checked, the
amount of expected blood loss discussed, and anaesthetic safety check completed.

Time Out

Before the first incision entire team pause and one of the members, normally the surgeon or anaesthetist,
will read out the “time out”. Everyone must introduce themselves stating their name and role. The
surgeon and anaesthetist must then confirm the name of the patient and the procedure taking place, its
site and side, and discuss any anticipated critical events, and equipment needed. Antibiotic prophylaxis,
patient warming, VTE prophylaxis (TEDS and Flowtrons), hair removal and diabetic control are all
considered on case by case basis.

Sign out

After the operation, before the patient leaves theatre there is a further check conducted by the scrub
nurses. The instrument, swab and needle counts are checked, any faulty equipment is noted and
specimens are appropriately labelled and sent. The surgeon, anaesthetist and nursing staff then mention
key concerns for recovery.

It is estimated that at implementation of the WHO surgical checklist has reduced surgical complications
by more than one third and reduced deaths by almost a half (from 1.5% to 0.8%) according to a study
carried out in the Netherlands. A surgical safety checklist has become the global standard of care.

Advice

When answering a question that is as broad as this it is important to structure your answer well and
directly answer the question. We also advise that you keep your answer brief. This is because the
examiner will normally have a prescribed list of questions for which you score marks. If you are very
lucky or very good, and your essay style response contains the answers to the questions they were
going to ask then fantastic, you have aced this station. If instead you talk for five minutes and only hit
two of the five answers they are looking for then you will have scored a maximum of 40%, regardless
of how good your answers are. If instead you are concise in your response as in the example above,
then the examiner can go on to ask you the questions that will score marks.

CORE SURGICAL INTERVIEW GUIDE 133


www.surgicalinterview.co.uk
HELPING YOU THROUGH YOUR CORE SURGICAL INTERVIEW 12th EDITION: 2023

3.6 Risk Stratification


Do you know of any classification system that quantifies the risk of death whilst under
anaesthetic?

The most commonly used classification is the ASA (American Society of Anesthesiologists) system which
grades patients from 1(healthy) to 6 (brain dead) and has been validated for use quantifying risk of death
under anaesthetic

Are there any limitations to the ASA?

There are four main limitations to the ASA grading system.

Firstly, there is a grey area between ASA2 and ASA3, namely for patients with a stable disease, which
is neither mild, nor severe, but which is of moderate nature.

Secondly, the classification of patients with 2 or more chronic stable diseases is unclear, especially if
they are of differing severity.

Thirdly, what constitutes a systemic disease can be controversial as technically a recent MI is a local not
systemic disease and therefore doesn't fit into the ASA categories but influences post anaesthetic survival.

And finally, there is no mention of age in the classification, yet this can have an impact on the
incidence of operative mortality.

There are other scoring systems, such as the APACHE II and APGAR score. However, these are more
time consuming than the ASA scoring system

What would be the ASA classification of the following patients?

40 year old female with Diabetes Mellitus – ‘ASA 2’

45 year old male with stable angina – ‘ASA 3’

70 year old female with heart failure – ‘ASA 4’

65 year old male with ruptured thoracic aneurysm – ‘ASA 5E’

Supplementary Information

The ASA grading system was introduced by the American Society of Anesthesiologists to quantify the
risk of mortality during and immediately post anaesthetic. It takes into account the premorbid state of
the patient and general physical status. It was introduced into clinical practice in 1963 with five clinical
categories. This was later expanded to include a further category.

CORE SURGICAL INTERVIEW GUIDE 134


www.surgicalinterview.co.uk
HELPING YOU THROUGH YOUR CORE SURGICAL INTERVIEW 12th EDITION: 2023

ASA 1- A normal healthy patient

ASA 2- A patient with mild systemic disease

ASA 3- A patient with severe systemic disease

ASA 4- A patient with severe systemic disease that is a constant threat to life ASA 5- A moribund
patient who is not expected to survive without the operation

ASA 6- A patient declared to be brain dead whose organs are being removed for donor purposes.

If the patient is an emergency, the physical status classification is followed by an ‘E’, e.g. 3E. Class 5
would normally be an emergency procedure, hence 5E. An emergency is therefore now defined as
existing when delay in treatment would significantly increase the threat to the patient's life or body part.
With this definition, severe pain due to broken bones, ureteric stone or parturition (giving birth) is not
an emergency.

Advice

The candidate here could have mentioned a number of risk stratification systems, however the
question is specifically looking for risk of death under anaesthetic. ASA is one of the simplest
stratification systems, the most commonly used world wide, and the one that all surgeons will have
heard of as it is mentioned specifically during the WHO checklist. The sample answer (above) uses a
good technique for answering questions where there are multiple answers, namely opening with the
structure you are about to use, "there are four...". However during your interview when you are
stressed and prone to your mind going blank, you may be better saying "there are several..." so that
you don't reach the third answer and find you have forgotten the fourth. In the final section, note how
when a direct question is asked the candidate does not waffle, and simply states his answer, showing
confidence and calm under pressure, an key attribute in any surgical trainee.

CORE SURGICAL INTERVIEW GUIDE 135


www.surgicalinterview.co.uk
HELPING YOU THROUGH YOUR CORE SURGICAL INTERVIEW 12th EDITION: 2023

3.7 NCEPOD

What is NCEPOD and how has it impacted on current surgical practice?

NCEPOD stands for the National Confidential Enquiry into Peri-Operative Deaths. It has made
suggestions to improve surgical safety, particularly during emergency surgery. It has for example
suggested that operations should not take place out of hours if they can be avoided. This has changed
practice in the UK. Now each emergency case must be assessed for urgency, and appropriately
positioned on the operating list

What are the CEPOD categories and what are they used for?

There are 4 CEPOD categories or codes that characterise an operation’s urgency. They are immediate,
urgent, expedited and elective. It allows the clinicians and managers who are responsible for allocating
theatre time to prioritise accordingly, and to ensure patient’s are operated on within the appropriate
time frame. It aims to ensure that surgeons are only operating out of hours when appropriate

Are there any procedures carried out by non surgeons that are covered?

Yes, coronary angiography for ACS (carried out by cardiologists) is covered.

What category would a ruptured aortic aneurysm fall under and in practice what would
happen?

It falls into the urgent category - 1. In practice the patient would be stabilised haemodynamically with
fluids and blood products at the same time as theatre was being prepared. If necessary, the case would
be prioritised over the next elective case if it occurred during the day, breaking into an elective list,
otherwise it would be performed as soon as theatre is ready.

Supplementary advice

NCEPOD began as a report looking specifically at what factors could have contributed to perioperative
patient mortality in 1987, and now covers all surgical specialities. It aims to alert doctors and hospital
management to practices which could be improved. It does not audit individual surgeon’s performance
rather focusing on hospital practice in general. The essential message from the initial report was that
patients undergoing surgery, especially emergency surgery after 9pm were at greater risk of
perioperative mortality.

It continues to review practice and cases that cause NCEPOD concern are referred to the Medical
Director of the Trust and the Consultant involved so that appropriate action can be taken.

CORE SURGICAL INTERVIEW GUIDE 136


www.surgicalinterview.co.uk
HELPING YOU THROUGH YOUR CORE SURGICAL INTERVIEW 12th EDITION: 2023

Code Category Description Target Expected Example Typical


time to Location Scenarios Procedures
Theatre

1 Immediate Immediate (A) Next available Ruptured aortic Repair of ruptured


lifesaving or (B) Within operating aneurysm aortic aneurysm
limb or organ- minutes of theatre – Major trauma to Laparotomy/
saving decision to “break-in” to abdomen or thorax thoracotomy for
intervention. operate existing lists if Fracture with major control of
Resuscitation required neurovascular deficit haemorrhage
simultaneous Compartment syndrome Fasciotomy
with surgical Acute myocardial Coronary angioplasty
treatment infraction

2 Urgent Acute onset or Day time Compound fracture Debridement plus


deterioration of Within hours “emergency” Perforated bowel with fixation of fracture
conditions that of decision to list peritonitis Laparotomy for
threaten life, limb operate and or Critical organ or limb perforation
or organ survival; normally once Out-of-hours ischaemia Coronary angioplasty
fixation of resuscitation emergency Acute coronary
fractures; relief completed theatre syndrome
of distressing (including at Perforating eye injuries
symptoms. night)

3 Expedited Stable patient Elective list Tendon and nerve Repair of tendon and
requiring early Within days of which injuries nerve injuries
intervention for a decision to has “spare” Stable & non-septic Excision of tumour
condition that is operate capacity patients for wide range with potential to
not an immediate or of surgical procedures bleed or obstruct
threat to life, limb Day time Retinal detachment Coronary angioplasty
or organ survival “emergency”
list
(not at night)

4 Elective Surgical Planned Elective Encompasses all Elective AAA repair


procedure theatre list conditions not classified Laparoscopic
planned or booked & as immediate, urgent or cholecystectomy
booked in planned expedited. Varicose vein surgery
advance of prior to Joint replacement
routine admission admission Coronary angioplasty
to hospital

Advice

Do not learn all the categories by heart, but know what each means and the type of case that fits into
each. Go on call with your SPR, to see how CEPOD works in practice.

CORE SURGICAL INTERVIEW GUIDE 137


www.surgicalinterview.co.uk
HELPING YOU THROUGH YOUR CORE SURGICAL INTERVIEW 12th EDITION: 2023

3.8 Sterilisation

What is the difference between sterilisation, disinfection and cleaning?

Sterilisation is the eradication of all organisms including bacterial spores and viruses

Disinfection is the eradication of most microorganisms; bacterial spores and some viruses may survive.

Cleaning is the physical removal of obvious dirt and contamination without eradication of any
organisms.

How may surgical instruments be sterilised?

There are several methods used:

Heat can be used, for instance; autoclaving, which uses pressurised steam at a temperature of 134°C for
3-10 minutes or 121°C for 15-30 minutes, is typically used for trays of surgical instruments. Dry heat,
where instruments are kept at 160 degrees for 2 hours, and is only suitable for instruments that can
withstand this, is used for non aqueous liquids. Chemical sterilisation, with chemicals such as
formaldehyde, a liquid sterilisation agent, is used for plastics and ethylene oxide a colourless gas, used
for sutures. Gamma irradiation can be used for catheters and syringes.

Which agents are used to disinfect skin?

• Alcohol, chlorhexidine and iodine are common skin disinfectants.


• Alcohol is effective against gram positive and negative bacteria, but not against fungi or spores.
• Chlorhexidine is effective against bacteria including Staphylococcus aureus.
• Iodine is effective against fungi, bacteria, viruses and spores.

Advice

A simple question that threw many people as they weren’t expecting it. reading around the practice of
surgery, for instance knowing what instruments are, and what they do may come up, although they are
asking this less frequently than previously.

CORE SURGICAL INTERVIEW GUIDE 138


www.surgicalinterview.co.uk
HELPING YOU THROUGH YOUR CORE SURGICAL INTERVIEW 12th EDITION: 2023

3.9 Decision making


What is the most difficult decision you have had to make?

The most difficult decision I’ve ever had to make was giving up my career as an accountant to study
medicine. I weighed up the benefits of staying in my job where I had a good salary and security with
my keen desire to study medicine. I consulted some friends who were doctors and found out what they
thought, spending a week shadowing them at work to see if I would enjoy the clinical side more than
my current desk job. I spoke to my partner, and went through the potential financial and social
implications, and decided that I had to choose to study medicine as it had and remains a life long dream
to be a doctor

Advice

This is a hard question to answer. It was asked at the London interviews in 2010 and 2011. As an SHO
the clinical decisions that you have to make are relatively simple, and often your seniors will help make
it for you. You may find non clinical answers from other aspects of your life, or find something you have
done on the ward suitable. Make it personal, but brief as you should focus on how you made that
decision rather than on explaining the example in detail. Thinking about this before your interview is
essential, otherwise you may be stuck wasting time coming up with a difficult decision, and not talking
about how you made it. The example doesn’t really matter as long as it is a decision, i.e. you could have
chosen one of two or more options. If the choice is obvious then the example is a bad one. The marks
are for how you approached the decision-making process.

Other Examples:

• Moving to the South Coast for my Foundation job away from by base in the North
• Breaking up with a partner (be careful to stay professional!)
• Whether to start tinzaparin in a patient with a metallic heart valve who had a cerebral
haemorrhage
• Applying to this deanery

CORE SURGICAL INTERVIEW GUIDE 139


www.surgicalinterview.co.uk
HELPING YOU THROUGH YOUR CORE SURGICAL INTERVIEW 12th EDITION: 2023

3.10 Rota Dispute


The rota is published and you are working every bank holiday and more on calls than
your colleagues, what would you do?

Normally one of your colleagues will be coordinating the rota. It is important to speak to them early,
and discuss this issue. It may have been an oversight, and one they are happy to rectify. It can be useful
to approach them with solutions, not just problems. Emphasise that you would not make any complaints
or comments personal. If they don’t help then you should discuss with some of your other colleagues,
to see if any of them can help out, and swap on calls to make the rota fairer. The next step is to contact
Medical Human resources who will hopefully be able to suggest a solution. Make it clear that it is
important that the on calls are covered but that you would like it to be done fairly. You may need to
escalate the issue to a consultant, but make sure you have taken the initiative to solve the problem
yourself first.

“I would start by making sure that I was indeed correct in thinking that the rota was unjust. I would
look over the schedule again and ask a friend to review the rota also, making sure that they understood
that this was in confidence. If there was only a slight discrepancy in the rota, for instance in a situation
where someone would have to work one more weekend than others for example because of the number
of weeks or months covered, then I would consider working the shift in question. However, if I felt that
the rota was grossly unfair, I would look to contact the member of staff/colleague responsible for
arranging the rota. I would speak to them with discretion, alerting them to where I felt the unfairness
arose. If it was a simple oversight, which they were happy to rectify, then I would not escalate the
situation any further and would not let this error affect my future relationship with the colleague. If they
were not happy to rectify this, then I would discuss the situation with my other colleagues to see whether
we would be able to reach a fairer agreement between ourselves. If this was not successful, I would look
to escalate the issue to Human resources. They would perhaps be able to find a solution. My concerns
would be magnified if the error meant that I was working more than the prescribed maximum number
of hours allowed by the European Working Time Directive”

This may seem like a straightforward question and so it may be easy to give an answer quickly without
any real thought. The first issue here is clarification that there actually is a problem. A large proportion
of grievances in the workplace actually stem from oversight and misunderstanding, rather than true
malice or intentional injustice. It would be wise in this situation, to firstly take a step back and to ask a
neutral opinion on the issue. The issue of patient safety here is key and should be considered in an
effective answer. If there are concerns about your effectiveness to work as a result of this rota error, that
can impact on patient safety. This must be of utmost importance in your mind and in the answer. Any
conversation with your colleague must be had in private, rather than on the ward and should be handled
in a non-confrontational manner. This question looks to test whether you have the clinical
communication skills to be able to deal with a challenging situation effectively.

If you are going to a consultant or another senior member of staff, it is essential that you have thought
of a solution, (e.g. splitting the excess on-call shifts with other colleagues) rather than just a problem.
The panel will want to see that you are an independent thinker and have the capacity to problem solve.
This should hopefully translate into your clinical practice. The worst answers here will be
confrontational, or at the other end of the scale, overly passive. If you are scheduled to work 7 weekends

CORE SURGICAL INTERVIEW GUIDE 140


www.surgicalinterview.co.uk
HELPING YOU THROUGH YOUR CORE SURGICAL INTERVIEW 12th EDITION: 2023

in a row and you meekly comply with this, you may be avoiding confrontation, but you will also be tired
and likely provide substandard care.

CORE SURGICAL INTERVIEW GUIDE 141


www.surgicalinterview.co.uk
HELPING YOU THROUGH YOUR CORE SURGICAL INTERVIEW 12th EDITION: 2023

3.11 Data Protection


You are the CT1 for a general surgical firm. You walk into your consultant's office to find his
secretary viewing a CT scan on the computer - her sister is a patient under another team at the
hospital and the CT shows metastatic colorectal cancer, the results of which you are aware have not
yet been communicated to the patient.

What do you do?

I would initially seek to clarify the situation by asking the secretary to talk with me somewhere private.
I would endeavour to find out what exactly she had read, and why, in order to ensure that I had not
misread the situation. However, should the situation be as first appeared, I would want to ensure that
the patient's rights were respected. It would be a difficult situation, because of the lack of time to prepare
my approach and the emotions likely to be surrounding the news, but I would explain, in a non-
judgemental manner, that the information was confidential and that it was not appropriate to view the
scan in this way. The news is likely to be distressing for the secretary and so I would want to show
empathy, and reassure her that I would be talking to the consultant and that we would endeavour to
have a family discussion if the patient were willing.

I would escalate the matter by notifying both my consultant and the patient's consultant of what had
happened, in order that a discussion could be held with the patient informing her of the results in a
timely, but appropriate manner. I would explain that I felt she should know that her sister was aware of
the results.

I would try to find out how the secretary had come to view the scan. For example - if the consultant had
left the CT scan up on his screen, or the image-viewing system logged on, then this is a confidentiality
issue that would need to be addressed. I would feel anxious about this, because obviously it can be hard
to bring up an error that a senior may have made - but I would be able to do it by reassuring myself
that it was in the best interests of the patient.

At all times I would be sensitive to the fact that this is likely to be devastating news for patient and family
alike, and so would treat the information with respect and ensure that the individuals were offered the
appropriate support.

I would use this a reminder of the importance of protecting patient confidentiality, endeavouring not to
leave patient-identifiable information where it can be viewed easily by others.

Advice

Try to picture the scene with scenarios like this, then you can summon up genuine empathy for what
someone might feel like in this situation. Talk about how it would make you feel, and why, as well as
what you would do. Uses a structured approach ensuring that you cover the need to gather information
about the situation and others’ involvement, ensure patient safety, escalate where appropriate, and
consider the support that those involved, and yourself, may need.

CORE SURGICAL INTERVIEW GUIDE 142


www.surgicalinterview.co.uk
HELPING YOU THROUGH YOUR CORE SURGICAL INTERVIEW 12th EDITION: 2023

What is the Data Protection Act?

The Data Protection Act is an extremely useful piece of legislation produced in 1988, which contains
rules governing the protection of an individual's personal information.
The main principles can be divided into two main areas. Firstly - the way in which information is stored:
it must not be kept for longer than is necessary, and while being stored it must be kept secure - for
example, use of an encrypted USB stick when being transferred. In addition, it must not be transferred
outside of the UK adequate protection. Secondly - how the information is used.

The information must be relevant, accurate and not excessive. It must be used fairly and lawfully.

In addition, the Data Protection Act allows individuals to access their information, provided the request
is made in the correct way. This includes being specific about the information required, and the request
being in writing. However, there are situations when organisations may be able to withhold information,
such as when it relates to criminal proceedings.

The Data Protection Act is extremely important in medicine. Patients trust us with extremely personal
information, and so we must endeavour to protect it and maintain this trust. The data protection
principles enable us to do this. In addition, in an era of increasingly evidence-based practice, access to
patient information is crucial for research and development. Therefore, it is imperative that we handle
the information correctly.

Advice

Questions like this can be tricky, because you either have heard of the act, or you haven’t. Make sure
that you have! Think about the introductory lectures you have on data protection, information
governance etc. and commit these to memory. Worst comes to the worst and you haven't heard of the
act - talk around the subject. You know that confidentiality is important, so talk about how you enforce
it every day. (Do you collect patient lists at the end of every MDT and shred them? Do you always log
off the computer?). Showing that you can apply the principles is much more important that saying what
year the legislation was passed.

CORE SURGICAL INTERVIEW GUIDE 143


www.surgicalinterview.co.uk
HELPING YOU THROUGH YOUR CORE SURGICAL INTERVIEW 12th EDITION: 2023

3.12 Nurse Practitioners

What is your opinion on nurse practitioners and their role?

Nurse practitioners are taking an increasing role in surgical care. If you have any experience of working
with a good one you will know that for a junior trainee their experience can be very helpful. When you
first start a job, having a nurse who has been there for a long time helps you settle into your job and
provides much needed continuity of care for the patients. Nurses may co-ordinate the administrative
side of an MDT and make sure that patients are not lost to follow up. If done well, they play a distinct
but interlinked role in parallel to the surgical team which provides a more efficient service. A BMJ
systematic review concluded that, “Nurse practitioners can provide care that leads to increased patient
satisfaction and similar health outcomes when compared with care from a doctor. Nurse practitioners
seem to provide a quality of care that is at least as good, and in some ways better, than doctors”

On the other hand, some trainees have found that their training is vastly reduced as a result. For
instance, nurse endoscopists often run the colonoscopy list which previously would have been carried
out by the colorectal SpR. Clinical problems arise when a complication occurs that the nurse cannot
deal with, for instance a bowel perforation during colonoscopy. In this instance it is left to the surgical
registrar to manage the complication of a patient they feel they should have been scoping themselves.

Whatever your opinion or experience, it is essential in any question asking for an opinion to give a
balanced answer, with a conclusion that avoids sitting on the fence. For instance, “I think the role of
nurse practitioners is in supporting patients in a more holistic way than doctors might have time for is
a good idea. It is important that they work well with the doctors, and that the roles do not overlap too
much as this can interfere with training which would have serious implications for the next generation
of consultants”

Advice

You may have limited experience of nurse practitioners. They are advanced practice registered nurses
who have usually completed a graduate level of education and expanded their skill set beyond that
which would be required of a registered nurse. What this means in practice is that often, they will
know more than the new FY1, FY2 or even CT1 about the current management of a condition within
their speciality. In some cases, this can lead to conflict, with doctors feeling that a nurse is questioning
their decisions, or somehow challenging their perceived authority. This shows a misunderstanding of
the way that modern medicine is supposed to work. The oft quoted concept of the ‘multidisciplinary
team’ entails a recognition of the skills of others around you and whilst a clinician may well continue
to lead the team, the views of others should be taken into account. This question is strong in that it
uses personal experience to answer the question. It is not unreasonable to mention character flaws that
you have had to resolve or challenges that you have faced in the past. This shows a good level of self
awareness and reflective practice, both key features of being a safe and competent surgeon.

CORE SURGICAL INTERVIEW GUIDE 144


www.surgicalinterview.co.uk
HELPING YOU THROUGH YOUR CORE SURGICAL INTERVIEW 12th EDITION: 2023

3.13 European Working Time Directive

How will you cope with the restrictions of the EWTD? How will you ensure you get
training?

The EWTD came into full effect in 2009. It restricts doctors to working only 48 hours per week, with a
minimum amount of rest and a maximum number of consecutive hours worked. It has had serous
implications for surgical training, as reducing the amount of time at work without reducing service
commitments has meant less time for training opportunities. In practice most surgical trainees simply
ignore EWTD rather than miss out on training, however, you are at interview, making this difficult to
say.

Less time training may mean less experienced consultants in the future, resulting in poorer quality of
care. Hospitals have had to compensate with shift pattern rotas,, meaning that SHOs in particular are
often on nights, or on compensatory ‘zero days’ and are rarely on the wards looking after the same
patients for an extended period of time. This reduces continuity of care. You may feel less responsibility
when you are busy on call and are asked to see a well but concerned ward patient who wants to talk to
you than if you are regularly there and therefore know that patient, impacting on the patient experience.
In addition it has financial implications as banding of more than 50% no longer exists, and has therefore
reduced junior doctors pay significantly.

The advantages include better rested doctors providing safer care, and indeed a better work life balance
for doctors.

To answer this question well, you need to appreciate the importance of working within EWTD, a legal
requirement, whilst at the same time maximising training opportunities. Be clear that your priority is
patient care and your own medical education, and that you do not feel it is unreasonable to stay late to
go to theatre, or to go in on days off as long as you are not too tired to work when required. Explain
that you intend to be organised and identify the training available early in each rotation by talking to
your consultant. You will set out the targets for training, such as go to theatre and ‘scrubbing in’ in on
a whole list twice a week. Decide on the best lists and clinics to go to with your consultant and get them
on board early so that it is an expectation that you will be there rather than on the wards, or indeed at
home on a random off day. If you are asked what you would do if the wards keep bleeping you because
no one else is available to do a TTO then we recommend you stress that patient care comes first and
that you will find time in between cases or patients in clinic to come do the TTO. If it is an unwell
patient then, obviously you need to attend them, but should discuss this with your consultant so that
service cover can be arranged perhaps with another trainee, so that you both can take full advantage of
training.

CORE SURGICAL INTERVIEW GUIDE 145


www.surgicalinterview.co.uk
HELPING YOU THROUGH YOUR CORE SURGICAL INTERVIEW 12th EDITION: 2023

3.14 Incidence and Prevalence

What do you understand by the terms incidence and prevalence? What is the
relationship between the two?

Incidence and prevalence both measure disease frequency. Incidence measures the number of new cases
in a specified time period in a specified population. Prevalence is a measure of all the cases in a
population at a given point of time.

Using diabetes as an example, if there were 250 cases of diabetes in a population of 10000 and 10 new
cases per year, the incidence is 10/10000 per yr, and the prevalence 250/10000. Disease with rapid
onset and short duration, such as acute pancreatitis or cholecystitis have a low prevalence and high
incidence, whereas diseases which are chronic, such as diabetes or ulcerative colitis have a
comparatively low incidence and high prevalence. Prevalence is dependent on the duration of the
disease and on the incidence of the disease. It can be approximated to the product of a disease incidence
and its mean duration.

What is meant by the terms sensitivity and specificity?

Sensitivity is the probability of testing positive when a disease is truly present Specificity is the
probability of a test screening negative if the disease is truly absent

Advice

This is a very common question - have a succinct answer ready.

Remember, If 100 patients known to have a disease were tested, and 60 test positive, then the test has
a sensitivity of 60% . If 100 patients with no disease are tested and 90 return a negative result, then the
test has 90% specificity.

You could also be asked about positive and negative predictive value, so here are some easy to
regurgitate definitions.

Positive predictive value is the proportion of subjects with positive test results who are correctly
diagnosed as having the disease

Negative predictive value is the proportion of subjects with negative test result who are correctly
diagnosed as not having the disease

The question has previously been asked in conjunction with the screening question below:

CORE SURGICAL INTERVIEW GUIDE 146


www.surgicalinterview.co.uk
HELPING YOU THROUGH YOUR CORE SURGICAL INTERVIEW 12th EDITION: 2023

3.15 Screening

What is screening?

Screening is a strategy used to identify disease or pre-disease in currently asymptomatic individuals. It


is an attempt to identify disease in a population early, to allow earlier intervention, at a stage when the
disease can be treated.

What makes a good screening programme?

The World Health Organisation Principles of Screening suggests that:

• The condition should be an important health problem.

• There should be a treatment for the condition.

• Facilities for diagnosis and treatment should be available.

• There should be a latent stage of the disease.

• There should be a test or examination for the condition.

• The test should be acceptable to the population.

• The natural history of the disease should be adequately understood.

• There should be an agreed policy on whom to treat

• The total cost of finding a case should be economically balanced in relation to medical expenditure
as a whole.

• Case-finding should be a continuous process, not just a "once and for all" project.

Are there any criticisms of screening?

Problems include the huge cost involved, and the use of NHS resources on people without disease or
need for treatment. False positive results (test is positive in someone without the disease) induce needless
further tests and worry in people, whilst false negative (a negative result is given for people with the
condition) miss the disease and falsely reassure. A screening procedure may be unpleasant or
uncomfortable, for instance colonoscopy for colonic carcinoma. Overdiagnosis can be a problem. In
prostate cancer, it has been suggested that screening diagnoses malignancy in many patients who would
have died with, rather than from prostate cancer.

It is subject to bias; for instance lead time bias. In some cases where screening leads to an earlier diagnosis,
but no prolongation of actual lifespan, screening simply introduces a longer period of worry into the life
of a patient and their family, whilst giving no prognostic benefit. Length time bias: slower-growing tumours

CORE SURGICAL INTERVIEW GUIDE 147


www.surgicalinterview.co.uk
HELPING YOU THROUGH YOUR CORE SURGICAL INTERVIEW 12th EDITION: 2023

have a better prognosis than tumours with high growth rates, but screening is more likely to detect
slower-growing tumours, due to a longer preclinical period. Therefore screening may tend to detect
cancers that would not have killed the patient or even been detected prior to death from other causes.
A selection bias may exist as patients who are selected, or who self select for screening trials are those who
are likely to be more healthcare conscious, wealthier and non-representative of the population as a
whole.

In your answer you should know about a current NHE screening programme, for instance, that for
breast cancer, and be able to discuss one or two of the controversies associated with it.

CORE SURGICAL INTERVIEW GUIDE 148


www.surgicalinterview.co.uk
HELPING YOU THROUGH YOUR CORE SURGICAL INTERVIEW 12th EDITION: 2023

3.16 Medical Ethics

What are the four key principles of medical ethics? Where are they used and how can
they come into conflict?

The four key principles are:

• Autonomy

• Non maleficence

• Beneficence

• Justice

‘Autonomy’ relates to the right of a person to make independent decisions concerning their own well
being. This has become of increasing importance as the NHS has shifted from a paternalistic model
(doctor centred) to a patient centred approach, where decisions are made collegiately, rather than on
the say of a senior clinician. Recognition of autonomy is reflected in the need for patient consent before
any invasive procedure or examination.

‘Non maleficence’ means that your intervention should not do harm to your patient [and comes from
the Latin ‘primim non nocere’, which means ‘first, do no harm’. This is reflected in risk-benefit analysis
for every procedure. If the potential for harm outweighs the chance of benefit, then a procedure is not
thought to be in the patient’s best interest.

Beneficence refers to actions that serve the best interest of a patient or more literally, actions that ‘do
good’ for the patient, whilst the principle of justice would suggest that all patients should be treated
equally and without prejudice. In modern practice, this means that doctors should not treat patients
differently based on race, gender or sexual orientation.

Conflict can arise where two of these pillars are in apparent opposition. A classic example of this could
be in a patient who is Jehovah witness and did not want to accept a blood transfusion where clinically
indicated. In this case your responsibility to recognise and respect their autonomy would conflict with
the pillars of beneficence and non maleficence if you believed that withholding the transfusion would
increase the chance of harm.

Some doctors would say that the practice of euthanasia respects a patients autonomy and right to
make their own choices about their own care, whilst others would say that assisting a patient’s death
contravenes the most basic of tenets of ethics, that of ‘primum non nocere’. Equally it could be argued
that by not carrying out the wishes of a patient without the capacity to end their own life, the pillar of
Justice is not applied, because we are discriminating against an individual without the ability to carry
out their own wishes due to a physical impairment.

Advice

CORE SURGICAL INTERVIEW GUIDE 149


www.surgicalinterview.co.uk
HELPING YOU THROUGH YOUR CORE SURGICAL INTERVIEW 12th EDITION: 2023

Ethical questions are relatively common and may involve a clinical scenario. Stations that have
appeared in the past include
Unconscious patient with a ruptured AAA, whose family tell you is a Jehovah’s witness

A patient with learning difficulties needs an operation, can they consent?

Should a 14 year old girl be able to consent to an abortion without the knowledge of her
parents?

Questions concerning ethics are rarely straightforward and do not commonly have a right or wrong
answer. A good answer will look at both sides of an argument and not be too narrow minded in its
approach. Do not try to be too structured in this sort of question. The panel expect you to think freely
and show the humanity that is part of the make up of a good surgeon. It is absolutely acceptable to have
your own view and to hold it strongly, but you must also show the ability to empathise with those with
a differing opinion, even whilst maintaining your standpoint.

A good answer will look at both sides of an argument and not be too narrow minded in its approach.
Do not try to be too structured in this sort of question. The panel expect you to think freely and show
the humanity that is part of the make up of a good surgeon. It is absolutely acceptable to have your own
view and to hold it strongly, but you must also show the ability to empathise with those with a differing
opinion, even whilst maintaining your standpoint.

CORE SURGICAL INTERVIEW GUIDE 150


www.surgicalinterview.co.uk
HELPING YOU THROUGH YOUR CORE SURGICAL INTERVIEW 12th EDITION: 2023

3.17 Statistics

What do you understand by a null hypothesis?

A null hypothesis is a default or general position. An example would be to say that there is no
relationship between two variables or that one outcome was not a result of a certain factor. It is typically
set against an ‘alternative hypothesis’ (also known as a research hypothesis). The two are compared
using one of any number of statistical tests. The aim of the statistical test is not to prove the null
hypothesis, as this is not possible, but instead a set of data can reject a null hypothesis (if the default
position is not shown to be statistically true) or to fail to reject it (when the default position is held)

What is a confidence interval?

A confidence interval indicates the reliability of an estimate. It is based on a sample of quantitative data.
It defines the frequency of a parameter being contained within a certain interval. Example, if the mean
in a population sample was 60, and the upper and lower limits of the 95% confidence interval are 65
and 55 respectively, you can therefore conclude that there is a 95% probability that the true population
mean is greater than 55 and less than 65.

What do you understand by a gaussian distribution?

A Gaussian distribution is more commonly known as a normal distribution and is one of the most
commonly illustrated probability distributions in statistics. It always represents continuous,
quantitative data. It is a distribution where the pattern of distribution above and below the mean is
identical. It forms a classical ‘bell shaped probability density function’ and is commonly encountered
in social and medical science

CORE SURGICAL INTERVIEW GUIDE 151


www.surgicalinterview.co.uk
HELPING YOU THROUGH YOUR CORE SURGICAL INTERVIEW 12th EDITION: 2023

3.18 Clinical governance

What is your experience of clinical governance?

I have been a part of clinical teams where all aspects of clinical governance are demonstrated, but have
not as yet been personally fully involved in all the pillars of clinical governance.

I developed a handbook for the management of non-complex orthopaedic injuries being seen in A&E.
This was based on NICE guidelines concerning these injuries and reflected the fact, that previously
some patients had been receiving inappropriate care in some cases. I performed an extensive literature
review of these common injuries and worked alongside a senior registrar to produce an intranet based
protocol which is now used on a daily basic in the A&E at the hospital. By doing this, I was able to
improve the clinical effectiveness of the A&E department, introducing evidence based practice to an
area where things were previously suboptimal.

When the issue of the management of these injuries was first raised my orthopaedic consultant, I was
unsure whether this was a valid problem. I conducted an audit of the treatment given at that time. I
used current NICE and RCOS guidelines as my standard and found that the department fell below the
expected target in terms of patient readmissions and post operative pain. 3 months after the introduction
of the handbook, I re-audited that same unit and found that there was a statistically significant
improvement in treatment of the cases mentioned in the pamphlet. I feel that my intervention led to a
marked change in practice.

I have taken an active role in teaching and training senior medical students. I am currently completing
a part-time teaching diploma at the University of London and have published a peer reviewed article
on effective clinical communication for International students at Undergraduate level. I teach anatomy
to 3rd and 4th year medical school students once a week. I collect verbal and electronic feedback via a
pro forma that I designed online. The feedback so far has been very good, although my teaching
methods are not perfect, this allows me to subjectively assess the strengths and weaknesses of my style.

I recently completed the European Computer Driving Licence. This has given me relevant skills to be
able to use information technology to improve efficiency in the workplace. I have developed an online
reporting system whereby fellow foundation doctors are able to discuss concerning clinical issues and
questions on an anonymous basis. Anecdotally, this has reduced the incidence of errors made by junior
staff, as we all feel more comfortable discussing issues on which we are not clear. I plan to audit the
results of this pilot in the near future. This forum plays a role in risk management and reducing errors
by junior staff.

I have become involved in rota design in the last year. This arose after a dispute between two
colleagues and I feel that it has improved my organisational skills hugely. The feedback on the
changes that I have made has been excellent, with a reduction in the number of ‘split shifts’ that FY1
and FY2s had to complete, often meaning that they previously finished at 2 or 3am. I believe that I
have had a good exposure to a number of aspects of clinical governance and hope to develop these
interests in coming years.

CORE SURGICAL INTERVIEW GUIDE 152


www.surgicalinterview.co.uk
HELPING YOU THROUGH YOUR CORE SURGICAL INTERVIEW 12th EDITION: 2023

Advice

Clinical governance is a process of quality assurance to ensure that standards of care are improved and
that public accountability is maintained. It is based around 7 pillars which are Clinical effectiveness and
Research, Audit, Risk management, Education & training, the use of IT, Medical staffing and Patient
and public involvement. It is tempting just to reel these off, especially if you happened to have learned
them verbatim, but if you read the question, you will see that it is asking what your experience has been.
Your answer can be framed in terms of these ‘pillars’ and you could even start with a very brief
description of them, but you must detail which experiences you have had that is relevant and how they
make you a stronger candidate in the eyes of the panel.

CORE SURGICAL INTERVIEW GUIDE 153


www.surgicalinterview.co.uk
HELPING YOU THROUGH YOUR CORE SURGICAL INTERVIEW 12th EDITION: 2023

3.19 Consent
You are the surgical Senior House Officer (SHO) on call. You are informed by the staff nurse that
your consultant has asked for you to consent the patient who is next on the operating list because both
he and the registrar are busy with the current case which is already one hour late. You have no
knowledge of the procedure that you have been asked to seek consent for. What do you do?

This is a difficult situation because I cannot take consent in order to maintain patient safety. According
to the GMC guidelines, ideally, the person doing the surgery should obtain consent from the patient.
However, the surgeon can delegate obtaining consent to someone else provided that person is suitably
trained and qualified, has sufficient knowledge of the proposed investigation or treatment and
understands the risks involved, and otherwise act in accordance with the guidance set out by the GMC
and the Department of Health. Unfortunately, in this scenario, I am not suitably qualified to seek
consent.

I will inform the nurse why I cannot obtain consent and take the initiative to look for someone suitable,
for example another registrar or consultant who is competent in that procedure, to ask their help in
consenting the patient. Even if there is no one free to consent the patient and I feel pressured to do so
myself, I still cannot consent because of the GMC guidelines and patient safety implications. I will
explain the situation to my consultant when I meet him and apologise that I was not been able to find
anyone to obtain the consent. I will also convey my enthusiasm to observe him consenting the patient,
assisting in the surgery and reading up on it so that I obtain a good grasp of it and can consent patients
for such a procedure in the future. In my experience, most consultants would be understanding of this.

The consultant gets angry that you did not consent the patient and the theatre list is
further delayed. He shouts at you in front of the nurse, registrar and patient. What do
you do?

In the first instance, I will not say anything because any discussion may result into conflict due to the
emotions of the situation. I will then arrange to meet the consultant in private later that day or the next
day. Once I am with the consultant, I will ask for an explanation for the shouting. I would discuss with
the consultant if he has identified areas of concern about my performance. If so, I would ask his advice
on how I could resolve it. Nevertheless, I would still politely point out that it was not professional to
shout at me in public. I would hope that this would resolve the situation for both of us.

The consultant has been verbally aggressive towards you on more than one occasion
and does not admit that his behaviour was unprofessional. How would you respond to
this?

In this situation, it seems that any further discussion with the consultant would be unproductive and
may even lead to conflict. I would then escalate this to my educational supervisor for advice and support.
If this consultant was my educational supervisor, I would seek advice from other consultants. If they are
not helpful, I would escalate to the clinical director and even the medical director if needed.

CORE SURGICAL INTERVIEW GUIDE 154


www.surgicalinterview.co.uk
HELPING YOU THROUGH YOUR CORE SURGICAL INTERVIEW 12th EDITION: 2023

What does consent normally involve?

Consent is a multistage process that has been described in GMC’s good medical practice in a basic
model:

Condition

The doctor makes a clinical assessment of the patient’s condition based on the history, examination and
investigations and explains the diagnosis as appropriate.

Options

The doctor discusses options for the investigation or treatment of likely benefit using their specialist
knowledge and taking into account the patient’s views and understanding of their condition. The doctor
takes care in explaining the potential benefits and risks of each option (conservative, medical and
surgical), including the option of no treatment.

Informed decision making

The patient weighs up the potential benefits and risks of the various options before communicating their
decision to the doctor.

Since consenting is a process where the patient is empowered to make an informed decision, it can be
time consuming depending on the complexity of the condition and the treatment. Therefore, in the
above scenario, initial consent should have taken place when the patient was seen in clinic or pre-
assessment. On the day of the surgery, consent should then be confirmed before proceeding.

What types of consent forms are used in the NHS?

4 forms are commonly used.

Form 1 Patient agreement to investigation or treatment (adults with capacity)

Form 2 Parental agreement to investigation or treatment (i.e. for children)

Form 3 Patient/parental consent for procedures where consciousness of the patient is not impaired
(i.e. patient is alert throughout the procedure)

Form 4 Adult unable to consent for investigation or treatment (i.e. lack capacity)

Please note that signing a consent form without going through the actual consent process described
above is invalid and would be useless if such a case were taken to the court of law in the event of a
post-operative complication

CORE SURGICAL INTERVIEW GUIDE 155


www.surgicalinterview.co.uk
HELPING YOU THROUGH YOUR CORE SURGICAL INTERVIEW 12th EDITION: 2023

3.20 Taking the initiative

Your consultant, who is the general surgery clinical lead, wants to find out whether the surgical team
is doing venous thromboembolism (VTE) assessment promptly within 24 hours of admission of a
patient and VTE prophylaxis prescribed as appropriate. He delegates this task to you. How would you
investigate this?

I would investigate this by undertaking an audit in the department. I would register the audit with the
Trust audit department after discussion with my consultant. I would first pilot the audit by going
through the medical admission notes and drug charts of the last 5 patients admitted to the ward. Once
I am happy with the data set that needs to be collected for analysis, I will embark on this retrospective
audit looking at the notes and drug charts of all the patients admitted on the ward. I will collate the data
and look for ways of improving compliance with VTE assessment, if low. I will present it at the next
Clinical Governance meeting and suggest my recommendations for improving practice. Following
discussion of the audit and agreement of new recommendations, I would like to repeat the audit cycle
once the changes are implemented. I would do a prospective audit looking at the same data sets so that
I can compare and present the findings to check whether the recommendations have actually led to
improvement of practice.

This question can be answered in many different ways. However you decide to answer, it’s important
to the show initiative any consultant would like to see in the people they are training.

HELPING YOU T

CORE SURGICAL INTERVIEW GUIDE 156


www.surgicalinterview.co.uk
HELPING YOU THROUGH YOUR CORE SURGICAL INTERVIEW 12th EDITION: 2023

3.21 Revalidation

What do you understand by revalidation?

Revalidation is a process by which licensed doctors are required to demonstrate to the GMC that they
are up to date and fit to practice on a regular basis. It is a means to promote and ensure continuing
medical education, standards of practice and fitness to practice. It also helps identify doctors in difficulty
who require additional support.

Since its launch in December 2012, revalidation is a legal requirement for all doctors. This revalidation
process occurs every five years, by having regular appraisals with the employer that are based on the
four domains of GMC’s Good Medical Practice (knowledge, skills and performance; safety and quality;
communication, partnership and teamwork; maintaining trust).

For trainees (StR/SpR), the responsible officer who makes the revalidation recommendation is the
postgraduate dean of the Local Education and Training Board (LETB, previously known as the
deanery). This recommendation is based on the appraisal done at the Annual Review of Competence
Progression (ARCP) after assessing the supporting evidence provided by trainees corresponding to the
four domains of GMC’s Good Medical Practice.

How would you, as a surgical trainee, provide evidence for the revalidation process?

During my surgical training, I would use the Intercollegiate Surgical Curriculum Portfolio (ISCP) to
record evidence of my ongoing training and professional development as well as feedback from 360
degree appraisals. My responsible officer would then assess this evidence at my ARCP before making
a recommendation to the GMC.

CORE SURGICAL INTERVIEW GUIDE 157


www.surgicalinterview.co.uk
HELPING YOU THROUGH YOUR CORE SURGICAL INTERVIEW 12th EDITION: 2023

3.22 Cancelled Theatre List

You are working with the Vascular Team today and you have been called by a nurse to say that the
Triple A that was planned for today has been cancelled as there are no ICU beds. This patient has
been cancelled twice previously for other reasons.

How do you proceed?

Firstly I would confirm with the theatre co-ordinator and inform my Consultant and the rest of the
team (Theatre an ward nurses, SpR, Anaesthetist etc) this was the case.

Then I would call the ICU SpR to see if there was any possible chance that another surgical patient
was well enough to be stepped down from ICU and transferred to HDU or the ward with increased
clinical supervision, thus freeing up a bed.

This is not the case and ICU tells you there are absolutely no beds free

I would go straight to the patient and explain the situation, apologizing that this has happened. If
possible I would ask my Consultant to come with me when I do this. I would re-book the patient on
the next possible theatre list and book an ICU bed at the same time, confirming it myself with the
ICU staff. This would all be done before the patient left the hospital, so they had a future date
planned.

You find out that the original ICU bed had not been booked properly anyway and was a
last minute request. How could you prevent this happening again?

A patient has been cancelled on a list for the third time now, this therefore requires a DATIX (incident)
form to be written, particularly in this case. I would confirm exactly how and ICU bed should be booked
and during the next team meeting relay this information to the rest of the team. If this has happened on
many occasions it may be worth auditing this and reviewing the procedure for booking a bed.

Is there anything else you could do now that the patient has been confirmed as
cancelled?

With this cancellation there leaves a lot of free time on the theatre list. I would se if there were any
inpatients/ emergencies that we could bump onto our list instead, discussing with my Consultant and
theatre Coordinator.

CORE SURGICAL INTERVIEW GUIDE 158


www.surgicalinterview.co.uk
HELPING YOU THROUGH YOUR CORE SURGICAL INTERVIEW 12th EDITION: 2023

Conclusion

This guide aims to de-mystify the core surgical interview by giving you some hints and tips that we
would like to have had before we applied. It has been written by surgeons in the first few years of their
career. Unlike other guides out there, it is based on recent experience of the interview. It is an essential
companion to the surgical interview course, and you will be expected to have read most of it before
attending.

We recommend you start practicing these stations as soon as possible with your colleagues who are also
applying. The more practice you get the more confident you will feel and the better you will be come
interview.

We hope that you have had as much fun reading this guide as we have had writing it!

Don’t forget to visit www.surgicalinterview.co.uk and join us for our course. There are several locations
around the country, but it is again expected to book up quickly. Please secure your place early to avoid
disappointment

Good luck!

The (ever growing) Core Surgical Interview Team

KC Surgical Training LTD

Acknowledgements

With many thanks to all our excellent contributors, we thank you for your time and effort.

For any future contributions, or to get involved in the core surgical interview course, please contact us
at admin@surgicalinterview.co

CORE SURGICAL INTERVIEW GUIDE 159


www.surgicalinterview.co.uk

You might also like